You are on page 1of 118

CONTENT

SR. NO. TOPIC PAGE NO.

1. SINGLE CHOICE QUESTIONS 1 – 40

2. MULTIPLE CHOICE QUESTIONS 41 – 62

3. INTEGER TYPE QUESTIONS 63 – 78

4. COMPREHENSION TYPE QUESTIONS 79 – 95

5. MATRIX MATCH TYPE QUESTIONS 96 – 117

COLLEGES: ANDHERI / BORIVALI / CHEMBUR / DADAR / KALYAN / KHARGHAR / NERUL / POWAI / THANE
SINGLE CHOICE QUESTIONS
1. Number of solutions of Re(z2) = 0 and z  a 2 where z is a complex number and a>0 is
(A) 0 (B) 8 (C) 4 (D) 2

2. The number of points P(x, y) lying inside or on the circle x2 + y2 = 9 and satisfying the equation
tan4x + cot4x + 2 = 4 sin2y, is
(A) 2 (B) 4 (C) 8 (D) Infinite

  dx is equal to
2
x1005  x 2010  1
3.  x 4021

(A)
1  x 2010  1 

12060  x 4020 
 9x  8x
2010 1005

x 2010  1  3  c 
(B)
1  x 2010  1 

12060  x 4020 
 8x  9x
2010 1005

x 2010  1  9  c 
(C) 
1  x 2010  1 

12060  x 4020 
 8x  9x
2010 1005

x 2010  1  9  c 
(D) 
1  x 2010  1 

12060  x 4020 
 9x  8x
2010 1005

x 2010  1  3  c 
4. If a1, a2, a3, .............. are positive numbers in G.P. and x1, x2, x3 ........ are positive real numbers such that
1 1 1

a x1
1 a x2
2 a x3
3  ................ then x1, x2, x3 ............... are in
(A) A.P. (B) G.P. (C) H.P. (D) none of these

5. Circles are drawn through the points (a, b) and (b,  a ) such that the chord joining the two points subtends an
angle of 450 at any point of the circumference. Then the distance between the centres is
(A) 3 times the radius of either circle (B) 2 time the radius of either circle
1
(C) times the radius of either circle (D) 2 times the radius of either circle
2

3
6. Let a and b be positive numbers not equal to 1 and    2 . If atan  > btan  > 1, then which one
2
of the following is always TRUE?
(A) a > b > 1 (B) a < b < 1 (C) b < a < 1 (D) b > a > 1

2 e4

2
7. If the value of the integral
1
e x dx is  , then the value of e
log x dx is

(A) e 4  e   (B) 2e 4  e   (C) 2(e 4  e)   (D) none of these

COLLEGES: ANDHERI / BORIVALI / CHEMBUR / DADAR / KALYAN / KHARGHAR / NERUL / POWAI / THANE # 1
3 3
 sin 1 b   cos 1 b 
8. If the function f  x   x  2bx  x  d , is monotonically increasing, then the range of 
3 2
  
     
is
 3 133   3 3   3 7 3 
(A)  ,  (B)  ,  (C)  ,  (D) none of these
 32 24   32 24   8 8 

9. The range of function f defined as f(x) = 6–x Px–2 is


(A) {1, 2, 3, 4} (B) {1, 2, 3} (C) {1, 2} (D) {2, 3, 4}

10. Two equilateral triangles are constructed from a line segment of length L. If M and m are the maximum and
minimum value of the sum of the areas of two plane figures, then
(A) M = 2m (B) M = 3m (C) 2M = 3 3 m (D) M = 4m

11. If four points P, Q, R, S in the plane be taken and the square of the length of the tangent from P to the circle on
QR as diameter be denoted {P, QR}, then:
(A) {P, RS }  {P , QS }  {Q, PR}  {Q, RS }  0 (B) {P, RS }  {P, QS }  {Q, PR}  {Q, RS }  0
(C) {P, RS }  {P, QS }  {Q, PR}  {Q, RS }  1 (D) {P, RS }  {Q, RS }  {Q, PR}  {P, QS}  0

12. The area bounded by the straight line x   1 and x  1 and the graphs of f ( x) and g ( x ), where f ( x )  x 3
 x5 if  1  x  0
and g ( x )   is
x if 0  x  1
(A) 1/3 (B) 1/8 (C) 1/2 (D) 1/4

13. The least value of the sum of the squares of the roots of the equation, x2  (a  2) x  a  1 = 0 is
(A) 0 (B) 5 (C) 6 (D) none

1  sin 2  cos 2
14. If f () = 2 cos 2 then value of f (11°) · f (34°) equals

1 3 1
(A) (B) (C) (D) 1
2 4 4

15. A variable triangle ABC is circumscribed about a fixed circle of unit radius. Side BC always touches the circle
at fixed point D. If B and C vary in such a way that BD.DC  2 , then the locus of vertex A will lie on straight line
(A) Parallel to side BC (B) Perpendicular to side BC
1  2 
(C) Making an angle of  6 with side BC (D) Making an angle of sin   with side BC
3

16. A groove is in the form of a broken line ABC and the position vectors of these three points are respectively,
2 i  3 j  2 k , 3 i  2 j  k , i  j  k . A force of magnitude 24 3 acts on a particle of unit mass kept at a
point A and moves it along the groove to the point C. If the line of action of the force is parallel to the vector ,
i  2 j  k all along the number of units of work done by the force is :

(A) 144 2 (B) 144 3 (C) 72 2 (D) 72 3

COLLEGES: ANDHERI / BORIVALI / CHEMBUR / DADAR / KALYAN / KHARGHAR / NERUL / POWAI / THANE # 2
17. Equation of one of the lines that pass through (10, –1) and are normal to x 2  4  y  2 , is
(A) 4x + y = 39 (B) 2x + y = 19 (C) x + y = 9 (D) x + 2y = 8

18. n different books (n  3) are put at random in a shelf. Among these books there is a particular book 'A' and a
particular book B. The probability that there are exactly 'r' books between A and B is
2 2( n  r  1) 2(n  r  2) (n  r )
(A) (B) (C) (D)
n ( n  1) n ( n  1) n (n  1) n ( n  1)

19. Let A and B be two fixed points and P, another point in the plane, moves in such a way that k1PA  k2 PB  k3 ,
where k1 , k2 and k3 are real constants. Then which one of the following is locus of P:
(A) a circle, if k1  0 and k2 , k3  0 (B) a circle, if k1  0 , k2  0 and k3  0
(C) an ellipse, if k1  k2  0 and k3  0 (D) a hyperbola, if k2  1 and k1 , k3  0

20. If A and B are square matrices such that B = –A–1 BA then


(A) (A + B)2 = AB + BA (B) (A + B)2 = A2 + B2
(C) (A + B) = 2AB
2
(D) (A + B)2 = A + B

x2 y2
21. Let 'E' be the ellipse + = 1 & 'C' be the circle x2 + y2 = 9 . Let P & Q be the points (1 , 2) and (2 , 1)
9 4
respectively, then
(A) Q lies inside C but outside E (B) Q lies outside both C & E
(C) P lies inside both C & E (D) P lies inside C but outside E

 
22. lim x  x  x  x  x  x  x  is equal to
x   
 
1 1
(A) 0 (B) (C) (D) none of these
4 2
      
23. If a is perpendicular to b and p is a non-zero scalar such that pr  ( r . b)a  c , then r is equal to
           
c (b.c)a a (c.a)b b (a.b)c c (b.c)a
(A) – 2 (B) – 2 (C) – 2 (D) 2 –
p p p p p p p p

24. X has 4 friends A, B, C, D. He wants to invite one or more of them at dinner. If the invited person(s) & X are
sitting around a circular table, then the number of arrangements in which C & D (if both are invited)are sitting
together is
(A) 64 (B) 48 (C) 36 (D) 72

 x
tan (ln x) tan  ln  tan(ln 2)
 2
25.  x
dx is equal to

 
 sec(ln x) 
(A) ln  C (B) ln (sec (ln x)) – C
 sec  ln x  
  
  2

COLLEGES: ANDHERI / BORIVALI / CHEMBUR / DADAR / KALYAN / KHARGHAR / NERUL / POWAI / THANE # 3
 
   sec  ln x  
(C) ln  sec  ln  x tan(ln x )   C
x
(D) ln  C
  2   sec  ln x  x tan(ln 2) 
   
  2 

26. If a, b, c, d, e are positive real numbers such that, a  b  c  d  e  15 and ab 2 c 3 d 4 e5  (120)3 .50 then the
value of a 2  b2  c 2  d 2  e2 is
(A) 40 (B) 45 (C) 50 (D) 55

27. Number of solutions of the equation tan x + sec x = 2 cos x in (– , 2), is


(A) 1 (B) 2 (C) 3 (D) 4

28. If the eccentricity of the hyperbola x2 – y2 sec2 = 5 is 3 times the eccentricity of the ellipse x2 sec2  + y2 =
25, then a value of  is
   
(A) (B) (C) (D)
6 4 3 2

tan  e 2x 1  1
29. If lim = a, then x 3  3ax 2  24a 2 x  28  0 has
ln x  ln 2
x 1/2

(A)1 real & 2 imaginary roots (B) 3 real & equal roots
(C)three real & distinct roots (D) 2 real & equal & 1 distinct roots

n
 x
30. If the coefficients of x7 & x8 in the expansion of 2   are equal, then the value of n is equal to
3  
(A) 15 (B) 45 (C) 55 (D) 56

 cos 2  sin  cos    cos 2  sin  cos  


31. If A =  2 ,B=   are such that, AB is a null matrix, then
 sin  cos  sin    sin  cos  sin 2  

which of the following should necessarily be an odd integral multiple of ?
2
(A)  (B)  (C)  –  (D)  + 

32. The set of value of 'k' for which the quadratic equation 3x2 + 2 (k2 + 1) x + (k2 - 3k + 2) = 0 possesses roots of
opposite sign, is
(A) (-  1) (B) (-, 0) (C) (1, 2) (D) (3/2, 2)

 yi
6  ex 
 
i 1

33. If
i 1
sin 1 xi  cos1 yi  9 , then
6
 x ln(1  x 2 ) 
 1 e
2x  dx is equal to

 xi
i 1

 37  6 6
(A) 0 (B) e6  e6 (C) ln   (D) e  e
 12 

COLLEGES: ANDHERI / BORIVALI / CHEMBUR / DADAR / KALYAN / KHARGHAR / NERUL / POWAI / THANE # 4
34. A circle of variable radius is drawn through the points P(2 , 3) & Q(4 , 6). If from a fixed point R, length
of tangent to this circle is independent of radius of the circle, then
(A) R lies on a concentric circle drawn on PQ as diametre
(B) R lies on perpendicular bisector of PQ
(C) R lies on a line passing through P & Q
(D) Given data is inconsistent

     
35. If a , b , c form a system of vectors & p , q , r form the reciprocal system of vectors, then

   
    

  a  m b  n c     p  m q  n r  is equal to
   
(A) l 2  m 2  n 2 (B) l m  mn  n l (C) 0 (D) None of these

 9
36. Let A(2 , 0) & B  2,   be two fixed points and P be a point on the line 3x + 4y = 1, for which location of
 2
P, AP  BP is maximum ?

3 7
(A) (-5, 4) (B) (3, -2) (C)  ,   (D) (7, -5)
2 8

37. Number of ways in which four prizes can be given away to 8 students, when a student can get any number of
prizes is
(A) 8C4 × 4! (B) 84 (C) 48 (D) (8!) (4!)

dy
If y  tan  log  ex6   e / x    tan  log  e/ x12   e x   then
1  2 3  1  4 3 
38. is equal to
    dx
1
(A) 3tan–1(log x) (B) 0 (C) (D) none of these
2

1
39. The minimum value of f  x    t t  x dt, x  R , is
0

1 1  1 1 1 1
(A)  1  (B) (C) (D) 
3 2 6 3 3 2

40. Let Pn is the probability. that n throws of a die contain an odd number of sixes, then
(A) Pn  2 Pn1 (B) 6 Pn  5 Pn1  1 (C) 4 Pn  3Pn1  1 (D) 6 Pn  4 Pn1  1

 3 17
41. The sum cos  cos  .......  cos equals
9 9 9
(A) 1/2 (B) 1/ 2 (C) 1 (D) 1

42. Radius of the circle touching the parabola y 2  x at (1, 1) and having the directrix of the given parabola as a
diameter is
1 3 5 5 5
(A) (B) (C) (D)
2 2 2 4
COLLEGES: ANDHERI / BORIVALI / CHEMBUR / DADAR / KALYAN / KHARGHAR / NERUL / POWAI / THANE # 5
dy(n)
If y  n   e x .e x .e x ...e x , 0 < x < 1. Then nlim
2 3 n
43. at x = 1/2 is
 dx
(A) e (B) 4e (C) 2e (D) 3e

44.  ,  and  are the angles of triangle. If sin   sin   sin   cos   cos   then  is equal to

(A) sec1  2  (B) cos ec 1  2 (C) cot 1  0  (D) tan 1  


2 1

45. If A is an idempotent matrix satisfying, (I – 0.4A)–1 = I – A, where I is the unit matrix of the same order as that
of A then the value of  is equal to
(A) 2/5 (B) 2/3 (C) – 2/3 (D) 1/2
x2 y2
46. The line, lx + my + n = 0 will cut the ellipse 2 + = 1 in points whose eccentric angles differ by /2 if
a b2
(A) a2l2 + b2n2 = 2 m2 (B) a2m2 + b2l2 = 2 n2 (C) a2l2 + b2m2 = 2 n2 (D) a2n2 + b2m2 = 2 l2

47. If the roots of a(b - c) x2 + b (c - a) xy + c (a - b) y2 are real and equal, then


(A) a, b, c are in A.P. (B) a, b, c are in G.P. (C) a, b, c are in H.P. (D) c, a, b are in H.P.

48. If the point representing the complex number za is a point on or inside the circle having centre 1 and radius a then
maximum value of z1  z 2  .........  z n is (where a  1, n  )
n(n  3) n(n  1) n(n  1)
(A) (B) (C) (D) None of these
2 2 2
dy
Let y(x) = 0 be the general solution of x  x  1  y  x 2  x  1 . Then 4 y  2   y  1 equals
2
49.
dx
(A) 0 (B) 4 (C) 2 (D) 6

50. The number of functions f : A  B , where A = {1, 2, 3, 4, 5} to B = {a1, a2, a3, a4, a5} such that f(i)ai,
i = 1, 2, 3, 4, 5, is
(A) 1025 (B) 512 (C) 1024 (D) none of these

7 3 15
51. The value of cot  2 cot  cot is.
16 8 16
(A) – 2 (B) – 3 (C) – 4 (D) – 6

52. A variable straight line passes through the points of intersection of the lines, x + 2y = 1 and 2x - y = 1 and meets
the co-ordinate axes in A & B . The locus of the middle point of AB is
(A) x + 3y - 10xy = 0 (B) x - 3y + 10xy = 0
(C) x + 3y + 10xy = 0 (D) none

53. Let a circle touch the side BC at P and AB & AC produced at Q & R respectively. If the sides and angles of
the  ABC satisfy the relation (b + c)cosA + (c + a)cosB + (a + b)cosC = 20, then length of tangent from A
to the circle is
(A) 20 (B) 10 (C) 40 (D) none of these

 d2 y 
54. If  y  x
2

2 1/ 2
 y  x 2

2 1/2
 2 , then 2y  2  
1/2 3

 dx 
(A) 3x 2  x 6 (B) 3x 2  2x 6 (C) 3x 2  x 6 (D) x 2  2x 6

COLLEGES: ANDHERI / BORIVALI / CHEMBUR / DADAR / KALYAN / KHARGHAR / NERUL / POWAI / THANE # 6
n 1  tan 2 ( 2 n )     
55. Let fn() =   2 n

2  where |  | < n  2 , then the value of f10  13  is__
n 1  (1  tan ( 2 ))  2 2 

   
(A) 1+ cos (B) 1 + cos (C) 1 + cos (D) 1 + cos
210 211 212 213


56. A rigid body rotates above an axis through the origin with an angular velocity 10 3 radians/sec. If  points in
the direction of i  j  k then the equation to the locus of the points having tangential speed 20 m/sec. is :
(A) x2 + y2 + z2  x y  y z  z x  1 = 0 (B) x2 + y2 + z2  2 x y  2 y z  2 z x  1 = 0
(C) x2 + y2 + z2  x y  y z  z x  2 = 0 (D) x2 + y2 + z2  2 x y  2 y z  2 z x  2 = 0

57. For real number ' x ', let  x  denote the largest integer smaller than or equal to ' x ' and  x  denote the smallest
4
integer greater than or equal to ' x '. Also for x in [0, 4], let f  x   min  x   x  ,  x   x  , then  f  x  dx 
4
(A) 2 (B) 8 (C) 4 (D) 6

58. The maximum value of the function f (x) = (7 cos x + 24 sin x)(7 sin x – 24 cos x), for every x  R.
625 625
(A) 25 (B) 625 (C) (D)
2 4

 2 1 
x   , x  0
59. If f  x     x 2  , where {x} denotes proper fractional part in the value of x, then which of the
 0 , x0

following is correct?
(A) f is discont. at 0 (B) f is cont. but nondiff. at 0
(C) f   0   0 (D) f   0  exists but can’t be determined

n n n n
60. If  an  and  bn  be two sequence given by a n  x 2  y 2 and bn  x 2  y2 , for n  N then the
value of a1a 2a 3 ......a n is

xy xy x 2  y2 x 2  y2
(A) b (B) b (C) (D)
n n bn bn

61. The variable 'x' stisfying the equation |sin x cos x| + 2  tan 2 x  cot 2 x = 3 , belongs to the interval

     3 
(A) 0,  (B)  ,  (C)  ,   (D) non existent
 3 3 2 4 

62. 3 Normal dice, one red, one blue and one green are rolled all three simultaneously. Let P be the probability that
the sum of the numbers on the red and blue dice is equal to the number on the green die. If P is the written in
lowest terms as a/b then the value of (a + b) equals
(A) 79 (B) 77 (C) 61 (D) 57

COLLEGES: ANDHERI / BORIVALI / CHEMBUR / DADAR / KALYAN / KHARGHAR / NERUL / POWAI / THANE # 7
63. The rectangular hyperbola (x - 1)(y - 2) = 4 meets a circle at the points P(2, 6) & Q(5, 3) and touches it at a
point R. Center of the circle is
3 5 5 3
(A)  ,  (B)  ,  (C) (1, 1) (D) none of these
2 2 2 2

64. Total number of four digit odd numbers that can be formed using 0,1,2,3,5,7 and having distinct digits are
(A ) 216 (B) 375 (C) 400 (D) 192

65.    
Solution of the differential equation y xy  2 x 2 y 2 dx  x xy  x 2 y 2 dy  0 is given by
1 1
(A) 2 log | x |  log | y |  C (B) 2 log | y |  log | x |  =C
xy xy
1 1
(C) 2 log | x |  log | y |  C (D) 2 log | y |  log | x |  C
xy xy

66. If the difference of the roots of the equation, x2 + ax + b = 0 is equal to the difference of the roots of the equation
x2 + bx + a = 0 then :
(A) a + b = 4 (B) a + b = - 4 (C) a - b = 4 (D) a - b = - 4

67. In the range of 0  x  5, then equation (3 + cos x)2 = 4 – 2 sin8x has
(A) exactly one solution (B) exactly three solutions
(C) exactly five solution (D) infinite solutions

68. The parabola y 2  4 x meets a variable line in P & Q such that PQ subtends a right angle at the vertex O.
Minimum area for triangle OPQ is –
(A) 8 (B) 4 (C) 16 (D) 32

69. The greatest coefficient in the expansion of (a + b)n , given that the sum of all the coefficients is equal to
4096, is equal to
(A) 924 (B) 462 (C) 1848 (D) 2048

70. Number of points of extremum of f  x   ln x  8bx  8x 2 in  0,   , is


(A) 0 if b > 1 (B) 1 if b = 1 (C) 2 if b > 1 (D) 2 if b < 1

1  x 2  y 2  z 2 2( xy  z ) 2(zx  y) 
 2 2 2 
71. Let A =  2( xy  z) 1 y  z  x 2( yz  x )  then | A | is equal to
 2(zx  y) 2( yz  x ) 1  z 2  x 2  y 2 

(A) (1 + xy + yz + zx)3 (B) (1 + x2 + y2 + z2)3


(C) (xy + yz + zx)3 (D) (1 + x3 + y3 + z3)2

e

1 ln x 
72.  
1 x ln x

x
dx 

2
(A) 4 e (B) e (C) 2 e (D)
e

COLLEGES: ANDHERI / BORIVALI / CHEMBUR / DADAR / KALYAN / KHARGHAR / NERUL / POWAI / THANE # 8
73. From P(1 , 2) two tangents are drawn to a circle, touching it at A(3 , 1) & B(0 , 0). Equation of this circle may
be
(A) x 2  y 2  3 x  y  0 (B) x 2  y 2  x  7 y  0
(C) x 2  y 2  2 x  4 y  0 (D) x 2  y 2  4 x  2 y  0

        
74. Let F(k) = 1  sin  1  sin (k  1)  1  sin (2k  1)  1  sin (3k  1) 
 2k   2k   2k   2k 
The value of F(1) + F(2) + F(3) is equal to
3 1 5 7
(A) (B) (C) (D)
16 4 16 16

  1  
75. Taken on side A C of a triangle ABC, a point M such that A M = A C . A point N is taken on the side CB
3
   
such that BN = CB then, for the point of intersection X of A B & M N which of the following holds good ?
 1   1   3   
(A) X B = AB (B) A X = AB (C) XN = MN (D) X M = 3 XN
3 3 4

76. The angle between the pair of tangents drawn to the ellipse, 3x2 + 2y2 = 5 from the point (1 , 2) is
 6   12 
(A) tan  6 5   (B) tan 
 5
 (C) tan 
 5
 
(D) tan 12 5 
2

 f ' (2 t ) e
f (2 t )
77. Let f (0) = 0 and dt  5 , then the value of f (4) equals
0
(A) 2 ln 3 (B) ln 10 (C) ln 11 (D) 3 ln 2
    
78. If x = cos  + cos  – cos( + ) and y = 4 sin sin cos   , then (x – y) equals
2 2  2 
(A) 0 (B) 1 (C) – 1 (D) – 2

79. Let f  x  2   f  x  2   2f  x  for all real x , then f(x) is periodic with period
(A) 6 (B) 12 (C) 8 (D) 16

80. Number of numbers greater than 24000 that can be formed by using digits 1, 2, 3, 4,5 when no digit being
repeated is
(A) 36 (B) 64 (C) 84 (D) 112

1 2 3
81. Let f () =  cos ec 2  sec 2  . The least value of f () for all permissible values of , is
2 3 8
31 61 61 61
(A) (B) (C) (D)
12 48 25 24
 /4

  sin x cos 4 x  dx 
6
82.
0

3 3 3
(A) 
10 (D) 9 (C) 11 (D) none of these
2 2 2
COLLEGES: ANDHERI / BORIVALI / CHEMBUR / DADAR / KALYAN / KHARGHAR / NERUL / POWAI / THANE # 9
83. Let A1 , A2 , A3 ,........, A2012 and H1 , H 2 , H 3 ,........, H 2012 are arithmetic and harmonic mean between a and b
A5 H 2008
respectively. If A1006 H1007 is 2013 then A H is
10 2003

(A) 1 (B) 2 (C) 3 (D) 4

84. Adjacent figure represents a equilateral triangle ABC of side length 2 units. Locus of vertex ‘C’ as the side
AB slides along the coordinate axes is
y

B C

x' x
O A
y'

(A) x 2  y 2  xy  1  0 (B) x 2  y 2  xy 3  1
(C) x 2  y 2  1  xy 3 (D) x 2  y 2  xy 3  1  0

85. From a pack of 52 playing cards, face cards and tens are removed and kept aside then a card is drawn at
random from the ramaining cards. If
E1 : The event that the card drawn is an ace
E2 : The event that the card drawn is a heart
E3 : The event that the card drawn is a spade
then which of the following holds ?
(A) 9 P(E1) = 4 P(E2) (B) P(E3) = 4P (E1  E2)
(C) 3 P(E2) = 4 P(E1 E3) (D) P(E2) = 12 P(E1 E3)

86. If the two roots of the equaton, x3  px2 + qx  r = 0 are equal in magnitude but oppositve in sign then
(A) pr = q (B) qr = p (C) pq = r (D) none

1 sin  1
87. The determinant  sin  1 sin 
1  sin  1
(A) can not be equal to three for atleast one value of   R.
(B) is zero for some value of R.
(C) lies in [2, 4]
(D) lies in [–1, 1]

88. If f is a twice differentiable function such that f   0   4 , then the value of

2f (sin x)  3f (sin 2x)  f (sin 4x)


lim is equal to
x 0 x2
(A) 12 (B) 2 (C) 11 (D) none of these

If  a , a  ,  b , b  ,  c , c  &  d , d  are concyclic points, then a + b + c + d is___


2 2 2 2
89.
(A) 1 (B) 0 (C) 1 (D) none of these

90. A plane 2x + 3y + 5z = 1 has a point P which is at minimum distance from the line joining A(1, 0, –3) and
B(1, –5, 7) then the distance AP is equal to
(A) 3 5 (B) 2 5 (C) 4 5 (D) 5 5
COLLEGES: ANDHERI / BORIVALI / CHEMBUR / DADAR / KALYAN / KHARGHAR / NERUL / POWAI / THANE # 10
dx cos x dx
91. If a    2 + C, then the value of a is
f  x  2  sin x
2
f x
(A) 5 (B) 5/2 (C) 5 (D) 3

92. The number of words formed by using all letters of word ‘RANDOM’ in which vowels are in alphabetical order
is
(A) 340 (B) 360 (C) 380 (D) 390

93. Minimum distance between the parabolas 4 x  y 2  8 y  40 & 4 y  x 2  8 x  40 is


(A) 0 (B) 3 (C) 2 2 (D) 2

94. The equation of the normal to the curve x4 = 4y through the point (2, 4) is
(A) x + 8y = 34 (B) x  8 y + 30 = 0 (C) 8x  2y = 0 (D) 8x + y = 20

1 2 2   x  0
95. The equation 1 3 4   y   0 has a solution for (x, y, z) besides (0, 0, 0). The value of k equals
3 4 k   z  0
(A) 0 (B) 1 (C) 2 (D) 3

cot 7.5  tan 7.5


96. The value of is equal to
cot 7.5  tan 7.5
6 2 6 2
(A) (B) (C) 4  2 3 (D) 4  2 3
4 4

97. The plane 3x + y 2z = 1 is rotated about its line of intersection with the plane x = y by a right angle. Its equation
in new position is
(A) 4x  8y  2z  1  0 (B) x  y  z  1  0
(C) 4x  2y  5z  1  0 (D) x  5 y  z  1  0

x 2 cos x  sin 2 x
98.   x  sin x  2
dx =

x cos x x sin x cos x sin x


(A) C (B) C (C) C (D) C
x  sin x x  sin x x  sin x x  sin x

99. The expression sin 27° cos 57° sin 87° simplifies to
sin 9 cos 9 sin 9 cos 9
(A) (B) (C) (D)
4 4 2 2

100. If 2a + b + c = 0 (c  0) then the quadratic equation ax2 + bx + c = 0 has


(A) no root in (0, 2) (B) exactly one root in (0, 2)
(C) both the roots in (0, 2) (D) exactly one root in (0, 1)

101. From a point on a variable line y = x + c, tangents are drawn to the hyperbola x 2  2y 2  2 such that their
x1
chords of contact pass through a fixed point (x1, y1), then is equal to
y1
(A) 2 (B) 3 (C) 4 (D) none of these
COLLEGES: ANDHERI / BORIVALI / CHEMBUR / DADAR / KALYAN / KHARGHAR / NERUL / POWAI / THANE # 11
x2 y2
102. Q is a point on the auxiliary circle corresponding to the point P on  = 1. If T is the foot of the perpendicular
a 2 b2
dropped from the focus S onto the tangent to the auxiliary circle at Q then the  SPT is
(A) isosceles (B) equilateral (C) right angled (D) right isosceles

103. If z1, z2, z3 represents the vertices of an equilateral triangle ABC and
2 2
Z 2  Z1  Z 3  Z1
2 2
 sin  ,  0      , then  is
Z 2  Z 3  2Z1  Z 3  Z 2
 2  5
(A) (B) (C) (D)
4 3 6 6


 1 
104. The value of  ln1  n 2  equals
n 2
(A) – ln 3 (B) 0 (C) – ln 2 (D) – ln 5

sin A sin B sin C c b a


105. In ABC, if   =   , then the value of angle A, is (All symbols used have
c sin B c b ab ac bc
their usual meaning in a triangle.)
(A) 120° (B) 90° (C) 60° (D) 30°

106. The point (4, 1) undergoes the following three transformation successively.
(i) reflection about the line y  x
(ii) transformation through a distance 2 units along the positive direction of x-axis.
(iii) Rotation through an angle of  / 4 about the origin in the anticlockwise direction.
The final position of the point is given by the coordinates
 1 7   1 7 
(A)  ,  (B) ( 2, 7 2) (C)   ,  (D) ( 2, 7 2)
 2 2  2 2
1 1 1
107. The sum   ...... , for n = 10, is equal to :
1 ! ( n  1) ! 2 ! (n  2) ! 1 ! ( n  1) !

210  1 210 210  2


(A) (B) (C) (D) none
10! 10! 10!

108. Midpoint of the chord cut off from the line x  y  3 by circle x 2  y 2  2 x  4  0 is
(A)  2,1 (B)  3,0 (C) 1, 2 (D) none of these

109. Let a red die, a blue die, a green die and a white die are rolled once, the dice being fair. The outcomes on the red,
blue, green and white die denote the numbers a, b, c and d respectively. Let E denotes the event that absolute
value of (a – 1)(b – 2)(c – 3)(d – 6) = 1, then P(E) is
1 1 2 1
(A) (B) (C) (D)
324 648 324 162

110. The number of quadratic expressions with the coefficients drawn from the set { 0, 1, 2, 3} is
(A) 27 (B) 36 (C) 48 (D) 64

COLLEGES: ANDHERI / BORIVALI / CHEMBUR / DADAR / KALYAN / KHARGHAR / NERUL / POWAI / THANE # 12
x 1
111.  dx =
 x  x  1 x 2  1
2

1 x2 1 x2 1
cos C 1
C
(A) 2  x 2  x  1 (B) sin
x2  x 1

1 x2 1 1 x2 1
sin C (D) 2sin C
(C) 2  x 2  x  1 2  x 2  x  1

1  cos C
112. In ABC, if A – B = 120° and R = 8r, then the value of equals
1  cos C
(All symbols used have their usual meaning in a triangle)
(A) 12 (B) 15 (C) 21 (D) 31

113. Suppose that f is a twice differentiable real valued function such that f   x   0 for all x in [a, b]. Value of c in
[a, b] at which the area between y = f(x), the tangent to the curve at x = c and the ordinates x = a & x = b is
minimum, is
(A) (a + b)/2 (B) (a b)/2 (C) 3(a + b)/2 (D) 3(a b)/2

 


114. Let   R and  sin 2   a. The value
k
  cot  2    cot  2   sin  2   equals
3 k k 4 k

k 2 k 0

a a a a
(A) (B) (C) (D)
2 4 8 16

x
ln t 1
115. Let f  x    dt, then f  x   f   
1
t 1 x

 ln x   ln x 
2 2

 ln x  (D)   ln x 
2 2
(A) (B)  (C)
2 2

116. If the line x  y  z intersects the lines sin A.x  sin B. y  sin C.z  2d 2 , sin 2 A.x  sin 2 B. y  sin 2C.z  d 2
A B C
then sin sin sin = (where A  B  c   c )
2 2 2
(A) 1/32 (B) 1/8 (C) 1/16 (D) 3/16

117. If a, b, p, q are non-zero real numbers, the two equations, 2 a2 x2 - 2 ab x + b2 = 0 and p2 x2 + 2 pq x + q2 = 0


have
(A) no common root (B) one common root if 2a 2  b 2  p 2  q 2
(C) two common roots if 3pq = 2ab (D) two common roots if 3qb = 2ap

118. The equation x  y  a , a  0 represents


(A) part of a parabola (B) an ellipse (C) a hyperbola (D) a line segment

COLLEGES: ANDHERI / BORIVALI / CHEMBUR / DADAR / KALYAN / KHARGHAR / NERUL / POWAI / THANE # 13
119. The radii of the escribed circles of ABC are ra , rb and rc respectively. If ra + rb = 3R and rb + rc = 2R, then
the smallest angle of the triangle is

(A) tan–1  
2 1 (B)
1 –1
2
tan  3 (C)
1
2
tan–1  
2 1 
(D) tan–1 2  3 
120. Let Dk is the k × k matrix with 0's in the main diagonal, unity as the element of 1st row and  f (k ) th column and
k for all other entries. If f (x) = x – {x} where {x} denotes the fractional part function then the value of det.
(D2) + det. (D3) equals
(A) 32 (B) 34 (C) 36 (D) none

121. Let ' M ' be a 3  3 matrix such that  0 1 2 M  1 0 0 and 3 4 5 M   0 1 0 then

6 7 8 M is equal to
(A)  2 1 2 (B)  0 0 1 (C)  1 2 0 (D) 9 10 8

122. Let f  x  y  f  x  y   f 2  x  , f  0   0 & f '  0  f  0   0 . Number of points of extremum of f(x) is


(A) 0 (B) 1 (C) 2 (D) 3

123. The number of ways in which 10 persons can sit around a table so that they do not have same neighbour(all at
once) in any two arrangements ?
1 1
(A) 9! (B) (9!) (C) 10! (D) (10!)
2 2

124. In ABC, the bisector of the angle A meets the side BC at D and the circumscribed circle at E, then DE equals
A A A A
a 2 sec a 2 sin a 2 cos a 2 cosec
(A) 2 (B) 2 (C) 2 (D) 2
2( b  c ) 2(b  c) 2( b  c ) 2(b  c)

1 2
3  2x
125.  tan 2 dx  k tan1x dx  0 , then k =
1

0
x  3x 1 0
(A) 1 (B) 2 (C) 3 (D) 4

5
126. For a triangle ABC, R = and r = 1. Let I be the incentre of the triangle and D, E and F be the feet of the
2
ID ·IE ·IF
perpendicular from I to BC, CA and AB respectively. The value is equal to
IA ·IB ·IC
(A) 5/2 (B) 5/4 (C) 1/10 (D)1/5

127. Let a real valued fumction satisfies, 10  f  x   ln f  x    x 3 for all positive x . Number of solutions of

f  x   f 1  x  , x  A , where A is the set of values of x for which f(x) is invertible, is


(A) 0 (B) 1 (C) 2 (D) 3

COLLEGES: ANDHERI / BORIVALI / CHEMBUR / DADAR / KALYAN / KHARGHAR / NERUL / POWAI / THANE # 14
128. Point 'O' is the centre of the ellipse with major axis AB & minor axis CD. Point F is one focus of the ellipse. If
OF = 6 & the diameter of the inscribed circle of OCF is 2, then the product (AB)(CD) is
(A) 65 (B) 63 (C) 64 (D) 49

129. Centre of the circle meeting each of the three circles x 2  y 2  1 , x 2  y 2  x  2 y  4 &
x 2  y 2  2 x  y  2  0 orthogonally, is
(A) (1, 2) (B) (3, 3) (C) (3, 0) (D) (2, 1)

 A  B
130. In a triangle ABC, C = . If tan   and tan   are the roots of the equation ax2 + bx + c = 0 (a  0) then
2 2 2
(A) a + b = c (B) b + c = a (C) a + c = b (D) b = c

131. Two cubes have their faces painted either red or blue. The first cube has five red faces and one blue face. When
the two cubes are rolled simultaneously, the probability that the two top faces show the same colour is 1/2.
Number of red faces on the second cube, is
(A) 1 (B) 2 (C) 3 (D) 4

132. If the equation x2 + 2 (k + 1) x + 9k – 5 = 0 has only negative roots, then


(A) k  0 (B) k  0 (C) k  6 (D) k  6

133. Solution of xdy   y 2  xy  y  dx is


1 x x
(A) y  (B) y  (C) y  (D) none of these
1  Ce x 1  Ce x Ce x  1

134. a, b, c are distinct and non zero real numbers in arithmetic progression. If the roots of the quadratic equation
1 1
ax 2  bx  c  0 are  and  such that  ,    and  2   2 are in geometric progression, then the
 
a
value of is
c
(A) 1 (B) 2 (C) 3 (D) 4

135. A circle is inscribed inside a regular pentagon and another circle is circumscribed about this pentagon. Similarly
a circle is inscribed in a regular heptagon and another circumscribed about the heptagon. The area of the regions
between the two circles in two cases are A1 and A2 respectively. If each polygon has a side length of 2 units then
which one of the following is true?
5 25 49
(A) A1 = A (B) A1 = A (C) A1 = A (D) A1 = A2
7 2 49 2 25 2

136. The number of ways in which the letters of the word ‘PROPORTION’ can be arranged without changing the
ralative positions of vowels and consonants is
6! 5! 6! 4! 10 ! 10 !
(A) (B) (C) (D)
2!2! 4! 2 !2! 3! 3 ! 2 ! 2! 3! 3 ! 2! 2!

       1
137. A parallelogram is constructed on 5a  2 b and a  3b , a  2 2, b  3, a .b  , then length of smaller
2
diagonal is
(A) 15 (B) 593 (C) 16 (D) 953
COLLEGES: ANDHERI / BORIVALI / CHEMBUR / DADAR / KALYAN / KHARGHAR / NERUL / POWAI / THANE # 15
138. If x 1  f (x)  f (x) 1  x  0 , then f(x) has
(A) exactly one local min. (B) exactly one local max.
(C) no extremum (D) exactly one minimum and two maxima

139. PQ and RS are two perpendicular chords of the rectangular hyperbola xy = c2. If O is the centre of the
hyperbola, then the product of the slopes of OP, OQ, OR and OS is equal to
(A) –1 (B) 1 (C) 2 (D) 4

140. Let the vertices of a quadrilateral ABCD be A(-1, -1), B(2, 0), C(3, 1) & D(4, 4). A If algebraic sum of
perpendiculars from the vertices on a variable line is zero, then this line will necessarily pass through
8 4
(A) (4, 2) (B) (2, 1) (C)  ,  (D) (0, 0)
3 3

141. If the angles subtended by the sides of a triangle at orthocentre and incentre are equal,then the triangle is
(A) Scalene (B) Isosceles but not equilateral
(C) Equilateral (D) Obtuse angled

142. If f: R  R is a function such that f  x   x 3  x 2 f  1  xf   2   f   3 for x  R then the value of f(2) is
(A) 5 (B) 10 (C) 6 (D) –2
dy
143. Solution of 2  tan x sin 2y  sec x 1  cos 2y  is
dx
(A) tan y tan x  sec x  C (B) tan y sec x  sec x tan x  C
(C) sec y sec x  tan x  C (D) tan y sec x  tan x  C

144. The last two digits of the number 3400 are


(A) 81 (B) 43 (C) 29 (D) 01

145. Let P, Q & R be three points of the parabola y 2  2 y  4 x  5  0 such that normals at these are concurrent in
(9, 5). Circum centre of triangle PQR is
(A) (6, 2) (B) (6, 2) (C) (1, 3) (D) None
a a  b a  2b
146. The value of the determinant a  2b a a  b is
a  b a  2b a
(A) 9a2 (a + b) (B) 9b2 (a + b) (C) 3b2 (a + b) (D) 7a2 (a + b)

147. Let there exist a unique point P inside a  ABC such that  PAB   PBC   PCA   .
If PA = x, PB = y, PC = z,  = area of  ABC and a, b, c, are the sides opposite to the angle A,B,C respectively,
then tan  is equal to
a 2  b2  c2 a 2  b2  c2 2 4
(A) (B) (C) (D)
4 2 a  b2  c2
2
a  b2  c2
2

4 x2 y2
148. A tangent having slope of  to the ellipse + = 1 intersects the major & minor axes in points A & B
3 18 32
respectively. If C is the centre of the ellipse then the area of the triangle ABC is
(A) 12 sq. units (B) 24 sq. units (C) 36 sq. units (D) 48 sq. units

COLLEGES: ANDHERI / BORIVALI / CHEMBUR / DADAR / KALYAN / KHARGHAR / NERUL / POWAI / THANE # 16
149. Consider the equation x2 + x – n = 0, where a is an integer lying between 1 to 100. Total number of different
values of ‘n’ so that the equation has integral roots, is
(A) 6 (B) 4 (C) 8 (D) none

150. Let f(x) = (x 1)n (x 2)m, where m & n both are even natural numbars. f(x) has
(A) exactly one local minimum (B) exactly one local maximum
(C) exactly one maximum and two minima (D) exactly one minimum and two maxima

151. If z1, z2 be two complex numbers  z1  z 2  satisfying z1  z 2 = z1  z2  2z1 z2 , then which of the following
2 2 2 2

may be correct

(A) |argz1 – argz2| =  (B) |argz1 – argz2| =
2

(C) z1 is purely real (D none of these


z2

152. Letters of the word ‘WATER’ is arranged in all possible ways and written as dictionary, then rank of the word
‘WATER’ is
(A) 99 (B) 100 (C) 101 (D) 102

153. In a  ABC with usual notations, if r = 1, r1 = 7 and R = 3, then the  ABC is


(A) equilateral (B) acute angled which is not equilateral.
(C) obtuse angled. (D) right angled.

154. If the orthogonal square matrices A and B satisfy A  B  0 , where |A| denotes determinant of matrix A, then
the value of |A + B| is neccessarly
(A) 0 (B) 1 (C) 1 (D) none of these

155. A committee of three persons is to be randomly selected from a group of three men and two women and the
chair person will be randomly selected from the committee. The probability that the committee will have exactly
two women and one man, and that the chair person will be a woman, is/are
(A) 1/5 (B) 8/15 (C) 2/3 (D) 3/10

x 2 y2 x2
156. A circle of radius 4 units is drawn through the points in which the hyperbola   1 & the ellipse 2  y 2  1
3 2 a
intersect. Length of major axis of the ellipse is
(A) 6 (B) 2 6 (C) 4 6 (D) none of these

157. Reflection of plane 2x + 3y + 4z + 1 = 0 in plane x + 2y + 3z – 2 = 0 is


(A) 6x – 19y + 32z = 47 (B) 6x + 19y + 32z = 47
(C) 6x + 19y + 16z = 47 (D) 3x + 19y + 16z = 47

16
158. Least value of the function, f  x   2  3 
2
x
x2 is
2 1
(A) 0 (B) 10 (C) 8 (D) 1

COLLEGES: ANDHERI / BORIVALI / CHEMBUR / DADAR / KALYAN / KHARGHAR / NERUL / POWAI / THANE # 17
159. The condition that the parabolas y 2  4ax & y 2  4c  x  b  , a, b, c being distinct positive real numbers, have
a common normal, other than the axis, is
b b
(A) a  c  (B) a  c  (C) a  c  b (D) a  c  b
2 2

1 1
160. Area of region bounded between the curves y  2 &
y  1 .
1 x 1 x2
2  1
(A)   2 (B) (C) (D)   1
2 2

161. A variable circle cuts each of the two circles x2 + y2  2x = 0 & x2 + y2  4x  5 = 0 orthogonally. The variable
circle passes through two fixed points whose co-ordinates are
 5  3   5  3 5   5  5 3   5  5 
(A)  , 0 (B)  , 0 (C)  , 0 (D)  , 0
 2   2   2   2 

162. Number of straight lines equidistant from three non collinear points in the plane of the points equals
(A) 0 (B) 1 (C) 2 (D) 3

163. Equation x2 + x + 2a = 0 will have exactly one root in the interval (0, 1] if
(A) –1  a < 0 (B) –2 < a < –1 (C) –1  a < 1 (D) 0  a < 1

164. The set of values of p for which both the extrema of the function, f  x   x 3  3px 2  3  p 2  1 x  1 lie in the
interval ( 2, 4) is
(A) ( 3, 5) (B) ( 3, 3) (C) ( 1, 3) (D) ( 1, 5)

165. The number of ordered triplets of positive integers which satisfy the inequality 20  x + y + w  50 is
(A) 50C3 – 19C3 (B) 50C2 – 19C2 (C) 51C3 – 20C3 (D) none of these

b 2  3  2 1
166. In a  ABC, if A = 30° and  , then the measure of C, is
c 2  3  2 1
1 1
(A) 75° (B) 15° (C) 52 (D) 97
2 2

167. In a square matrix A of order 3 the elements, ai i's are the sum of the roots of the equation x2 – (a + b)x + ab = 0;
ai , i + 1's are the product of the roots, ai , i – 1's are all unity and the rest of the elements are all zero. The value of
the det(A) is equal to
(A) 0 (B) (a + b) 3 (C) a3 – b3 (D) (a2 + b2)(a + b)

x2 y2
168. The tangent at the point '' on the ellipse 2 + 2 = 1,  a  b  , meets the auxiliary circle in two points which
a b
subtends a right angle at the centre, then the eccentricity 'e' of the ellipse is given by the equation
(A) e2 (1 + cos2 ) = 1 (B) e2 . (cosec2 1) = 1
(C) e (1 + sin ) = 1
2 2
(D) e2 (1 + tan2 ) = 1

COLLEGES: ANDHERI / BORIVALI / CHEMBUR / DADAR / KALYAN / KHARGHAR / NERUL / POWAI / THANE # 18
169. If a variable line drawn through the origin meets at least one of the sides of a triangle whose vertices are (1, 2),
(4, 1) & (4, 4), then range of its slope is
(A) (0, 2) (B) [-4, 4] (C) [1/2, 2] (D) [1/4, 2]

170. Equation of a line tangential to the parabola 4 y  x 2  2 x  13 for all values of the parameter ‘t’ is____
(A) tx  y  t 2  t  3 (B) tx  y  t 2  t  3 (C) tx  y  t 2  t  3 (D) tx  y  t 2  t  3

171. 310 20C0 20 C10  39 20 C1 19 C9  ....  20 C10 10


C0 , where n C r denote coefficient of xr in (1 + x)n, is equal to
(A) 10 C10 .310 (B) 20
C10 .310 (C) 20
C10 .210 (D) 20
C10 10 C8

172. Let the line y = mx divides the area bounded by y = x2, x = 0 & y = 4 in two parts. If the area above y = mx is
7 times the area below it, then the value of m is? (0 < m < 2)
(A) 2 (B) 2 (C) 3 4 (D) 1
3 x 2
173. Let f : [0,  )  R be a continuous and strictly increasing function such that f  x    t.f  t  dt, x  0 . The
0
area enclosed by y = f (x), the x-axis and the ordinate at x = 3, is
3
(A) 1 (B) (C) 2 (D) 3
2

174. The equation of the transverse axis of the hyperbola (x – 3)2 + (y + 1)2 = ( 4x + 3y)2 is
(A) 3x  4y = 0 (B) 4x + 3y = 9 (C) 3x – 4y = 13 (D) 4x + 3y = 0

175. Let area bounded by y = x sin x & y = 0 from x = n to x = (n + 1) be An, then
An, An + 1, An + 2 are in
(A) A.P. (B) G.P (C) H.P. (D) none of these

176. A point 'P' is an arbitrary interior point of an equilateral triangle of side 4. If x, y, z are the distances of
'P' from sides of the triangle then the value of (x + y + z)2 is equal to
(A) 3 (B) 12 (C) 18 (D) 48

177. The legs of a right angle triangle are 'a' and 'b'. The line segment of length 'd' connecting the vertex of the right
angle to a point 'P' of the hypotenuse enclose an angle  with the leg a. The quantities a, b, d and  are correctly
related as
1 cos  sin  2 cos  sin 
(A)   (B)  
2d a b d a b
1 cos  sin  2 cos  sin 
(C)   (D)  
d a b d b a

178. Events A and C are independent. If the probabilities relating A, B and C are P (A) = 1/5,
P (B) = 1/6, P(A  C) = 1/20, P(B  C) = 3/8 then
(A) events B and C are independent
(B) events B and C are mutually exclusive
(C) events B and C are neither independent nor mutually exclusive
(D) events A and C are equiprobable

COLLEGES: ANDHERI / BORIVALI / CHEMBUR / DADAR / KALYAN / KHARGHAR / NERUL / POWAI / THANE # 19
 
179. The length of the perpendicular from the origin to the plane passing through three non-collinear points a, b, c is
     
a b c  2 a b c
   
(A)       (B)      
ab  c a  bc ab  bc  ca
  
   a b c 
 
(C)  a b c  (D)      
2 ab  c a  bc

180. If each of the quadratic equations x 2  2ax  b  0, x 2  2bx  c  0 and x 2  2cx  a  0 has real & equal
roots and a  b  c & a  0, b  0, c  0 , then which of these are incorrect?
(A) a, b, c can’t be rational numbers (B) a, b, c can’t be integers
(C) a, b, c can’t be irrational numbers (D) None of these

181. Let I n  
0
/ 2
e n sin  d , n  N , then for k < 1, lim
n 
n k In  
(A) e (B)  (C) 1 (D) e

2n 1
Pn 1 3
182. Let n N satisfy 2n 1
 , then n equals to
Pn 5
(A) 3 (B) 4 (C) 6 (D) None of these

183. If (1 + x + x²)25 = a0 + a1x + a2x² + ..... + a50 . x50 then a0 + a2 + a4 + ..... + a50 is :
(A) even (B) odd & of the form 3n
(C) odd & of the form (3n - 1) (D) odd & of the form (3n + 1)

px
 ; x2
184. Consider the function f  x    x  2 where P(x) is a polynomial such that p  x   0 everywhere
 7 ; x  2

and p(3) = 9. If f (x) is continuous at x = 2, then p(x) is
(A) 2x2 + x + 6 (B) 2x2 – x – 6 (C) x2 + 3 (D) x2 – x + 7

185. If  ,  ,  ,  are real numbers such that        then

1  e    e 2(   ) 1  e   e 2(  ) 1  e   e 2(  )


1  e2   e4  1  e    e2(   ) 1  e    e 2(   ) is
1  e    e2(   ) 1  e 2   e 4 1  e   e 2(  )
(A) negative (B) positive (C) non-negative (D) non-positive

186. If the chord through the points whose eccentric angles are  &  on (x2/a2) + (y2/b2) = 1 passes through the
focus, then the value of (1 + e) tan(/2) tan(/2) is
(A) e + 1 (B) e  1 (C) 1  e (D) 0

187. In ABC, if sin 2A + sin 2B + sin 2C = 4 – 4 cosA cosB, then the triangle is
(A) equilateral. (B) right angled but not isosceles.
(C) isosceles but not right angled. (D) isosceles and right angled.

COLLEGES: ANDHERI / BORIVALI / CHEMBUR / DADAR / KALYAN / KHARGHAR / NERUL / POWAI / THANE # 20
188. Four distinct integers form an A.P. If one of these is equal to the sum of squares of the other three, then sum of
squares of all the four numbers is
(A) 6 (B) 4 (C) 2 (D) 5  2

189. Let g  x  
1
4  2
 
1
2
2

f 2x  5  f 7  x x  R, where f "  x   0  x  R , g (x) is necessarily increasing in

the interval
(A)  2, 2  (B)  2,0    2,   (C)  , 2    0, 2  (D)  , 2    2,  

 sec 2 6x 
190. 12 tan 2x tan 4x tan 6x dx  f  3 6   C , then f (e) is
 sec 4x sec 2x 
(A) e (B) 1 (C) 1/e (D) 1

191. In a  ABC with usual notations, If A = 55°, B = 15°, C = 110° then the value of (c2 – a2) equals.
ab ab
(A) 2ab (B) (C) ab (D)
2 4

192. If b < 0 & a  b  1 , then the roots of the equation x 2  a x  b  0 are


(A) of opposite sign (B) greater than 1 (C) real and negative (D) real and positive

193. There are two parallel lines, one having 10 points and the other having 5 points. The number of triangles formed
with vertices as these points is
(A) 225 (B) 100 (C) 325 (D) 125

194. Drawn from the point (1, 2) are two mutually perpendicular straight lines forming an isosceles triangle
with the straight line 2x + y = 9. Area of the triangle is
(A) 5/2 (B) 10 (C) 5 (D) 25/3

195. Locus of point of intersection of the straight lines (where t is a real variable parameter)
x   t  1 y   t  1  0 & x  t y  2  t  1  0 is
(A) A parabola for all values of t except one value
(B) A straight line for exactly one value of t
(C) a parabola for all real values of t
(D) none of these

 1   1  1  1 
196. A is a 2 × 2 matrix such that A 1 =  2  and A2 1 = 0 . The sum of the elements of A, is
       
(A) –1 (B) 0 (C) 2 (D) 5

197. Radius of the circle which touches the line x + y = 5 at the point (–2, 7) and cuts the circle x2 + y2 + 4x – 6y + 9 = 0
orthogonally is
3 2
(A) 3 2 (B) (C) 3  2 (D) None of these
2

198. The slope of the tangent to curve y = f (x) at any of its point is given by 2x  4. If the curve passes through
the point (2 , 12) then the area of the region bounded by the curve , the x-axis & the line x + 1 = 0 is
(A) 8/3 (B) 3 (C) 7/3 (D) none of these
COLLEGES: ANDHERI / BORIVALI / CHEMBUR / DADAR / KALYAN / KHARGHAR / NERUL / POWAI / THANE # 21
199. A line is drawn from the point P 1,1,1 and perpendicular to a line with direction ratios 1,1,1 to intersect the
plane x  2 y  3 z  4 at Q. The locus of point Q is
y5 x
(A) x  z2 (B)  y5  z2
2 2
x y z
(C) x = y = z (D)  
2 3 5

200. A triangle is formed by the line pair xy + 3x – 2y – 6 = 0 and the line 3x – 2y – 6 = 0. The radius of the circle
inscribed in the triangle is
5  13 6 3

(A) 5  13  (B)
2
(C)
13
(D)
5  13

201. A real valued function satisfies f  x   f  y   x  y , x, y  R , then lim f '  x  =


2 2
x 0
(A) 0 (B) –1 (C) 1 (D) Does not exist

202. A particle starts to travel from a point P on the curve C : z  3  4i  5 , where z is maximum.From P, the
1

3
particle moves through an angle tan 1 about the centre in anticlock wise direction on z  3  4i  5 and
4
reaches at point Q. From Q, it comes down parallel to imaginary axis by 2 units and reaches at point R. Complex
number corresponding to point R in the Argand plane is
(A) (3 + 5i) (B) (3 + 7i) (C) (3 + 8i) (D) (3 + 9i)

203. An urn contains 10 balls coloured either black or red. When selecting two balls from the urn at random, the
probability that a ball of each colour is selected is 8 15 . Assuming that the urn contains more black balls than red
balls, the probability that at least one black ball is selected, when selecting two balls, is
18 30 39 41
(A) (B) (C) (D)
45 45 45 45

204. Let g(x) = ax + b, where a < 0 and g is defined from [1,3] onto [0,2] then the value of
 
cot cos1 (| sin x |  | cos x |)  sin 1 ( | cos x |  | sin x |) is equal to
(A) g(1) (B) g (2) (C) g(3) (D) g(1) + g(3)

   x2  k 
205. Let g : R   0,  is defined by g(x) = cos–1  
 3  1  x 2  . Then the possible values of 'k' for which g is
 
surjective function, is
1   1  1  1 
(A)   (B)   1,   (C)   (D)  , 1
2  2  2  2 

206. Given the base of a triangle and sum of its sides then the locus of the centre of its incircle is
(A) straight line (B) circle (C) ellipse (D) hyperbola

207. Let a & b be integers satsfying a b  ba , 0  a  b . Number of ordered pairs (a, b) is


(A) 1 (B) 2 (C) 0 (D) more than 2

COLLEGES: ANDHERI / BORIVALI / CHEMBUR / DADAR / KALYAN / KHARGHAR / NERUL / POWAI / THANE # 22
 ln x dx
208. If 2   k , then the value of greatest integer less than or equal to k is
0 x 2  e2
(A) 2 (B) 1 (C) 0 (D) none of these

 1 5  1  
209. If    cos 2
,  then which one of the following inequalities hold good?
2

(A) cos  < sin  < cot  < tan  (B) cos  < cot  < sin  < tan 
(C) sin  < tan  < cos  < cot  (D) sin  < cot  < tan  < cos 

210. There are n married couples at a party. Each person shakes hand with every other person other than her or his
spouse. If the total number of hand shakes was 264, then n is
(A) 11 (B) 13 (C) 10 (D) 12

211. Let f be a continuous and differentiable function in  x1 , x2  . If f ( x). f '( x)  x 1   f ( x)  and


4

1
lim  f  x    1 and lim  f  x    . Then the minimum value of  x12  x22  is
2 2

x  x1 x x 2 2
 2 
(A) (B) (C) (D) None of these
6 3 3
e e'
212. If a variable line has intercepts on the coordinate axes as e, e’ where & are the eccentricities of a
2 2
hyperbola and its conjugate hyperbola then the line always touches the circle x2 + y2 = r2 where r =
(A) 1 (B) 2 (C) 3 (D) none of these

213. If the roots of x 3  12x 2  39x  k  0 are in A.P., then the value of k is
(A) 45 (B) 15 (C) 28 (D) none

2
214. If f (x) is an even twice differentiable function then  (x 3 f  x   x f "  x   2) dx =
2
(A) 8 (B) 4 (C) 0 (D) 6

215. The value of sin–1(cos2) – cos–1(sin2) + tan–1(cot4) – cot–1(tan4) + sec–1(cosec6) – cosec–1(sec6) is


(A) 0 (B) 3 (C) 8 – 3 (D) 5 – 16

 
216. If   cos  i sin , then the value of 1    2  ....  n 1 is
n n
  
(A) 1  i tan (B) 1  i tan (C) 1  i cot (D) none of these
2n n 2n

217. A straight line L with negative slope passes through the point (3, 24) and cuts the positive coordinate axes at the
points P and Q respectively. Minimum value of PQ, as L moves, where O is the origin, is
(A) 657 (B) 33 (C) 15 5 (D) none of these

COLLEGES: ANDHERI / BORIVALI / CHEMBUR / DADAR / KALYAN / KHARGHAR / NERUL / POWAI / THANE # 23
2 1 3 4  3  4
218. Let three matrices A = 4 1 ; B = 2 3 and C =  2 3  then
     

 ABC   A (BC) 2   A (BC) 3 


tr(A) + tr   +t     + ....... +  =
 2  r  4  + tr  8 
  
(A) 6 (B) 9 (C) 12 (D) none

f  x, t 

219. Let f : R  R be a continuous bounded function and g  x    dt , then g(x) is
1  t2
(A) continuous everywhere (B) differentiable everywhere
(C) discontinuous at a finite number of points (D) nondiferentiable at at least one point

n
220. If (1 + x)n = C0 + C1 + C2x2 + ...... + Cn xn, then the value of  (k + 1)2 . Ck is
k0
n1 n2
(A) 2 (n + 5n + 4)
2
(B) 2 (n + 5n + 4)
2

(C) 2n  2 (5n + 4) (D) none of these

221. The range of f (x) = cot–1(–x) – tan–1 x + sec–1 x is


  3      3     3      3 
(A)   ,  (B)  ,     ,  (C)  ,  (D)  ,     , 
 2 2 2   2   2 2  2   2 

222. The equation(s) of the common tangent(s) to the parabolas y = x2 + 4x + 8 & y = x2 + 8x + 4 is/are
(A) y = 8x + 4 (B) y = 4x + 2 (C) y = 2x + 1 (D) none of these

x7 y z2 


223. Let P(3, –2, 6) be a point in space and Q be a point on the line   . If PQ is parallel to the plane
3 1 5
 
 
r. i  4j  3k  1 , then PQ is
(A) 3 14 (B) 2 14 (C) 14 (D) 4 14

224. The area bounded by a curve, the axis of co-ordinates & the ordinate of some point of the curve is equal to the
length of the corresponding arc of the curve. If the curve passes through the point P (0 , 1) then the equation of
this curve can be


(A) 2y  x e  e
x x
 
(B) 2x  y e  e
x x
 (C) 2x  y e  e
x

x
 (D) y  e  e   2
x x

225. If a, b and c are three numbers (not necessarily different) chosen randomly and with replacement from the set
{1, 2, 3, 4, 5}, the probability that (ab + c) is even, is
35 59 64 75
(A) (B) (C) (D)
125 125 125 125

226. A person writes letters to his 5 friends and addresses the corresponding envelopes. Number of ways in which
the letters can be placed in the envelope, so that atleast two of them are in the wrong envelopes,is,
(A) 1 (B) 2 (C) 118 (D) 119

   2 
227. Let  = cot–1   ,  = sin–1   and  = sec–1   , then the correct order sequence is
3 4  3 
(A)  <  <  (B)  <  <  (C)  <  <  (D)  <  < 
COLLEGES: ANDHERI / BORIVALI / CHEMBUR / DADAR / KALYAN / KHARGHAR / NERUL / POWAI / THANE # 24
228. Let f & g be two functions both being defined from R  R as follows :
x x x for x  0
f(x) = and g(x) =  2 , then
2 x for x  0
(A) fog is defined but gof is not
(B) gof is defined but fog is not
(C) both gof & fog are defined but they are unequal
(D) both gof & fog are defined and they are equal function

229. A ladder 12 units long slides in a vertical plane with its ends in contact with a vertical wall and a horizontal floor
along x-axis. The locus of a point on the ladder 4 units from its foot will be an ellipse whose latus rectum is
(A) 6 (B) 8 (C) 4 (D) 12

y
f 
dy y x
230. Solution of the differential equation     , where k is an arbitrary constant, is
dx x y
f ' 
x

y k y y k y


(A) f    (B) f    kx (C) f    (D) f    ky
x x x x y x

231. Coordinates of vertex A of the ABC are (1 , 2) and perpendicular bisector of the side AC is 2x + 3y + 5 = 0,
then Coordinates of C can be
(A) () (B) (2 , 6) (C) (0 , 6) (D) (9 , 0)

232. If both the roots of x 2  2  p  7  x  p 2  0 are greater than 1, then the set of values of p is

(A)  3,   (B)  2,  (C)  5,  (D) none

234. If A and B are two square non zero matrices and AB = I, then which of the following is necessarily incorrect?
(A) BA = I (B) A–1 = B (C) B–1 = A (D) A2 = B

   4 
235. Let f : R   0,  be defined as f (x) = sin–1  2  then f (x) is
 6   4 x  12 x  17 
(A) injective as well as surjective. (B) surjective but not injective.
(C) injective but not surjective. (D) neither injective nor surjective.

236. One value of k for which the area of the region bounded by the curve y = 8x2 – x5, the straight lines x = 1 and
x = k and the x-axis is equal to 16/3 is
(A) –1 (B) 3 (C) 2 (D) none of these

5 2
237. If (sin–1a)2 + (cos–1b)2 + (sec–1c)2 + (cosec–1d)2 = , then the value of
2
(sin–1a)2 – (cos–1b)2 + (sec–1c)2 – (cosec–1d)2 is equal to
2 2
(A) – 2 (B)  (C) 0 (D)
2 2

COLLEGES: ANDHERI / BORIVALI / CHEMBUR / DADAR / KALYAN / KHARGHAR / NERUL / POWAI / THANE # 25
(x  1) n
238. Let g(x) = ; 0 < x < 2 and m and n are integers, m > 0, n > 0, and let p be the left hand derivative
log cos m (x  1)
of |x – 1| at x = 1. If lim g(x)  p then
x 1

(A) n = 1, m = 1 (B) n = 1, m – 1 (C) n = 2, m = 2 (D) n > 2, m = n

239. Let the differential equation of a curve passing through the point (0,1) be given by dy  x 2 e x dx . If the equation
of the curve is written in the form x = f(y) then f(e1) is
(A) 0 (B) 1 (C) 2 (D) e2

240. Let f (x) =


2
1 1 1
 sin [x ]  tan [x ]  cot [ x ] 
where [x] denotes greatest integer less than or equal to x. If A and
B denote the domain and range of f (x) respectively, then the number of integers in (A B), is
(A) 1 (B) 2 (C) 3 (D) 4

241. Area of the figure bounded by the curves y  a x & y  1  x and the y  axis is equal to A, where a > 0
is a parameter. For what set of values of A, there always will exist a corresponding value of ‘a’
 2  2 
(A)  0,  (B)  0, 4 / 3 (C) 0,  (D) none
 3  3

a2 1 ab ac
2
242. If a, b, c are real then the value of determinant ab b 1 bc = 1 if
2
ac bc c 1
(A) a + b + c = 0 (B) a + b + c = 1 (C) a + b + c = –1 (D) a = b = c = 0

243. For a game in which two partners oppose two other partners, 8 men are available. If every possible pair must
play with every other pair, the number of games played is
(A) 8C2 . 6C2 (B) 8C2 . 6C2 . 2 (C) 8C4 . 3 (D) none

  x  
244. Number of integral solutions of the equation sgn  sin 1    = 1, where [x] denotes the greatest integer less
  6 
than or equal to x and sgn x denotes signum function of x.
(A) 2 (B) 3 (C) 5 (D) 7

 
    ab   
 
2
245. If a and b are two non collinear unit vectors such that a  b  then the value of a  a  b is equal to
2
1 1 2 3
(A) (B) (C) (D)
4 2 3 4

1  e f ( x)
246. Consider a real valued function f(x) such that  x . If f(x) satisfies 2 f  a   f  2a  , then a must
1  e f ( x)  2 
1 a 
lie in
(A) (1, 1) (B) (1, 0) (C) (0, 1) (D)  , 1  1,  

COLLEGES: ANDHERI / BORIVALI / CHEMBUR / DADAR / KALYAN / KHARGHAR / NERUL / POWAI / THANE # 26
247. If the roots of x3 – kx2 + 14x – 8 = 0 are in G.P. then the value of k is
(A) –3 (B) 7 (C) 4 (D) 9

1
248. If ,  are the roots of the equation, x2 + (sin  1) x  cos2  = 0 then the maximum value of the sum of
2
the squares of the roots is
(A) 3 (B) 4 (C) 5 (D) 2

249. The D.E. of all the circles passing through the origin and having x axis as diameter, is
dy x 2  y 2 dy x 2  y 2 dy x 2  y 2
(A)  0 (B)  0 (C)  0 (D) none of these
dx 2xy dx 2xy dx 2xy

250. The locus of the mid points of a chord of the circle x2 + y2 = 4 which subtends a right angle at the origin is
(A) x2 + y2 = 1 (B) x2 + y2 = 2 (C) x2 + y2 = 3 (D) x + y = 4

251. The angle between the tangents drawn from the point  
7 , 1 to the ellipse 3x2 + 5y2 = 15 is
(A) /6 (B) /4 (C) /3 (D) /2

252. Lot A consists of 3G and 2D articles. Lot B consists of 4G and 1D article. A new lot C is formed by taking 3
articles from A and 2 from B. The probability that an article chosen at random from C is defective, is
(A) 1/3 (B) 2/5 (C) 8/25 (D) none

253. Let the co-ordinates of the two points A & B be (1, 2) and (7, 5) respectively. The line AB is rotated through 45º
in anti clockwise direction about the point of trisection of AB which is nearer to B. The equation of the line in new
position is
(A) 2x  y  6 (B) x  y  1 (C) 3x  y  11 (D) x y  17

254. Sides of a triangle are given by y  x  2, y  2 x  1, 3 y  9 x  2 . If from the orthocentre of this triangle two
tangents are drawn to the parabola y 2  8 x , then the chord of contact will always pass through
(A) (2, 0) (B) (2, 0) (C) () (D) (4, 0)

255. The range of the function f (x) = cos


1

log 4 x   
2
1 x2 
 sin 1 
4 x
 is equal to

 

        
(A)  0, 2  2  (B)  2 , 2  2  (C)  ,  (D)  
    6 2 6

f  x, t 

256. Let f : R  R be a continuous bounded function and g  x    dt , then g(x) is
1 t2
(A) continuous everywhere (B) differentiable everywhere
(C) discontinuous at a finite number of points (D) discontinuous at infinite number of points

x2 y2 
257. A tangent drawn to hyperbola 2
 2  1 at a point with eccentric angle forms a triangle of area 3a2 square
a b 6
units with coordinate axes, then the square of its eccentricity is equal to
(A) 17 (B) 36 (C) 28 (D) 4
COLLEGES: ANDHERI / BORIVALI / CHEMBUR / DADAR / KALYAN / KHARGHAR / NERUL / POWAI / THANE # 27
258. The number 916238457 is an example of nine digit number which contains each of the digit 1 to 9 exactly once.
It also has the property that the digits 1 to 5 occur in their natural order, while the digits 1 to 6 do not. Number
of such numbers are
(A) 2268 (B) 2520 (C) 2975 (D) 1560

259. If a1Z  a1 Z  b1 & a 2 Z  a 2 Z  b 2 represent two mutually perpendicular straight lines, then

(A) a1a 2  b1b 2  0 (B) a1b1  a 2 b 2  0 (C) a1b2  a 2 b1  0 (D) none of these

260. The points A(a, 0) , B(0, b) , C(c, 0) & D(0, d) are such that ac = bd & a, b, c, d are all non - zero. Then
the points A, B, C & D
(A) form a parallelogram (B) do not lie on a circle
(C) form a trapezium (D) lie on a circle

261. Equation of the image of the line x + y = sin–1 (a6 + 1) + cos–1 (a4 + 1) – tan–1 (a2 + 1), a  R about x axis is
given by
 
(A) x – y = 0 (B) x – y = (C) x – y =  (D) x – y =
2 4

3 d2 y
262. If y2 = x2 + x + 1, then 4y is equal to
dx 2
(A) 1 (B) – 3 (C) 3 (D) 0

263. The roots of the equation x 3 – 10x + 11 = 0 are u, v, and w. The value of (tan–1u + tan–1v + tan–1w) equals
(A) – 1 (B) tan 1 1 (C) 1 (D) tan 1  1

1
2A  1  5  A  5 B 14 D
264. For two possible values of A ,   4 A  2A  2 C =  E F  , where A, B, C, D, E, F are real

numbers. The absolute value of the difference of these two solutions, is
8 11 1 19
(A) (B) (C) (D)
3 3 3 3

265. If 3  a < 4 then the value of sin–1(sin [a]) + tan–1 (tan [a]) + sec–1 (sec [a]), where [x] denotes greatest integer
function less than or equal to x, is equal to
(A) 3 (B) 2 – 9 (C) 2 – 3 (D) 9 – 2

266. If n be a positive integer such that n  3, then the value of the sum to n terms of
 n 1  n  1  n  2  n  1 n  2  n  3
S=1.n- (n - 1) + (n - 2) (n - 3) + ...... is
1! 2! 3!
(A) 0 (B) 1 (C) –1 (D) none of these

x2  a2 
267. Let f (x)   dx, f  a   0 & lim f  x   , then a =
x a x a
4 2 2 4 x  6
(A) 2/ 3 (B) 1/ 3 (C) 2 3 (D) 3

COLLEGES: ANDHERI / BORIVALI / CHEMBUR / DADAR / KALYAN / KHARGHAR / NERUL / POWAI / THANE # 28
268. If n1, n2 are positive integers then 1  in1  1  i3 n1  1  i5 n 2  1  i7 n 2 is a real number if and only if
(A) n1 = n2 + 1 (B) n1 + 1 = n2 (C) n1 = n2 (D) none of these

log e (1  3f (x))
269. y = f(x) is a continuous function such that its graph passes through (a, 0). Then lim is
x a 2f (x)
(A) 1 (B) 0 (C) 3/2 (D) 2/3

 
270. The vector equation of a line through the point a , which is parallel to the plane r.b  0 and intersect the line
  
r  c  ld is
     
(A) r  a  [b c – a d]  (where  is a parameter)
     
(B) r  a  {(c – a )  (b  d)}  (where  is a parameter)
     
(C) r  a  {b  ((c – a )  d)}  (where  is a parameter)
(D) none of these

271. If f (x) = x11 + x9 – x7 + x3 + 1 and f (sin–1 (sin 8)) = , is constant, then f (tan–1 (tan 8)) is equal to
(A)  (B)  – 2 (C)  + 2 (D) 2 – 

ln 1  x   ln x 1
272. If  dx  C  f 2  x  , then lim f  x  is equal to
1  x  x 2 x 

(A) 0 (B)  (C)   (D) does not exist

273. If  &  are the eccentric angles of the extremities of a focal chord of an standard ellipse,then the eccentricity
of the ellipse is
cos   cos  sin   sin  cos   cos  sin   sin 
(A) (B) (C) (D)
cos (  ) sin (  ) cos(   ) sin (   )

   
274. The solution set of inequality (cot–1x) (tan–1x) +  2   cot–1x – 3tan–1x – 3  2   > 0, is
 2  2
(A) x (tan 2, tan 3) (B) x (cot 3, cot 2)
(C) x  (–  , tan 2) (tan 3, ) (D) x  (–  , cot 3) (cot 2, )

275. If a positive constant ‘t’ satisfies ex  x t for all positive x, then


(A) t < e (B) t > e (C) t > 1/e (D) none of these

276. A question paper on mathematics consists of twelve questions divided into three parts A, B and C, each
containing four questions. In how many ways can an examinee answer five questions, selecting atleast one from
each part.
(A) 624 (B) 208 (C) 1248 (D) 2304

 33  1  46  1  13  1   19   a


277. If sin 1  sin   cos  cos   tan   tan   cot  cot    , where a and b are in
 7   7   8    8  b
their lowest form, then (a + b) equals
(A) 17 (B) 20 (C) 23 (D) 28

COLLEGES: ANDHERI / BORIVALI / CHEMBUR / DADAR / KALYAN / KHARGHAR / NERUL / POWAI / THANE # 29
278. Consider the cubic x3 – x2 + 3x + 4 = 0 where a, b and c are its roots and let w = tan–1a + tan–1b + tan–1c. If the
m
absolute value of sec  = where m and n are prime numbers, then the value of (m + n) equals
n
(A) 13 (B) 31 (C) 29 (D) 63

279. Mr. A and Mr. B each have a bag that contains one ball of each of the colours blue, green, orange, red and violet.
'A' randomly selects one ball from his bag and puts it into B's bag. 'B' then randomly selects one ball from his bag
and puts it into A's bag. The probability that after this process the contents of the two bags are the same, is
(A) 1/6 (B) 1/5 (C) 1/3 (D) 1/2

280. If the function, f(x) = (cot a) x2 + 2  


sin a x +
1
2
tan a , f : R   ,0  , is an onto function, then the set of
values of a in [0, 2 ] is

(A)  56 ,  (B)  116 , 2  11 


(C)  6 , 2  (D) none of these

1x y
281. If cos  sin 1 =  (a, b  0), then the maximum value of b2x2 +a2y2 + 2ab xy sin equals
a b
(A ) ab (B) (a + b)2 (C) 2(a + b)2 (D) a2b2

282. Let g(x) = ax + b, where a < 0 and g is defined from [1,3] onto [0,2] then the value of
 
cot cos1 (| sin x |  | cos x |)  sin 1 ( | cos x |  | sin x |) is equal to
(A) g(1) (B) g (2) (C) g(3) (D) g(1) + g(3)

283. A variable parabola is drawn touching the x -axis at origin and having its vertex on the line y = 2. The axis of this
parabola will always be tangential to a fixed
(A) circle (B) parabola (C) hyperbola (D) none

 x  1  x2   1 1 
284. Range of the function , f  x   sin 1 x  sin 1  , f : R    ,   R,
 2   2 2
 
(A) contains exactly one element (B) contains exactly two elements
(C) is an infinite set (D) none of these

285. Domain of definition of the function f(x) = 3 cos 1 (4 x )   is equal to

 1 1  1  1 1   1
(A)  ,  (B)  , 1 (C)  ,  (D)  1, 
 4 8 8  8 4   8

286. If a sin–1 x – b cos–1 x = c, then the value of a sin–1 x + b cos–1 x (whenever exists) is equal to
ab  c(b  a )  ab  c(a  b)
(A) 0 (B) (C) (D)
ab 2 ab

287. A real valued function satisfies f  x   x  n if n  x  n  1, n  Z , then f  sin x  is periodic with period
(A)  (B)  (C)  (D) non periodic

COLLEGES: ANDHERI / BORIVALI / CHEMBUR / DADAR / KALYAN / KHARGHAR / NERUL / POWAI / THANE # 30
288. If 0 < cos–1 x < 1 and 1 + sin (cos–1 x) + sin2(cos–1 x) + sin3 (cos–1 x) + ........  = 2, then x equals
1 1 3 1
(A) (B) (C) (D)
2 2 2 2 3

 1 tan x  T –1
289. A =  tan x 1  then let us define a function f (x) = det. (A A ) then which of the following

can not be the value of f f f f ...........f ( x )  , (n  2), is

n times

(A) f n(x) (B) 1 (C) f n – 1(x) (D) n f (x)

290. If the straight lines x + y – 2= 0, 2x – y + 1 = 0 & px + qy – r = 0 are concurrent then the slope of the family of
lines 2px + 3qy + 4r = 0 which is farthest from origin is
1 2 3
(A) (B) –2 (C) (D)
2 3 10

291. The common chord of two intersecting circles c & c can be seen from their centres at the angles of 90º and
1 2
60º respectively . If the distance between their centres is equal to 3 + 1 then the radii of c & c are :
1 2

(A) 3 &3 (B) 2 & 2 2 (C) 2 &2 (D) 2 2 & 4

292. If m denotes the number of 5 digit numbers if each successive digits are in their descending order of magnitude
and n is the corresponding figure, when the digits are in their ascending order of magnitude then (m – n) has the
value
(A) 10C4 (B) 9C5 (C) 10C3 (D) 9C3

   
293. Vector OA = 2 i  j  2k is rotated by 90° about O (origin) such that while rotating it crosses the positive y-
axis. Then new vector may be.
   
 i  k   –i  k 
(A)  –2 2j  (B)  2 2j– 
 2 2  2 2
   
  j k    j k 
(C)  2 2 i – –  (D)  –2 2 i   
 2 2   2 2 

 3  3 6
294. If tan–1  x   – tan–1  x   = tan–1 , then the value of 5x8 – 4x4 + 7 equals
 x  x x
(A) 397 (B) 393 (C) 376 (D) 379

295. A common tangent to 9x2 + 16y2 = 144, y2  x + 4 = 0 & x2 + y2  12x + 32 = 0 is


(A) y = 3 (B) x =  4 (C) x = 4 (D) y =  3

296. From a point P two tangents (including asymptotes) are drawn to the hyperbola 4 x 2  y 2  4 , then the least
value of the angle between these tangents which contain the hyperbola is
3 4
(A)   tan (B)   tan
1 1
(C)   tan 3 (D)   tan 1 4
1
4 3

297. The latus rectum of the conic passing through the origin and having the property that normal at each point (x, y)
intersects the x - axis at ((x + 1), 0) is :
(A) 1 (B) 2 (C) 4 (D) none
COLLEGES: ANDHERI / BORIVALI / CHEMBUR / DADAR / KALYAN / KHARGHAR / NERUL / POWAI / THANE # 31
298. If the lines 2x  k y  1  0 & k 2 x  4y  2  0 meet the coordinate axes in four distinct and real points, which
lie on a circle, then the number of value of k is
(A) 3 (B) 2 (C) 1 (D) none of these

4 4 4 4
299. The value of tan 1 + tan 1 + tan 1 + tan 1 +........  equals
7 19 39 67
1 1
(A) tan 1 1 + tan 1 + tan 1 (B) tan–11 + cot 1 3
2 3
1 1
(C) cot1 1 + cot1 + cot1 (D) cot–11 + tan1 3
2 3

 3 5   2 3 
300. Number of values of x satisfying the equation tan–1  x  x  x  ...... + cot–1  x  x  x  ...... = 
 4 16   2 4  2
   
for 0 < | x | < 2, is
(A) 0 (B) 1 (C) 2 (D) 3

301. A bag contains 3 black and 7 white balls. Three balls are drawn one after other replacing the ball drawn each
time along with two more balls of the same colour. If the third drawn ball is black, then the first drawn is also
black with probability
7 5 1 3
(A) (B) (C) (D)
12 12 4 4

 [x]2  sin[x]
 for[x]  
302. If  [x]
 0 for[x]  0

where [x] denotes the greatest integer less than or equal to x, then lim f (x) equals
x 0

(A) 1 (B) 0 (C) –1 (D) does not exist

f (ln 2012)
x y x s y s cos xesin x
303. If x, y, z, s, t  R; z, t  0 and f     f     f    then
z z z t z t
 1  e2sin x dx is equal to
f   ln 2012 

(A) 1 (B) 22012 (C) 32012 (D) none of these

 sin 1 x 
304. If tan   = 1 then x is equal to

 5 
1 1
(A) 1 (B) (C) – (D) none
2 2
305. Given positive rational numbers a, b, c such that a + b + c = 1, then a a bb cc  a bbc c a  a cba c b is
1
(A)  1 (B)  1 (C)  (D)  3
3

COLLEGES: ANDHERI / BORIVALI / CHEMBUR / DADAR / KALYAN / KHARGHAR / NERUL / POWAI / THANE # 32
1cos 2  cos 2  I1
306. Let I1  2  
x f 2x  x dx & I2 
2
 2 f 1  x  dx , where ' f ' is a continuous function , then I
2

2
=
sin   cos 
(A) 3/2 (B) 1/2 (C) 1 (D) 2/3

nr
307. If  nk
Cr = xCy, then
k1

(A) x = n + 1, y = r (B) x = n, y = r + 1 (C) x = n, y = r (D) x = n + 1, y = r + 1

308. If the straight line y = 2kx + 1 neither touches nor intersects the parabola, y = (k - 6) x2 - 2 then k must lie in the
interval
(A) (6) (B) (6]  [3,) (C) (6,  3) (D) (6, 3)

309. Let A, B, C, D be (not necessarily square) real matrices such that AT = BCD, BT = CDA, CT = DAB and
DT = ABC. For the matrix S = ABCD, consider the two statements.
(I) S3 = S (II) S2 = S4
(A) II is true but not I (B) I is true but not II
(C) both I and II are true (D) both I and II are false.

  n   n 1  
310. The value of   tan 1 n  2   tan 1 n  1   is equal to
n 1  

   3
(A) (B) (C) (D)
4 3 2 4
1
sin xdt
311. Range of f  x    1  2t cos x  t 2 contains
1

         
(A)   ,  (B)   (C)   ,  (D)   , 
 2 2 2  2 2  2 2

tan[e 2 ]x 3  tan[e2 ]x 3
312. Let f(x) = , x  0, the value of f(0) for which f(x) is continuous is
sin 3 x
(A) 15 (B) 12 (C) –12 (D) 14

x 1 y  2 z  3
313. Equation of plane which passes through the point of intersection of lines   and
3 1 2
x  3 y 1 z  2
  and at greatest distance from the point (0, 0, 0) is
1 2 3
(A) 4x +3y + 5z = 25 (B) 4x + 3y + 5z = 50 (C) 3x + 4y + 5z = 49 (D) x + 7y – 5z = 2

314. A circle is touching both the coordinate axes and its centre lies on the curve y 2  4 x , then area of this circle is.
(A) 16  (B) 4  (C) 32  (D) none
a2 x2 y2
315. If tan 1 . tan 2 =  2 then the chord joining two points 1 & 2 on the ellipse 2  2 = 1 will subtend a right
b a b
angle at
(A) focus (B) centre
(C) end of the major axis (D) end of the minor axis

COLLEGES: ANDHERI / BORIVALI / CHEMBUR / DADAR / KALYAN / KHARGHAR / NERUL / POWAI / THANE # 33
1
316. If f (x)  2f    3x for all real x  0 , then  f (x)dx is equal to :
x
1 2 1 2 1 2
(B) 2 log x  x  c (C) e  x  c (D) 2 log x  x  c
x
(A) log x  x 2  c
2 2 2

4
317. If Z   3 , then the range of values of |Z| is
Z
(A) [0, 3] (B) [1, 4] (C) [0, 4] (D) [1, 7]

x3
S (x)
. Let S (x) = 2 l n t d t (x > 0) and H (x) =
x
. Then H (x) is
x

(A) continuous but not derivable in its domain


(B) derivable and continuous in its domain
(C) neither derivable nor continuous in its domain
(D) discontinuous at finite number of points

dy 1
319. If the solution of the differential equation  is x = cesin y – k (1 + sin y), then k is equal to
dx x cos y  sin 2y
(A) 1 (B) 2 (C) 3 (D) 4

 
320. A real valued function satisfies 4f  x  1  f  x   2x  sin 2x . The complete interval in which f(x) is
2

monotonically increasing, is
(A) 0,   (B) 0, 2  (C)   ,   (D) R

321. Three of six vertices of a regular hexagon are chosen at random. The probability that the triangle with these
vertices is an equilateral triangle is
3 1 3 1
(A) (B) (C) (D)
5 10 10 2

322. The equation of curve passing through (1, 0) for which the product of the abscissa of a point P & the intercept
made by a normal at P on the x - axis equals twice the square of the radius vector of the point P, is

(A) x2 + y2 = x4 (B) x2 + y2 = x6 (C) x2 + y2 = x (D) none

 f  7    f  2 
3 3

 Let f : [2,7] [0,) be a continuous and differentiable function. Then , for some k   2,7  ,
3
is equal to

(A) 5  f  k   f '  k  (B) 5f  k  f '  k  (C) 3  f  k   f '  k 


2 2
(D) none of these

dy 1 1
324. The solution of the differential equation, x 2 cos  y sin  1  0 , where y  1 as x  is
dx x x

1 1 x 1 1 1 x 1
(A) y  sin  cos (B) y  (C) y  sin  cos (D) y  x cos 1
x x x sin 1x x x x
COLLEGES: ANDHERI / BORIVALI / CHEMBUR / DADAR / KALYAN / KHARGHAR / NERUL / POWAI / THANE # 34
x g ''  x 
 
1/ 2
325. If f  x    1  t 3 dt and g (x) is the inverse of f, then the value of , g '  x   0 , is
g2  x
0
(A) 3/2 (B) 2/3 (C) 1/3 (D) 1/2

x 3 2
326. Matrix A =  1 y 4  , if x y z = 60 and 8x + 4y + 3z = 20 , then A (adj A) is equal to
2 2 z

 64 0 0   88 0 0   68 0 0   34 0 0 
(A)  0 64 0  (B)  0 88 0  (C)  0 68 0  (D)  0 34 0 
 0 0 64   0 0 88   0 0 68   0 0 34 

3
dx
327.  x 2  [x]2  1 – 2x[x] = ......, where [.] denotes the greatest integer function
1

  
(A) (B) (C)  (D)
2 4 3

328. Equation of the circle which bisects the circumference of the circle x2 + y2 + 2 y  3 = 0 and touching the line
y = x at the origin is
(A) 2x2 + 2y2 - 5 x + 5 y = 0 (B) x2 + y2 + 5 x - 5 y = 0
(C) x2 + y2 - 5 x + 5 y = 0 (D) none

329. If f(x + y) = f(x) + f(y) – xy – 1 for all x, y and f(1) = 1, then the number of solutions of f(n) = n, n  N is
(A) one (B) two (C) three (D) none
x2 y2
330. From point (2, 2) tangents are drawn to the hyperbola   1 , then point of contacts of these tangents with
16 9
the hyperbola lie in
(A) I & II quadrants (B) I & IV quadrants (C) I & III quadrants (D) III & IV quadrants

331. The triangle PQR of area 'A' is inscribed in the parabola y2 = 4ax such that the vertex P lies at the vertex of the
parabola and the base QR is a focal chord. The difference of the distances of Q and R from the axis of the
parabola is
A A 2A 4A
(A) (B) (C) (D)
2a a a a

332. The position vector of angular points of a tetrahedron DABC are A(1, 1, 1), B (1, 2, 3) and C(1, 1, 2) and the
3 ˆ 3 ˆ ˆ
position vector of the centre 'G' are  i  j  2k  then shortest distance between the skew line AB and
2 4 
CD is
(A) 0.9 (B) 0.8 (C) 1 (D) 1/3

333. A series of concentric ellipses E1, E2,...,En are drawn such that En touches En–1 at the extremities of the major axis
of En – 1 and the foci of En coincide with the extremities of minor axis of En – 1. If the eccentricity of the ellipses is
independent of n, then the value of eccentricity, is
5 5 1 5 1 1
(A) (B) (C) (D)
3 2 2 5

COLLEGES: ANDHERI / BORIVALI / CHEMBUR / DADAR / KALYAN / KHARGHAR / NERUL / POWAI / THANE # 35
334. If equations of bisector of internal angles B & C of a triangle ABC are respectively x  y  2 & x  y and
coordinates of A are ( 2, 3), then equation of BC is
(A) 2y  x  1 (B) 2y  x  1 (C) 2y  x  1 (D) none of these


cos x  x sin x
335. The value of the integral  2 2
dx is
 x  cos x

 
(A) – (B) (C)  (D) none of these
2 2

336. If k is positive integer and x1 , x2 , x3 ,....., xn , xn 1 are any (n+1) consecutive discontinuities of the function
f ( x )   sin 4( x  k )  , ([.]denotes G.I.F) then n  tan x1 tan x2  tan x3 tan x4  ......  tan xn tan xn 1 is equal
to
n n
(A) sec x1 sec xn 1 sin (B) cos ecx1 cos ecxn 1 sin
4 4
n n
(C) sec x1 sec xn 1 cos (D) cos ecx1 cos ecxn 1 cos
4 4

2 x (x ln x ln 2 –1)
337.  dx
x(ln x) 2
2 x ln x 2 x log 2 e 2x 2x
(A) C (B) C (C) C (D) C
2 log 2 x log 2 x log 4 x

338. A real valued function satisfies 2 f  x  y   f  x   f  y    f  y   f  x   for all real values of x & y, then f
2 2

must be
(A) Odd (B) One-One (C) even (D) none

2001
 3 1 
339. The sum of the binomial coefficients in the expansion of  2  x  2  is
 2 2x 
(A) 0 (B) 1 (C) 1 (D) 1/6

3 dx
340. The value of  if f  x  f   x   9 is
 
3 3  f x

(A) 0 (B) 1 (C) -1 (D) none

341. Given f (x) = x4 + ax3 + bx2 + cx + d such that x = 0 is the only real root of f   x  = 0.
If f (1) < f (1), then in the interval [1, 1]
(A) neither f (1) is the minimum nor f (1)is the maximum of f
(B) f(0) is the maximum & f (1) is the minimum of f
(C) f(0) is the minimum & f (1) is the maximum of f
(D) f (1) is the minimum & f (1) is the maximum of f

COLLEGES: ANDHERI / BORIVALI / CHEMBUR / DADAR / KALYAN / KHARGHAR / NERUL / POWAI / THANE # 36
342. Indicate the correct order sequence in respect of the following :
I. If the probability that a computer will fail during the first hour of operation is 0.01, then if we turn on
100 computers, exactly one will fail in the first hour of operation.
II. A man has ten keys only one of which fits the lock. He tries them in a door one by one discarding the
one he has tried. The probability that fifth key fits the lock is 1/10.
III. Given the events A and B in a sample space. If P(A) = 1, then A and B are independent.
IV. When a fair six sided die is tossed on a table top, the bottom face can not be seen. The probability that
the product of the numbers on the five faces that can be seen is divisible by 6 is one.
(A) FTFT (B) FTTT (C) TFTF (D) TFFF

343. Equation of the circle which bisects the circumference of the circle x2 + y2 + 2 y - 3 = 0 and touching the line
y = x at the origin is
(A) 2x2 + 2y2 - 5 x + 5 y = 0 (B) x2 + y2 + 5 x - 5 y = 0
(C) x + y - 5 x + 5 y = 0
2 2
(D) none
     
344. If the acute angle that the vector  i   j  k makes with the plane of the two vectors 2i  3 j – k and
   –1
i – j  2k is cot 2 then.
(A) ( + ) =  (B) ( + ) =  (C) ( + ) =  (D) = 0

f x
345. If lim f  x  exists and is finite but f(x) is discontinuous at a, then what will be lim given that |f(x)| is
x a x a f a 
continuous at x = a?
(A) 1 (B) 0 (C) 1 (D) none of these

x g ''  x 
 
1/ 2
346. If f  x    1  t 3 dt and g (x) is the inverse of f, then the value of g 2 x is
 
0
(A) 3/2 (B) 2/3 (C) 1/3 (D) 1/2

347. If four points on the parabola y 2  4ax are concyclic, then sum of ordinates of these points is
(A) 1 (B) 0 (C) 1 (D) none of these

348. If A is a non singular maxrix satisfying AB – BA = A, then which one of the following holds true
(A) | B | = 0 (B) B = 0 (C) | A | = 1 (D) |B + I| = | B – I |

a b
349. Let a + b = k, a < k/2 & g  x  > 0. If f  b  a    g 1  x  dx   g 1  x  dx , then f(x) is
0 0

(A) an increasing function (B) a decreasing function


(C) has one point of extremum (D) has two points of extremum

 
6
350. The value of  x  12x  36  x  12x  36 dx is equal to
3

(A) 6 3 (B) 4 3 (C) 12 3 (D) 2 3

COLLEGES: ANDHERI / BORIVALI / CHEMBUR / DADAR / KALYAN / KHARGHAR / NERUL / POWAI / THANE # 37
x 2 y2
351. Let N be the foot of perpendicular drawn from the origin O on to a tangent to the ellipse   1 at a point
25 9
P on the curve. Maximum area of triangle OPN is
(A) 16 (B) 8 (C)  (D) 2

352. Area of region bounded by y 2  8x, y2  8x  16 & y  4 x  1 is

8 2 4 8 2 7 8 2 8 4 2 2
(A) (B) (C) (D)
3 3 3 3

353. If a, b, c are three unequal numbers such that a, b, c are in A.P. and b - a, c - b, a are in G.P., then a : b : c
is
(A) 1 : 2 : 3 (B) 1 : 3 : 5 (C) 2 : 3 : 4 (D) 1 : 2 : 4

354. Let i = –1 . Define a sequence of complex number by z1 = 0, zn + 1 = z 2n  i for n  1 then arg (z2009 + z1004)
is equal to
 
(A) 0 (B) (C)  (D) –
2 2

COLLEGES: ANDHERI / BORIVALI / CHEMBUR / DADAR / KALYAN / KHARGHAR / NERUL / POWAI / THANE # 38
ANSW ER KEY
1. (C) 2. (C) 3. (D) 4. (A) 5. (D) 6. (B)
7. (B) 8. (A) 9. (B) 10. (A) 11. (A) 12. (A)
13. (B) 14. (A) 15. (A) 16. (C) 17. (D) 18. (B)
19. (A) 20. (B) 21. (D) 22. (B) 23. (A) 24. (B)
25. (D) 26. (D) 27. (B) 28. (B) 29. (C) 30. (C)
31. (C) 32. (C) 33. (A) 34. (C) 35. (A) 36. (C)
37. (B) 38. (B) 39. (A) 40. (D) 41. (D) 42. (D)
43. (B) 44. (C) 45. (C) 46. (C) 47. (C) 48. (A)
49. (D) 50. (C) 51. (C) 52. (A) 53. (B) 54. (C)
55. (B) 56. (C) 57. (A) 58. (C) 59. (C) 60. (B)
61. (D) 62. (B) 63. (A) 64. (D) 65. (A) 66. (B)
67. (B) 68. (C) 69. (A) 70. (C) 71. (B) 72. (C)
73. (D) 74. (D) 75. (C) 76. (C) 77. (C) 78. (B)
79. (B) 80. (C) 81. (D) 82. (D) 83. (A) 84. (C)
85. (A) 86. (C) 87. (C) 88. (A) 89. (B) 90. (B)
91. (C) 92. (B) 93. (D) 94. (A) 95. (C) 96. (B)
97. (A) 98. (B) 99. (B) 100. (B) 101. (A) 102. (A)
103. (A) 104. (C) 105. (B) 106. (C) 107. (C) 108. (A)
109. (A) 110. (C) 111. (C) 112. (B) 113. (A) 114. (B)
115. (A) 116. (C) 117. (A) 118. (A) 119. (B) 120. (B)
121. (C) 122. (A) 123. (B) 124. (A) 125. (A) 126. (C)
127. (C) 128. (A) 129. (B) 130. (A) 131. (C) 132. (C)
133. (B) 134. (C) 135. (D) 136. (B) 137. (A) 138. (C)
139. (B) 140. (B) 141. (C) 142. (D) 143. (D) 144. (D)
145. (A) 146. (B) 147. (D) 148. (B) 149. (D) 150. (C)
151. (B) 152. (C) 153. (D) 154. (A) 155. (A) 156. (B)
157. (B) 158. (B) 159. (B) 160. (A) 161. (B) 162. (D)
163. (A) 164. (C) 165. (A) 166. (C) 167. (D) 168. (C)
169. (D) 170. (A) 171. (C) 172. (D) 173. (B) 174. (C)
175. (A) 176. (B) 177. (C) 178. (A) 179. (A) 180. (D)
181. (B) 182. (B) 183. (A) 184. (B) 185. (A) 186. (B)
187. (D) 188. (A) 189. (B) 190. (B) 191. (C) 192. (A)
193. (C) 194. (C) 195. (A) 196. (D) 197. (B) 198. (C)
199. (A) 200. (B) 201. (A) 202. (B) 203. (C) 204. (C)
205. (C) 206. (C) 207. (A) 208. (B) 209. (B) 210. (D)
211. (C) 212. (B) 213. (C) 214. (A) 215. (D) 216. (C)
217. (C) 218. (A) 219. (A) 220. (B) 221. (B) 222. (A)
223. (B) 224. (A) 225. (B) 226. (D) 227. (D) 228. (D)
229. (C) 230. (B) 231. (A) 232. (D) 234. (C) 235. (B)
236. (D) 237. (C) 238. (C) 239. (B) 240. (D) 241. (A)

COLLEGES: ANDHERI / BORIVALI / CHEMBUR / DADAR / KALYAN / KHARGHAR / NERUL / POWAI / THANE # 39
242. (D) 243. (C) 244. (A) 245. (D) 246. (A) 247. (B)
248. (B) 249. (B) 250. (B) 251. (D) 252. (C) 253. (D)
254. (B) 255. (D) 256. (A) 257. (A) 258. (B) 259. (D)
260. (D) 261. (D) 262. (C) 263. (B) 264. (D) 265. (A)
266. (A) 267. (D) 268. (D) 269. (C) 270. (D) 271. (D)
272. (A) 273. (D) 274. (B) 275. (A) 276. (A) 277. (B)
278. (B) 279. (C) 280. (D) 281. (D) 282. (C) 283. (B)
284. (B) 285. (A) 286. (D) 287. (B) 288. (C) 289. (D)
290. (D) 291. (C) 292. (B) 293. (B) 294. (C) 295. (C)
296. (B) 297. (B) 298. (C) 299. (B) 300. (A) 301. (B)
302. (D) 303. (D) 304. (D) 305. (A) 306. (C) 307. (B)
308. (D) 309. (C) 310. (A) 311. (A) 312. (A) 313. (B)
314. (A) 315. (B) 316. (D) 317. (B) 318. (B) 319. (B)
320. (D) 321. (B) 322. (A) 323. (A) 324. (A) 325. (A)
326. (C) 327. (A) 328. (C) 329. (A) 330. (D) 331. (C)
332. (D) 333. (B) 334. (A) 335. (B) 336. (A) 337. (C)
338. (C) 339. (C) 340. (B) 341. (C) 342. (B) 343. (C)
344. (A) 345. (C) 346. (A) 347. (B) 348. (D) 349. (A)
350. (A) 351. (C) 352. (B) 353. (A) 354. (C)

COLLEGES: ANDHERI / BORIVALI / CHEMBUR / DADAR / KALYAN / KHARGHAR / NERUL / POWAI / THANE # 40
MULTIPLE CHOICE QUESTIONS
1. The expression cos2 ( + +) + cos2(+) + cos2  – 2 cos  cos (+) cos( + +), is
(A) independent of  (B) independent on 
(C) dependent on only (D) dependent on and

 1
2. Let f  x   x ln  1   , 0  x   , then which of the following is correct
 x
(A) f is strictly increasing (B) lim f x   0
x0

(C) lim f x   1 (D) f has exactly one point of extremum


x

     
3.  
If a  x i  y j  z k , b  y i  z j  x k & c  z i  x j  y k , then a  b  c is

(A) parallel to (y  z) i + (z  x) j + (x  y) k
(B) orthogonal to i + j + k
(C) orthogonal to (y + z) i + (z + x) j + (x + y) k
(D) orthogonal to x i + y j + z k

4. Which of the following statements is true about circles x2 + y2 + 2x = 0 & x2 + y2 – 6x = 0


(A) length of the transverse common tangents is 2 units
(B) length of the direct common tangents is 2 3 units
(C) region, which is bisected by the line joining the centre, enclosed by the common tangents is a right isoseles
trapezium
(D) area of trapezium, enclosed by the common tangents and two chords of contact is 6 3 units2

5. Let A, B, C be the three sets of complex numbers as defined below A  Z : Im Z  1 , B  Z : Z  2  i  3

 
and C  Z : Re 1  i  Z  2 . Let ' Z ' be any point in A  B  C and let '  ' be any point satisfying

  2  i  3.
(A) |Z| < 3 (B) |w| < 4 (C) |Z| - |w| > - 6 (D) |z| > 3

6. If ,  are the roots of ax 2  4x  1  0 and ,  are the roots of bx 2  6 x  1  0 and , , ,  are in H.P..
then which of the following is correct?
1 1
(A)   (B)   (C) b = 8 (D) a = 3
2 3
sin 4 x cos 4 x 1
7. If + = , then which of the following is/are TRUE?
5 4 9
4 4
(A) cot2x = (B) tan2x =
5 5
64 125 125 64
(C) 6 + = 1458 (D) + = 1458
cos x sin 6 x 6
cos x sin 6 x
COLLEGES: ANDHERI / BORIVALI / CHEMBUR / DADAR / KALYAN / KHARGHAR / NERUL / POWAI / THANE # 41
8. If the equations x 2  ax  2  0 & x 2  4x  a  0 have exactly one root in common, then the value of a is
(A) 3 (B) 2 3 (C) 2 3 (D) 2  3

9. Equation(s) of possible common tangents to y 2  8  x  3 & x 2  8  y  3 is/are


(A) x  y  1 (B) 2 x  y  5 (C) x – 2y + 5 = 0 (D) None

10. For all a, b  R the function f (x) = 3x4  4x3 + 6x2 + ax + b


(A) has exactly one local extremum (B) has no point of inflexion
(C) has two points of inflexion (D) has three points of local extremum

11. The expression sin 4 (37.5)  4 cos 2 (37.5)  cos 4 (37.5)  4 sin 2 (37.5) simplifies to
(A) an irrational number
(B) a prime number
(C) a natural number which is not composite
(D) a real number of the form a  b where a and b are prime.

12. Consider the planes P1 : 2 x  y  z  4  0, P2 : y  z  4  0 and P3 : 3 x  2 y  z  8  0.


Let L1, L 2 , L3 be the lines of intersection of the planes P2 and P3 , P3 and P1, and P1 and P2 respectively..
Then,
(A) Atleast two of the lines L1, L 2 and L 3 are non–parallel
(B) Atleast two of the lines L1, L 2 and L 3 are parallel
(C) The three planes intersect in a line
(D) The three planes form a triangular prism

a  ib
13. Suppose three real numbers a, b, c are in G.P. Let z  , then
c  ib
ib ia ia
(A) Z  (B) Z  (C) Z  (D) Z  0
c b c

14. Let PQ be a chord of the parabola y 2  4 x . A circle drawn with PQ as a diameter passes through the vertex V
of the parabola. If ar(ΔPVQ) = 20 unit 2 then the coordinates of P are
(A) (16, 8) (B) (16,  8) (C) (16,8) (D) (16, 8)

         
15. Let a  ˆi – ˆj, b  ˆj – kˆ and c  kˆ – ˆi . If d is a unit vector such that a .d  0  [b c d] the d
1 ˆ ˆ ˆ 1 ˆ ˆ ˆ 1 ˆ ˆ ˆ 1 ˆ ˆ ˆ
(A) (i  j – 2k) (B) (i  j – k) (C) – (i  j – 2k) (D) – (i  j – k)
6 3 6 3

1
16. If the length of latus rectum of the ellipse x 2 sin 2   y 2  cos 2  is , then the value of    0,   is
2
  5 2
(A) (B) (C) (D)
12 6 12 3
COLLEGES: ANDHERI / BORIVALI / CHEMBUR / DADAR / KALYAN / KHARGHAR / NERUL / POWAI / THANE # 42
17. A line segment of length 7 units has one of its ends on x axis and other end on y axis. A point P divides this line
segment inthe ratio 3 : 4. locus of P will be an ellipse with
(A) length of latus rectum 9/2 (B) eccentricity 7/9
(C) length of major axis 8 (D) None of these

 
9
18. If 3 3  5  R and [R] denotes the greatest integer less than or equal to R, then
(A) [R] is divisible by 10 (B) [R] is divisible by 3
(C) [R] is divisible by 15 (D) [R] is an even number

19. If the quadratic equation x 2  2ax  b  a  1  0 has real roots for all real values of a, and b is an integer, then
which of the following is correct?
(A) Greatest vlue of b is 4 (B) no such value of b is possible
(C) Least value of b is 0 (D) b can be any integer

4  x2 , x0

20. let f  x    2  3x , 0  x  1 , which of the following is correct about f(x)
 2x  4 , x 1

(A) x = 1 is a point of local minimum (B) x = 0 is a point of local maximum
(C) f has no local extremum (D) f  x   3 x has 2 real roots

21. If the equation ax2 – 6xy + y2 + bx + cy + d = 0 represents pair of lines whose slopes are m and m2, then value
of a is
(A) a = – 8 (B) a = 8 (C) a = -27 (D) a = 27

22. Consider the matrix equation X 2  I 2 , where X is a 2  2 matrix (all the elements being real). Which of the fol-
lowing may be correct?
(A) The equation has exactly two solutions (B) The equation has infinitely many solutions

 2 3 4 2   3 
  3 2 1 
(C) X   3 1  is a solution (D) X   2  is a solution
 2  2 3  2  4 2  3 
 

23. If the quadratic equations ax 2  bx  c  0 & cx 2  bx  a have exactly one common root, then which of the
following is correct?
(A) a + b + c =0 (B) b = 0 (C) a = c (D) a – b + c = 0

24. A real valued nonzero function satisfies f  x   f  x  y  f   y  , f '  0   1, then which of the following may be
correct
(A) f '  x  f   x   1 (B) f  x  f   x   1 (C) f  x  f   x   1 (D) f '  x   f  x 

25. A circle is touching the lines x  2 y  3& 2 x  y  1 , the locus of its centre is
(A) 3x  y  4  0 (B) x  3 y  4  0 (C) x  3 y  2  0 (D) 3x  y  2  0

COLLEGES: ANDHERI / BORIVALI / CHEMBUR / DADAR / KALYAN / KHARGHAR / NERUL / POWAI / THANE # 43
26. Equation of normal(s) drawn to the parabola y 2  4  4 x , from the point  6,2  is/are
(A) 2 x  y  14 (B) x  y  8 (C) 2 x  y  10 (D) x  y  4

27. If (sin x + sin 2x + sin 3x)2 + (cos x + cos 2x + cos 3x)2 = 1 then cos x can have the value equal to
(A) 1 (B) 0 (C) – 1/2 (D) – 1

28. Let A and B be events such that P(A ) = 4/5, P(B) = 1/3, P(A/B) = 1/6, then
1 8 5
(A) P(A  B) = (B) P(A  B) = (C) P(BA) = (D) A and B are independent
18 15 18

29. In context of the equations ax 2  2bx  c  0, bx 2  2cx  a  0 & cx 2  2ax  b  0 , where a, b, c  R ,


a  b  c which of the following statements is NOT CORRECT?
(A) All of the equations may have imaginary roots
(B) All of the equation may have real and equal roots
(C) All of the equations may have real and distinct roots
(D) All of the equations may have rational roots
dy sin 2 x
30. A function y = f(x) satisfying the differential equation sin x y cos x + = 0 is such that, y0 as
dx x2
x  , then
/2

(A) Limit
x  0 f(x) = 1 (B)  f(x) dx is less than
0 2
/2
(C)  f(x) dx is greater than unity (D) f(x) is an even function
0

31. The number of ways in whcih three numbers in A.P.can be selected from 1,2, 3, .....n is
1 1
(A) (n – 1)2, if n is odd (B) (n – 1)(n – 2), if n is odd
4 4
1 1
(C) n(n – 1), if n is even (D) n(n – 2), if n is even
4 4

32. The tangent at any point P on a standard ellipse with foci as S & S meets the tangents at the vertices A & A in
the points V & V , then
(A) l (AV) . l (A V) = b2 (B) l (AV) . l (A V) = a2
(C) V SV = 90º (D) V S VS is a cyclic quadrilateral

33. If both roots of the quadratic equation x2 + x + p = 0 exceed p where p  R, then which of the following is
correct?
(A) Greatest integral value of p is 2 (B) p can’t be positive
(C) Greatest integral value of p is 3 (D) p can’t be negative
   
34. If z1, z 2 be two complex numbers z1  z 2  satisfying z12  z 22  z12  z 22  2z1z 2 , then


(A) arg z1  arg z 2   (B) arg z1  arg z2 
2

z1 z1
(C) is purely imaginary (D) is purely real
z2 z2
COLLEGES: ANDHERI / BORIVALI / CHEMBUR / DADAR / KALYAN / KHARGHAR / NERUL / POWAI / THANE # 44
35. The planes x  y  z  1  0,  x  3 y  2 z  3  0, 3 x   y  z  2  0 will form a triangular prism if
(A)   4 (B)   3 (C)   4 (D)   3

n
1
36. Let fn() =  4n sin 4 (2 n ) . Then which of the following alternative(s) is/are correct ?
n 0

 1  2 2  3 
(A) f2   = (B) f3   = (C) f4   = 1 (D) f5() = 0
4 2 8 4  2 

37. A parabola is drawn through (2, 0) & (2, 0), having a tangent to x 2  y 2  a 2 as its directrix. Locus of focus
of this parabola will be –
(A) an ellipse if a > 2 (B) a hyperbola if a < 2
(C) a circle for all values of a (D) a prabola for all values of a

  x x
x x   a 
 a   
, gx  a
x
38. If f  x   a  x   
for a > 0, a  1 and x  R – {0}, where { } & [ ] denote the fractional
part and integral part functions respectively, then which of the following statements holds good for the function
ln  f  x    ln  g  x  
h x  .
ln a
(A) ‘h’ is even (B) ‘h’ is decreasing in domain a  1
(C) ‘h’ is odd (D) ‘h’ increasing in domain a  1

39. If the roots of the equation, x3 Px2 + Qx  19 = 0 are each one more than the roots of the equaton, x3  Ax2 +
Bx  C = 0 where A, B, C, P & Q are constants,
(A) P  A  3 (B) C  P  Q  18 (C) A  B  C  18 (D) none

6
 (n  1)   n  
40. If f() =  cosec    4 
 cosec     , where 0 <  < , then minimum value of f
 4  2
n 1
(A) lies between 3 and 4 (B) lies between 2 and 3
 
(C) occures when  = (D) occures when 
4 6

41. Let P(x) = ax2 + bx + c such that P(1) + P(2) = 0 & P(1) = 0, then

(A) P  8   0 (B) P  x    169


5 20

(C) P  3  a   P  3  a  (D) P(1) = 6


2  2 

42. Let h  x   f  x   f 2  x   f 3  x  for every real value of x & f(x) is monotonically increasing & ONTO func-
tion for all x, then
1 1
(A) h(x) is increasing for f 1  1  x  f 1   (B) h(x) is increasing for f  x   1 or f  x  
3 3
(C) h(x) has exactly two points of extrema (D) h(x) has exactly one point of inflexion

COLLEGES: ANDHERI / BORIVALI / CHEMBUR / DADAR / KALYAN / KHARGHAR / NERUL / POWAI / THANE # 45
43. If the line x + 2y = 3 is rotated about its point of intersection with the line x = y by an angle of 450, then it’s
equations in the new position is/are
(A) 3x - y - 2 = 0 (B) x + 3y - 4 = 0 (C) 3x + y - 4 = 0 (D) x - 3y + 2 = 0

44. A hyperbola passing through origin has x 2y = 1 and x + 3y = 6 as its asymptotes.Eccentricity of the hyperbola
may b
(A) 42 2 (B) 22 2 (C) 42 2 (D) 4  2

45. If A and B are 3 × 3 matrices and | A |  0, then which of the following are true?
(A) | AB | = 0  | B | = 0 (B) | AB | = 0  B = 0
(C) | A–1 | = | A |–1 (D) | A + A | = 2 | A |

 2 3
46. Let E = cos2 + cos2 + cos2 . Then which of the following alternative(s) is/are incorrect?
7 7 7
1 3 3 3 3 7
(A) <E< (B) <E<1 (C) 1 < E < (D) <E<
2 4 4 2 2 4

47. A circle S2 = 0 has its centre lying on the circle S1 : x2 + y2 = 4 and radius half of that of S1.If a common tangent
of these circles is inclined at an angle of 300 with positive direction of x-axis, then possible equation(s) of S2 is/
are?
(A) x2 + y2 + 4x + 3 = 0 (B) x2 + y2 – 4x + 3 = 0
(C) x2 + y2 - 2x - 2 3 y + 3 = 0 (D) x2 + y2 + 2x + 2 3 y + 3 = 0

48. Two whole numbers are randomly selected and multiplied. Consider two events E1 and E2 defined as
E1 : Their product is divisible by 5
E2: Unit's place in their product is 5.
Which of the following statement(s) is/are correct?
(A) E1 is twice as likely to occur as E2. (B) E1 and E2 are disjoint
(C) P(E2/E1) = 1/4 (D) P(E1/E2) = 1

49. the expression x 8  x 4  1 is divisible by


(A) x2 + x + 1 (B) x2 x + 1 (C) x2 + 3 x + 1 (D) x2  3 x + 1

m
50. Let z be a complex number satisfying equation z n  z , where n, m  N , then
(A) if n = m, then number of solution of equation will be finite
(B) if n = m, then number of solution of equation will be infinite
(C) if n  m , then number of solution of equation will be n+m
(D) if n  m , then number of solution of equation will be n+m+1

1 1
51. Given sin  + sin  = and cos  + cos = then which of the following is/are CORRECT?
4 3
    4 – 263
(A) cos   (B) cos    
 2  5 288
24 8
(C) sin      (D) tan     
25 27

COLLEGES: ANDHERI / BORIVALI / CHEMBUR / DADAR / KALYAN / KHARGHAR / NERUL / POWAI / THANE # 46
 
ˆ b  2 ˆi  kˆ and 
Given a  3iˆ  2jˆ  4k,  
52. c  4iˆ  2ˆj  3kˆ . Values of  for which the equation
  
 
xa  yb  zc   xiˆ  yjˆ  zkˆ has non-trival solution (x, y, z) is
(A) –1 (B) 2 (C) 8 (D) –2

 7
 max f  t  ,1  t  x , if 1  x  2
53. Let f  x   2x 3  15x 2  36x  23 & g  x    . Which of the following
 7
12  2x, if  x  6
 2
is correct about g(x).
(A) Continous everywhere (B) Differentiable everywhere
(C) has greatest value 5 (D) None of these

54. If the equation (x a)(x 10) + 1 = 0 has equal roots, then integral value of ‘a’ is
(A) 10 (B) 12 (C) 9 (D) 8

55. If 10! = 2p 3q 5r 7s, then


(A) p = 8 (B) q = 4 (C) r = 2 (D) s = 2

x 2 y2
56. Equation of common tangent to   1 & y 2  16x may be
10 6
(A) x + y = 4 (B) x  y = 4 (C) x + y = 4 (D) x  y = 4

57. Which of the following expression(s) have their value equal to unity?
(A) cos4  + 2 sin2  – sin4 
(B) sin2  cos2  + cos2  sin2  + sin2  sin2  + cos2  cos2 
(C) sin (285° – ) cos ( + 165°) + sin ( + 165°) cos ( – 285°)
sin 2  cos 2 
(D) (1 + cot2 ) + (1 + tan2 )
2 2

Let f  x   x  1  3 , then the point(s) where f(x) is non differentiable is/are


2
58.
(A) 2 (B) 2 (C) 1 (D) 1

59. If tangents to y 2  4ax at P & R meet in Q and S is the focus, then wich of the following is correct
(A) Abscissae of P, Q & R are in G.P. (B) Ordinates of P, Q & R are in A.P.
(C) SP, SQ & SR are in G.P. (D) None of these

60. Let (1 + x2)2 (1 + x)n = A0 + A1 x + A2 x2 + . . . If A0, A1, A2 are in A.P. then the value of n is / are
(A) 2 (B) 3 (C) 5 (D) 7

61. Consider the points O(0, 0) A(0, 1), B(1, 1) in the xy plane, suppose that points C (x, 1) and D(1, y) on a line
passing through O are chosen such that 0 < x < 1. Let sum of the areas of DOAC & DBCD be ‘S’ then which
of the following are correct?
1 1
(A) min. value of S is irrational lying in  , 
3 2
 

2 
(B) min. value of S is irrational in  ,1
3 
COLLEGES: ANDHERI / BORIVALI / CHEMBUR / DADAR / KALYAN / KHARGHAR / NERUL / POWAI / THANE # 47
2 
(C) The value of x for minimum value of S lies in  ,1
3  

1 1
(D) The value of x for minimum values of S lies in  , 
3 2

          
62. If three vectors a , b , c are such that a  0 and a  b  2 a  c , a  c = 1, b = 4 and the angle between
  1   
b & c is cos -1 then b  2 c   a where  is equal to
4
(A) 2 (B) 4 (C) –4 (D) –2

n
     
63. Let fn() =  tan 2n  sec 2n 1  Then which of the following alternative(s) is/are correct ?
n 1

 4 
(A) f3(2) = –1 (B) f4   = 2( 3  1)
 3 
(C) f5 (4) = 2 1 (D) f6 (48) = 1

1
Let f  x   f '  x   e  x  f  x   , f  0   , 2  x   . Which of the following is correct?
2
64.
2
(A) least value of f is 1/e (B) f(x) = x has two solutions
(C) greatest value of f is e (D) f(x) is a One - One function

65.  
Which of the following trigonometric ratio's can be equal to log0. 5 (1.8)  log1.8 (0. 5 ) for some value of 
(A) cot  (B) cos  (C) sec  (D) cosec 

66. Let f : R  R satisfies f  f  x   f  y    f  f  x    2x 2 f  y   f  y 2  where f(x) is not identically zero


everywhere. f(x) can not be
(A)  (B)  (C) Monotonic (D) MANY - ONE

67. The value of x satisfying the equation ln (tan x) = 0, is/are


7 3 5 
(A) – (B) – (C) (D)
4 4 4 4

68. A tangent drawn from the point (4, 0) to the circle x2 + y2 = 8 touches it at a point A in the first quadrant. The
coordinates of another point B on the circle such that l(AB) = 4 are
(A) (2, - 2) (B) (- 2, 2) (C) (- 2, -2) (D) (2, 2)

69. A boy has a collection of blue and green marbles. The number of blue marbles belong to the sets
{2, 3, 4, ..... 13}. If two marbles are chosen simultaneously and at random from his collection, then the probability
that they have different colour is 1 2 . Possible number of blue marbles is :
(A) 2 (B) 3 (C) 6 (D) 10

70. If AB = A and BA = B, then


(A) A2B = A2 (B) B2A = B2 (C) ABA = A (D) BAB = B

COLLEGES: ANDHERI / BORIVALI / CHEMBUR / DADAR / KALYAN / KHARGHAR / NERUL / POWAI / THANE # 48
71. Least value of f(x) = |px – 1| + 2|x|, x  R, p > 0 is
(A) 1 if p  2 (B) 2/p if p > 2 (C) 1 if p  2 (D) 2/p if p < 2

72. The two lines through (2, 3) from which the circle x2 + y2 = 25 intercepts chords of length 8 units have equations
(A) 12x - 5y = 9 (B) 12x + 5y = 39 (C) x = 2 (D) y = 3

73. Which of the following identities wherever defined hold(s) good?


(A) (tan  + cot )2 = sec2 cosec2 (B) tan 3 – tan2 – tan  = tan 3 tan 2 tan 
cot 2   1    
(C) = sec2 + cosec2 (D) tan     + tan     = 2 cosec2
1  sin 2  4  4 

74. The parametric co-ordinates of any point on the parabola y2 = 4x can be


(A) (sin2 , 2 sin ) (B) (cos2 , 2 cos ) (C) (sec2 , 2 sec ) (D) none

eax  e x  x
75. If L = lim exists and is a non-zero finite number, then
x 0 x2
(A) a = 1 (B) a = 2 (C) L = 2 (D) L = 3/2

3 3
76. The equation x3 – x=– is satisfied by
4 8
 5   7   23   17 
(A) x = cos   (B) x = cos   (C) x = cos   (D) x = cos  
 18   18   18   18 
          
77.    
If a & b are unit vector and c is a vector satisfying c  2 c  a  a  b then  a b c  may be

1 1 1 1
(A) (B) (C) (D)
2 4 5 7

78. The number of diagonals in a regular polygon is 54. Triangles are formed by joining the vertices of the polygon.
Now which of the following is/are true?
(A) The maximum number of triangles that can be formed is 220
(B) The number of triangles, having one side common with the sides of polygon is 96
(C) The number of triangles, having two sides common with the sides of polygon is 12
(D) The number of triangles, having no side common with sides of polygon is 112
 3 
79. Which of the following is a tangent to y  1  2e x drawn from   , 0 
 2 
(A) 2x  y  3  0 (B) 2x  y  3  0 (C) 4x  y  6  0 (D) 4x  y  6  0

80. The equation, 2(a +1)x2 + 3ay2  4(a +1)x  12ay  16 = 0, represents
(A) a circle if a = 2 (B) an ellipse if a > 2 (C) a point if a = 1 (D) a pair of lines if a = 1

2
x 2  y 2  dy 
Solution of    2 2
dy
81.    1  0 may be?
 dx  x  y 2  dx 
y
(A) y 2  2xy  x 2  C (B) 2 tan
1
 ln  x 2  y 2   C
x
x
(C) 2 tan y  ln  x  y   C
1 2 2
(D) x 2  2xy  y 2  C

COLLEGES: ANDHERI / BORIVALI / CHEMBUR / DADAR / KALYAN / KHARGHAR / NERUL / POWAI / THANE # 49
82. A hyperbola with T.A. = 2a & C.A. = 2b has centre ' C ' and one focus at P  6,8  . If its two directrices are
3 x  4 y  10  0 and 3 x  4 y  10  0 then

11
(A) e  5 (B) a  2 5 (C) 15 (D) e 
2 2

83. The value of sin 27° – cos 27° is equal to

3 5 5 5 5 1 3 5
(A) – (B) – (C)  (D)
2 2 2 2 2

84. Consider the function f  x   cos


1
8x 4
 8x 2  1 , f :  1,1   0,  .
(A) f(x) is not differentiable at 3 points (B) f(x) has 3 points of local extrema in (-1, 1)
(C) f(x) = cos x has 4 solutions (D) f(x) is increasing in (0, 1)

85. A square matrix A with elements from the set of real numbers is said to be orthogonal if AT = A–1. If A is an
orthogonal matrix, then
(A) AT is orthogonal (B) A–1 is orthogonal (C) Adj. A = AT (D) |A–1| = 1

86. A line passing through (1, 0) and divides the area bounded between y = ln x, y = -ln x & the y-axis in the ratio
1 : 2. If the line cuts one of the two curves in exactly one point, then slope of the line may be?
(A) 3/2 (B) 2/3 (C) -2/3 (D) -3/2

87. Three circles with different radii r1, r2, r3 cut each other orthogonally. Their centers are c1, c2 & c3. Then D c1c2c3
(A) can be equilateral (B) can’t be obtuse angled
(C) can’t be right angled (D) nothing can be specified in general

88. If A & B are two events such that P(B)  1, BC denotes the event complementry to B, then


(A) P A BC =  P (A)  P (A  B)
1  P (B)
(B) P (A  B)  P(A) + P(B)  1
(C) P(A) > < P A B according as P A BC > < P(A)  
  
(D) P A BC + P A C BC = 1 
1
89. A line is drawn through the point P(–1,1) to meet the curve y  in the points A and B (Points A and B lie on
x
same side of P). A point R is chosen on this line such that PA, PR and PB are in A.P. Then the locus of R may be
(A) 2xy = 1 (B) 2xy = x – y (C) 2xy = x + y (D) x – y = 2

B C A
90. The base BC of ABC is fixed and the vertex A moves, satisfying the condition cot  cot  2 cot , then
2 2 2
(A) b + c = a (B) b + c = 2a
(C) vertex A moves on a straight line. (D) vertex A moves on an ellipse.

1
91. Greatest term in the binomial expansion of (a + 2x)9 when a = 1 & x = is
3
(A) 4th (B) 5th (C) 6th (D) 7th

COLLEGES: ANDHERI / BORIVALI / CHEMBUR / DADAR / KALYAN / KHARGHAR / NERUL / POWAI / THANE # 50
92. f(x) is a polynomial function f : R  R such that f  2x   f '  x  .f ''  x  . Which of the following is correct?
(A) f(x) is ONE-ONE (B) f(3) = 12
(C) f(x) = x has three roots (D) f(x) is INTO

93. From an external point P, pair of tangent lines are drawn to the parabola, y2 = 4x . If 1 & 2 are the inclinations

of these tangents with the axis of x such that 1 + 2 = , then the locus of P is
4
(A) x - y + 1 = 0 (B) x + y –1 = 0 (C) x - y - 1 = 0 (D) x + y + 1 = 0

 /2 n  /2n

94. Let  ln  sin x  dx  a n ,  ln  cos x  dx  b n , then


0 0

  
(A) a1  b1   ln 2 (B) a2  b2   ln 2 (C) a2  b2  ln2 (D) a1  b1   ln 2
2 2 4

95. In ABC, D is a point on BC such that DB = 14, DA = 13 and DC = 4. If the circumcircle of the  ADB is
congruent to the circumcircle of the  ADC then which of the following is/are correct?
(A) angle B > 45° but angle C < 45° (B) both the angles B and C are greater than 45°
 24 
(C) area of the triangle is 108 sq. units (D) measure of angle A equal to tan–1  
 7 
n
 x
 
, g  x   lim 1  x  x n e & h(x) = tan  g f  x   .
n
Let f  x   lim  cos
1 1 1
96. 
n   
n n 

ln  f  x   1
(A) xlim
 0 ln  g  x  
 = (B) g(x) > 0
2

 
(C) 0  f  x   1 (D)   h x 
2 2

97. There are 15 seats round a table marked 1,2,3,...,15. The number of ways in which 6 seats can be selected, such
that no two or more are adjacent, is
(A) 8 C 6 (B) 10 C 4  8 C 4 (C) 2  8 C4 (D) 2  10 C 4
a

98. I   [x 2  3x  2]dx 
0

3 5 2 5
(A) a = 1, then I = (B) a = 3/2, then I =
2 2
(C) a = 3, then I = 2  5 (D) a = 2, then I = 1  5

99. The equation of the line x  y  z  1  0 , 4 x  y  2 z  2  0 written in the symmetrical form is


x 1 y  2 z  0 x y z 1
(A)   (B)  
2 2 1 1 2 1
1 1
x z x 1 y  2 z  2
(C) 2 y  1 2 (D)  
 2 1 2
1 2 1
COLLEGES: ANDHERI / BORIVALI / CHEMBUR / DADAR / KALYAN / KHARGHAR / NERUL / POWAI / THANE # 51
x2 y2
100. Equation of tangent at the point which is at a distance 2 from the centre of the ellipse + = 1, is
6 2

y x y x
(A) x  2 (B) y2 (C) x  2 (D) y2
3 3 3 3

101. A line passes through (2, 0). The slope of the line, for which its segment intercept between the lines y = x – 1 and
y = – x + 1, subtends a right angle at the origin is
1 1
(A) 3 (B)  3 (C)  (D)
3 3

a b 
102. If A = c d  (where bc  0) satisfies the equations x2 + k = 0, then
 
(A) a + d = 0 (B) k = –|A| (C) k = |A| (D) none of these

C
103. In  ABC , if cos A + cos B = 4 sin2 , then which of the following hold(s) good?
2
A B A B
(A) cot cot = 2 (B) cot cot = 3 (C) a, c, b are in A.P. (D) a, b, c are in G.P.
2 2 2 2

104. A variable circle is drawn through the points A(3, 2) & B(7, 4). Which of the following is correct about this
circle
(A) it will always pass through a fixed point other then A & B
(B) length of tangent to this circle from (1, 1) is constant
(C) area of smallest such circle is 5p
(D) none of these

3 1 5
105. For P(A) = ; P(B) = ; P(A  B) = which of the following do/does hold good?
8 2 8
 
(A) P A c B  2P A Bc   (B) P(B) = PA B
 
(C) 15 P A c Bc  8 P B A c    
(D) P A Bc  PA  B

dy 5x 4  2xy3  3x 2 y 2
106. Let the general solution of the differential equation  4 be the family of curves given by
dx 5y  2yx 3  3x 2 y2
R  x, y   c and initial value of  x, y  be  0, 0  , then which of the following points lies on the curve
(A) 1, 1 (B)  2, 2  (C)  1,1 (D) none of these

x
107. Let ' f ' be a real valued function defined on the interval  0,  by f  x   ln x   1  sin t dt . Then which of
0
the following statement(s) is/are true?
(A) f '  x  exists for all x   0,   & f ' is continuous, but not differentiable on  0,  

(B) f " x  exists for all x in  0,  

(C) There exists   1 such that f '  x   f  x  for all x   ,  

(D) There exists   0 such that f  x   f '  x    for all x   0,  


COLLEGES: ANDHERI / BORIVALI / CHEMBUR / DADAR / KALYAN / KHARGHAR / NERUL / POWAI / THANE # 52
 
108. Let  e  x dx  a & I n   x n e  x dx, n  N , then In may be
2 2

0 0

1  n 1   n  1! a
1  n 1   n  1!
(A)   !a (B)
n2
(C)  ! (D)
2 2 
2n 1 
2 2  n2
! 2n 1  !
 2   2 

109. In a AEX, T is the mid point of XE, and P is the mid point of ET. If the APE is equilateral of side
length equal to unity then which of the following alternative(s) is/are correct?
(A) AX = 13 (B) EAT = 90°
1 1
(C) cos  XAE = (D) AT =
13 3

1
tan x
110. If f  x  
x x 0

then lim f  x    x 
2 f x

  (where [.] denotes the greatest integer function and {.}
denotes fractional part).
(A) 3 (B) ln 3 (C) e3 (D) doesn’t exist

111. A variable circle touches two distinct given circles of radii r1 & r2. Locus of center of this circle depending on the
two given circles may be
(A) a straight line (B) a circle (C) an ellipse (D) a hyperbola

3/ 2
 2 3
 sin
1
112. Let I   x  2x   dx , then
1/ 2  2

  3 3
(A) I  (B) I  (C) I  sin 1 (D) I  sin 1
6 8 4 4

113. Which of the following expression(s) have their value equal to four times the area of the triangle ABC?
(All symbols used have their usual meaning in a triangle)

(a  b  c ) 2
(A) rs + r1(s – a) + r2(s – b) + r3(s – c) (B)
A B C
cot  cot  cot
2 2 2
(C) (a2 + b2 – c2) tan B 2 2
(D) b sin 2C + c sin 2B

114. Slope of a line through (-5, 4), such that the lines x + 2y + 1 = 0 & x + 2y - 1 = 0 cut off an intercept of length
5 units from it, may be
(A) 3/4 (B) –7/24 (C) 4/3 (D) –3/4

115. If f : R  R, f(x) is a differentiable bijective function, Then which of the following may be true?
(A) (f(x) – x) f(x) < 0  x R
(B) (f(x) – x) f(x) > 0  x R
(C) If (f(x) – x) f(x) > 0, then f(x) = f–1(x) has no solution
(D) If (f(x) – x) f(x) > 0, then f(x) = f–1(x) has at least one real solution

COLLEGES: ANDHERI / BORIVALI / CHEMBUR / DADAR / KALYAN / KHARGHAR / NERUL / POWAI / THANE # 53
 x2
116. Equation of the line(s) through the point (1/2, 2) which is/are tangent to the parabola y = + 2 and secant to
2
the curve y = 4  x 2 is/are
(A) 2x + 2y - 5 = 0 (B) 2x + 2y - 9 = 0 (C) y - 2 = 0 (D) none

117. Which of the following are true?


(A) Number of ways of giving away 3 prizes, 1st, 2nd and 3rd in Mathematics in a class of n students, is n(n – 1)
(n – 2)
(B) Number of ways of distributing 6 identical balls among 3 identical boxes is 8C2
(C) Number of ways of distributing 6 different balls among 3 different boxes is 6 × 5 × 4
(D) Number of ways of distributing 3 prizes, 1st in Physics, Mathematics and Chemistry to a class of 30 students
is (30)3

1
118.  3x  x 2  2
dx 

x 1 x2
(A) cos 1  2x  3  C (B) 2 tan  C (C) sin 1  2x  3  C
1 1
(D) 2 tan C
2 x 1 x

119. Which of the following functions represent identical graphs in x-y plane  x  (2, 3)?
(A) y  cos 1 3  x (B) y  sin 1 x  2

1  3  x 
(C) y  cot  
 (D) y 
1 1

sin 2 (3  x )( x  2) 
 x2  2

120. Coefficient of x4 in the product of the expansions 1  x  1  x  is


5 6

  1
r 6
(A) 5 C 2 (B) Cr 5C4  r (C)  5C (D) 5 C 4
3

1  1 1  1 
121. Given the matrices A = 4  1 and B = 2  2 . The two matrices X and Y are such that XA = B and
   
AY = B then which of the following hold(s) true?
1  1  1 1 3 0 4  1
(A) X = (B) Y = (C) | X | = | Y | (D) 3(X + Y) = 4 2 
3  2 2  3 4 0  

  
122. Let a, b & c be non-coplanar unit vectors equally inclined to one another at an acute angle .
Then which of the following is/are true.
     
(A) | [a b c] | = (1 – cos) 1  2cos  (B) [a  b b  c c  a ] = (1 – cos)2 ( 1  2cos  )
     
(C) | [a b c] | = (1 + cos) 1  2 cos  (D) [a  b b  c c  a ] = (1 + cos)2 ( 1  2cos  )

123. If E1 and E2 are two events such that P(E1) = 1/4, P(E2/E1) =1/2 and P(E1/ E2) = 1/4
(A) then E1 and E2 are independent
(B) E1 and E2 are exhaustive
(C) E2 is twice as likely to occur as E1
(D) Probabilities of the events E1  E2 , E1 and E2 are in G.P.

COLLEGES: ANDHERI / BORIVALI / CHEMBUR / DADAR / KALYAN / KHARGHAR / NERUL / POWAI / THANE # 54
x 2 y2
124. Let   1 be a given ellipse and a bi-focal Conic‘C’ has its foci same as those of the given ellipse. If the
9 4
conic ‘C’ passes through (3, 2), then its latus rectum may be
x 2 y2 x 2 y2 x 2 y2
(A) x 2  y 2  5 (B)  1 (C)  1 (D)  1
3 2 15 10 18 8

125. A ray emanating from the point ( 4, 0) is incident on the ellipse 9x² + 25y² = 225 at the point P with abscissa
3. Equation of the reflected ray after first reflection may be
(A) 12 x + 5 y = 48 (B) 12 x  5 y = 48 (C) 5 x + 12 y = 20 (D) 5 x  12 y = 20

x2 1
126. If  dx  f  g  x    c , where g(x) is a rational algebraic function of x, then which of the following
x 4  3x 2  1
may be g  f 1  x   ?
(A) 2 cosec 2x (B) 2 cos ec 2x (C) 2 cot 2x (D) 2 cot 2x

3  5  12  5
127. Let P = 7  12 and Q =  7  3 then the matrix (PQ)–1 is
   
(A) nilpotent (B) idempotent (C) involutory (D) symmetric

128. If a circle passes through the points of intersection of the co-ordinate axes with the lines px - y + 1 = 0 &
x - 2y + 3 = 0, then the value of p can be :
(A) 2 (B) - 2 (C) 3 (D) 1/3

129. Let 5x 2  4xy  y 2  1 & z 2  x 2  y 2 , x  y  0 . Which of the following is correct?


(A) minumum value of z is 2  1 (B) maximum value of z is 2  1

2 1 2 1
(C) z is increasing for x < (D) z is increasing for x >
2 2 2 2

130. Differential equation of a curve passing through (0, 1) is  x 2  y  dx   y 2  x  dy  0 . Y Coordinate of the
point on the curve with X Cordinate 1, is
(A) 1 (B) 2 (C) 0 (D) 1
1
131. Let A = cot–1 7 and B = tan–1 , then which of the following hold(s) good?
3
24 5 cos 2A 3
(A) cos 2A = (B) sec 2B = (C) =1 (D) cot 2B =
25 4 sin 4B 4


sin  n x 
132. If I n  
 1  e  sin x
x
dx for n  0,1, 2,..... then

20
(A) I2 = I3 (B) I1 = I3 (C)  Ik  10 
k 1
(D) none

133. The bisector of internal angles A in DABC meets BC in point D such that BD = 4 & CD = 2. If altitude
AE > 10 & AB & AC are integers, then AC may be
(A) 4 (B) 5 (C) 6 (D) none
COLLEGES: ANDHERI / BORIVALI / CHEMBUR / DADAR / KALYAN / KHARGHAR / NERUL / POWAI / THANE # 55
ax 2  bx  c x  1
134. Let f  x    . If f(x) is differentiable at x = 1, then
 2ax  b x 1
(A) minimum of f is a when a > 0 (B) maximum of f is a when a < 0
(C) f increases, when x and a have same sign (D) f decreases when x and a have same sign

1 1 1 2
ex x2 ex  x  x
135. Let 0 1 x
dx  a , I1  
0 x  1
dx & I 2   
0  x  1
 e dx , then

e e
(A) I1  a  e  2 (B) I 2   a 1 (C) I 2  a (D) I1  a  e
2 2

136. Which of the following statement(s) is/are TRUE?


(A) Domain of y = cos–1(ex) is same as range of y = –  x .
(B) Number of elements common in the range of function y = tan–1(sgn x) and y = cot–1(sgn x) is only 1 (where sgn
x denotes signum function of x.)
(C) The function y = sgn (cot–1x) and y = 1 are identical functions.
(D) Number of integers in the solution set of 1 < log2(tan–1x) < 2 is 4.

137. The number of non-negative integral solutions of x1 + x2 + x3  n (where n is a positive integer) is


(A) n+3C3 (B) n+4C4 (C) n+3Cn (D) n+4Cn

 7 9
138. The equation(s) of the normal(s) drawn to the curve y = x2  x + 1 from the point P  ,  is/are
2 2
(A) x  y + 1 = 0 (B) x  3y + 10 = 0 (C) 2x + 8y  43 = 0 (D) x + y = 8

max{f (t) / 0  t  x, 0  x  3
139. Let f(x) = x3 – 6x2 + 9x – 3 and g(x)   , then
 4  x, 3 x  4
(A) maximum of g(x) is 1 (B) minimum of g(x) is 0
(C) g(x) is not differentiable at x = 1 (D) g(x) is continuous everywhere

140. If A and B are two 3 × 3 matrices such that their product AB is a null matrix then
(A) | A  0  B must be a null matrix.
(B) | B  0  A must be a null matrix.
(C) If none of A and B are null matrices then atleast one of the two matrices must be singular.
(D) If neither det. A nor det. B is zero then the given statement is not possible.

141. Let f : R  R be a function such that f (2  x)  f (2  x) and f (4  x)  f (4  x) and given that


2 50

 f ( x)dx  5 then the value of  f ( x)dx is equal to


0 0

46 51 52

(A) 125 (B) 


4
f ( x) dx (C) 
1
f ( x) dx (D)  f ( x)dx
2

142. Two events A and B are such that the probability that at least one of them occurs is 5/6 and both of them
occurring simultaneously is 1/3. If the probability of not occurrence of B is 1/2 then
(A) A and B are equally likely (B) A and B are independent
(C) P(A/B) = 2/3 (D) 3 P(A) = 4 P(B)

COLLEGES: ANDHERI / BORIVALI / CHEMBUR / DADAR / KALYAN / KHARGHAR / NERUL / POWAI / THANE # 56
143. Let a variable line L be such that algebraic sum of its perpendicular distances from three points A(2 , 0), B(2 , 1)
& C(2 , 2) is always 6 units. This line will always touch a fixed circle. Which of the following is correct about this
circle?
(A) centre lies at (2, 1) (B) radius is 2 units
(C) tangential to Y – axis (D) intercept on X – axis is 2 3 units

144. Let f : R  R defined by f (x) = cos–1 ( –{ – x }) where {x} is fractional part function. Then which of the
following is/are correct?
(A) f is many one but not even function. (B) Range of f contains two prime numbers.
(C) f is aperiodic. (D) Graph of f does not lie below x–axis.
x 2 y2
145. If ellipses   1 be described, then for all values of b, the tangent at one of the ends of their latus recta
4 b2
passes through
(A) (0, 2) (B) (0, – 2) (C) (0, 4) (D) (0, – 4)

x
146. Let f  x   
1 x
 sin  t 2   dt. Then

3 3
(A) f ' 1  (B) xlim f ' x   0 lim f '  x   0
(C) x  (D) f ' 1  sin1
2 0 2
x 3t
147. Let f (x)  1 dt, x  0 , then
1 t2
(A) for 0    , f ( )  f () (B) for 0    , f ( )  f ()
 
(C) for all x > 0, f (x)   tan 1 x (D) for all x > 0, f (x)   tan 1 x
4 4

148. A ray of light travelling along the line x + y = 1 is incident on the x-axis and after refraction it enters the other side
of the x-axis by turning p/6 away from the x-axis . The equation of the line along which the refracted ray travels
is :

 
(A) x + 2  3 y = 1  
(B) 2  3 x + y = 1

(C) y +  2  3  x = 2 + 3 (D) none of these

 x  1   y  2   x  2   y  2
2 2 2 2
149. If   k , then locus of P(x, y) may be

(A) a line segment (B) two rays (C) a hyperbola (D) no locus

5  25  9
150. In ABC, if B = sec 1    cos ec 1 5 , C = cosec–1   + cot–1   and c = 3.
4  7   13 
Which of the following statement(s) is/are correct?
(where all symbols used have their usual meanings in a triangle)
(A) tan A, tan B, tan C are in arithmetic progression.
5
(B) The distance between orthocentre and centriod of ABC is .
3
(C) Area of ABC is irrational.
(D) Radius of escribed circle drawn opposite to vertex A is rational.
COLLEGES: ANDHERI / BORIVALI / CHEMBUR / DADAR / KALYAN / KHARGHAR / NERUL / POWAI / THANE # 57
151. If f  x    1  x  2 , 0  x  4 and g  x   2  x ,  1  x  3 then at x = 0,  fog  x  is
(A) discontinuous (B) continuous (C) differentiable (D) non differentiable
n
 2 2
152. If there exist a nonzero term independent of x in the expansion of  x  3  , then n can’t be
 x 
(A) 7 (B) 12 (C) 9 (D) 15

2
153. Consider a real valued continuous function f such that f (x)  sin x   (sin x  t f (t))dt . If M and m are


2

maximum and minimum values of the function f, then


M
(A) 3 (B) M  m  2  1 (C) M  m  4(  1) (D) Mm  2(2  1)
m

154. If the first three terms of an arithmetic progression are the roots of the equation 4x3 24x2 + 23x + 18 = 0, then
the sum of the first 12 terms can be
(A)  (B) 159 (C) 45 (D) 111/2

155. Consider the planes 3x  6y  2z  5  0 and 4x  12y  3z  3 . The plane 67x  162y  47z  44  0 bisects
that angle between the given planes which
(A) contains origin (B) is acute (C) is obtuse (D) none of these

1
156. Let function f (x) be defined as f (x) = | sin–1x | + cos–1   . Then which of the following is/are TRUE?
x
(A) f (x) is injective in its domain.
(B) f (x) is many-one in its domain.
(C) Range of f is a singleton set.
(D) sgn f ( x )  = 1 where sgn x denotes signum function of x.
3 bi
157. If Ai is the area bounded by | x  a i |  | y | bi ,i  N, where a i 1  a i  bi and bi 1  , a1 = 0, b1 = 32,
2 2
then
n
8
 Ai   32  (D) none
2
(A) A3 = 128 (B) A3 = 256 (C) lim
n 
i 1 3

158. Maximum number of regions into which 10 lines can divide a plane, is
8
(A) 42 (B) 56 (C) P2 (D) 7 P2

159. Normals at the points P, Q & R of the parabola y2 = 4ax are concurrent in a point on the line y = 2a. Chords PQ,
QR and RP of this parabola touch a fixed parabola.Which of the following is correct
(A) vertex of this parabola is (0, 0)
(B) latus rectum length of this parabola is same as that of the given parabola
(C) the two parabolas are reflections of each other in x = y.
(D) the two parabolas are having same focus
 x
160. The values of x for which the equation, 2 sin–1  sin  = x is valid?
 2
 
(A) for all x. (B) – 1  x  1 (C) –   x   (D) – x
4 4

COLLEGES: ANDHERI / BORIVALI / CHEMBUR / DADAR / KALYAN / KHARGHAR / NERUL / POWAI / THANE # 58
161. In a  ABC, if “ a , b , c” are in A.P, such that a < b < c, then it is necessary that
2 b b 1 c b
(A)  1 (B) 1  2 (C)  1 (D) 1  2
3 c c 3 a a
3

162. Let I =  1
3  x 5/ 2 .dx then I is

(A) greater than 4 (B) greater than 27 (C) less than 26 (D) greater than 2 3

n
163. If there are three square matrix A, B, C of same order satisfying the equation A2 = A–1 and let B = A 2 &
( n 2)
C = A2 then which of the following statements are true?
(A) det. (B – C) = 0 (B) (B + C)(B – C) = 0
(C) B must be equal to C (D) none

164. The probabilities of events, A  B, A, B & A  B are respectively in A.P. with probability of second term equal
to the common difference. Therefore the events A and B are
(A) mutually exclusive (B) independent
(C) such that one of them must occur (D) such that one is twice as likely as the other

165. Let (x1, y1), (x2, y2) & (x3, y3) be respectively the vertices of a D ABC, then
(A) (x1 - x2) (x1 - x3) + (y1 - y2) (y1 - y3) is positive, then angle A is acute
(B) (x1 - x2) (x1 - x3) + (y1 - y2) (y1 - y3) is negative, then angle A is obtuse
(C) (x1 - x2) (x1 - x3) + (y1 - y2) (y1 - y3) is zero, then angle A is a right angle
(D) (x1 - x2) (x1 - x3) + (y1 - y2) (y1 - y3) is 3 , then angle A is 600.

166. A circle passes through the points (-1, 1), (0, 6) and (5, 5). The point(s) on this circle, the tangent(s) at which is/
are parallel to the straight line joining the origin to its centre is/are
(A) (1, - 5) (B) (5, 1) (C) (- 5, - 1) (D) (- 1, 5)

x2 y2
167. A tangent to ellipse   1 at a point P meets the lines x =  5 at the points Q & R. Then a circle whose
25 16
extremities of diameter are R and Q is passes through a fixed point, whose coordinates are
(A) (3, 0) (B) (3, 0 ) (C) (5, 0) (D) (4,0)

   2
 
2
168. If sin 1 x  sin 1 y  sin 1 z  
2 3 2
4
, then the value of (x – y + z) can be

(A) 1 (B) – 1 (C) 3 (D) – 3

dx
169.  cos x cos 2x
is equal to

cos 2x
(A) sin 1  tan x   C  C (C) cos 1  tan x   C (D)  sin 1  tan x   C
1
(B) cot
sin x

170. If a, b, c are in H.P., then


a b c 2 1 1
(A) , , are in H.P.. (B)  
bca ca b a bc b ba bc
b b b a b c
(C) a  , , c  are in G.P.. (D) , , are in H.P..
2 2 2 bc ca ab
COLLEGES: ANDHERI / BORIVALI / CHEMBUR / DADAR / KALYAN / KHARGHAR / NERUL / POWAI / THANE # 59
171. Let f(x) = sin–1 |sin x| + cos–1(cos x). Which of the following statement(s) is/are TRUE ?
(A) f f (3)  =  (B) f (x) is periodic with fundamental period 2.
(C) f (x) is neither even nor odd. (D) Range of f (x) is [0, 2]

2
 dy  dy
172. Solution of x     y  x   y  0 passing through (1, 2) may be
 dx  dx
(A) y 2  x 2  3 (B) xy = 2 (C) x 2  y 2  5 (D) y = x +1

173. The sum of first ten terms of an A.P. is equal to 155 and the sum of first two terms of a G.P. is 9. If the first term
of the A.P. is equal to the common ratio of the G.P. and the first term of the G.P. is equal to the common difference
of the A.P., then
2 2
(A) first term of the G..P. is ,3 (B) first term of the A.P. is ,3
3 3
25 2
(C) common ratio of the G.P. is ,2 (D) common difference of the A.P. is ,3
2 3

174. If f(x) = sin–1 x · cos–1 x · tan–1 x · cot–1 x · sec–1 x · cosec–1 x, then which of the following statement(s) hold(s)
good ?
(A) The graph of y = f(x) does not lie above x axis.
3 6
(B) The non-negative difference between maximum and minimum value of the function y = f(x) is .
64
(C) The function y = f(x) is not injective.
(D) Number of non-negative integers in the domain of f(x) is two.

dy 5x 4  2xy3  3x 2 y 2
175. Let the solution of the differential equation  4 be a curve given by C = 0 and passing
dx 5y  2yx 3  3x 2 y 2
through  0, 0  , then
(A) C = 0 is a homogeneous equation in (x, y)
(B) C = 0 represents exactly one straight line
(C) Area of region bounded by C = 0, x = 2 & y = 0 is 2 unit2
(D) slope of tangent to C = 0 at (1, 1) is 2

176. If the quadratic equations ax 2  bx  c  0 & cx 2  bx  a have exactly one common root, then which of the
following is correct?
(A) a + b + c =0 (B) b = 0 (C) a = c (D) a – b + c = 0

177. If x, y, z are positive numbers in A.P. then


x y y  z x y y z
(A) y 2  xz (B) y  2 xz (C)  2 (D)  4
2y  x 2y  z 2y  x 2 y  z

p  sin x q  sin x p  r  sin x 2

178. If p, q, r, s are in A.P. and f (x) = q  sin x r  sin x  1  sin x such that  f (x)d x = – 4 then the
0
r  sin x s  sin x s  q  sin x
common difference of the A.P. can be :
1
(A) –1 (B) (C) 1 (D) 2
2
COLLEGES: ANDHERI / BORIVALI / CHEMBUR / DADAR / KALYAN / KHARGHAR / NERUL / POWAI / THANE # 60
179. Let A   1,0  and B   2, 0  be two points on the x-axis. A point M is moving in the xy-plane in such a way
that MBA  2MAB . Then the point ' M ' moves along a conic whose
(A) Eccentricity is 1/2 (B) Latus – rectum is of length 6
(C) Directrices are 2 x   1 (D) Focii are  0, 2 

180. The lines L1 : x - 2y + 6 = 0 & L2 : x - 2y - 9 = 0 are tangents to the same circle. If the point of contact of
L1 with the circle is (- 2, 2), then
(A) the centre of the circle is (- 7/2, 5)
(B) the centre of the circle is (- 1/2, - 1)
(C) area of the circle is 45p/4 sq. units
(D) the point of contact of L2 with the circle has the co-ordinates (- 5, -7)

181. A set contains 5 elements. If the number of ways to construct 3 subsets of this set with replacement i s
N, then which of the following is correct
(A) N = 12005, if the subsets have exactly one common element
(B) N = 3430, if union of these subsets contains 3 elements
(C) N = 1024, if theses subsets are pairwise disjoint
(D) N = 85 1, if none of the three subsets is empty

COLLEGES: ANDHERI / BORIVALI / CHEMBUR / DADAR / KALYAN / KHARGHAR / NERUL / POWAI / THANE # 61
ANSW ER KEY
1. (AB) 2. (ABC) 3. (ABCD) 4. (BD) 5. (ABC)
6. (ABCD) 7. (AC) 8. (ABC) 9. (ABC) 10. (AB)
11. (BC) 12. (BC) 13. (AB) 14. (AB) 15. (AC)
16. (AC) 17. (ABC) 18. (AD) 19. (AC) 20. (BD)
21. (BC) 22. (BCD) 23. (AD) 24. (ABD) 25. (AC)
26. (AD) 27. (BD) 28. (AC) 29. (AB) 30. (ABCD)
31. (AD) 32. (ACD) 33. (BC) 34. (BC) 35. (B)
36. (CD) 37. (AB) 38. (CD) 39. (ABC) 40. (BC)
41. (A) 42. (AC) 43. (CD) 44. (AC) 45. (AC)
46. (ABD) 47. (ABCD) 48. (CD) 49. (ABCD) 50. (BD)
51. (BC) 52. (AC) 53. (AC) 54. (BD) 55. (ABC)
56. (CD) 57. (ABCD) 58. (ABCD) 59. (ABC) 60. (AB)
61. (AC) 62. (BC) 63. (ABD) 64. (AB) 65. (ACD)
66. (ABC) 67. (ABCD) 68. (AB) 69. (BCD) 70. (ABCD)
71. (AB) 72. (BD) 73. (ABC) 74. (D) 75. (BD)
76. (ABD) 77. (CD) 78. (ABCD) 79. (AB) 80. (ABCD)
81. (AB) 82. (AB) 83. (AC) 84. (ABC) 85. (AB)
86. (BC) 87. (ABC) 88. (ABCD) 89. (B) 90. (BD)
91. (AB) 92. (ABC) 93. (C) 94. (AB) 95. (BCD)
96. (ABCD) 97. (BC) 98. (ABCD) 99. (BC) 100. (BD)
101. (CD) 102. (AC) 103. (BC) 104. (BC) 105. (ABD)
106. (ABC) 107. (AC) 108. (BC) 109. (ABC) 110. (C)
111. (ABCD) 112. (AD) 113. (ABD) 114. (BD) 115. (BC)
116. (A) 117. (AD) 118. (BC) 119. (ACD) 120. (AB)
121. (CD) 122. (B) 123. (ACD) 124. (BC) 125. (AC)
126. (AB) 127. (BCD) 128. (AD) 129. (AC) 130. (BD)
131. (ABC) 132. (BC) 133. (D) 134. (ABC) 135. (AC)
136. (ABC) 137. (AC) 138. (ABC) 139. (AD) 140. (ABCD)
141. (ABD) 142. (BCD) 143. (ABCD) 144. (ABD) 145. (AB)
146. (CD) 147. (AD) 148. (AC) 149. (BCD) 150. (AB)
151. (BD) 152. (ABC) 153. (AC) 154. (AB) 155. (AB)
156. (AD) 157. (AC) 158. (BC) 159. (ABC) 160. (BCD)
161. (AD) 162. (ACD) 163. (ABC) 164. (AD) 165. (ABC)
166. (BD) 167. (AB) 168. (ABC) 169. (AD) 170. (ABCD)
171. (AB) 172. (BD) 173. (ACD) 174. (AB) 175. (ABC)
176. (AD) 177. (ACD) 178. (AC) 179. (BC) 180. (BC)
181. (ABC)

COLLEGES: ANDHERI / BORIVALI / CHEMBUR / DADAR / KALYAN / KHARGHAR / NERUL / POWAI / THANE # 62
INTEGER TYPE QUESTIONS
1.SN Tangents drawn from the point P(2, 3) to the circle x2 + y2 – 8x + 6y + 1 = 0 touch the circle at the points A
( x  5) 2 y  32
and B. The circumcircle of the PAB cuts the director circle of ellipse  = 1 orthogonally..
9 b2
Find the value of b2.

      
2. Let a, b, c are three vectors having magnitude 1, 1, 2 respectively if a  (a  c)  b  0 , and angle
  
between a & c is then find the value of k
k

3. Two lines zi  zi  2  0 and z 1  i   z 1  i   2  0 intersect at a point P. There is a complex number

  x  iy at a distance of 2 units from the point P which lies on line z 1  i   z 1  i   2  0 . Find [| x |]


(where [ . ] represents greatest integer function).

11 11
2  r   r 
4. Let a =  tan   and b =
 24 
  1r 1 tan 2  24  then find the value of log(2b – a)(2a – b)
r 1 r 1

5. The smallest positive integer n with 24 divisors (where 1 and n are also considered as divisors of n) is

1
1
6. Let f (x) 
8
 8sin 7
x cos x  8sin 3
x cos x  8sin x cos x  sin 8
x  2sin 4
x  4 sin 2
x  4
.

4
    5
Find  10  f    , given that f(0) = 0.
  2 

7. The number of angles   (0,  ) such that tan   sec  .sec   tan  . tan  & cos 2  0 is?

8. Let a, b, c, d be distinct integers such that the equation  x  a  x  b  x  c  x  d   9  0 has only repeated
integer root ‘r’, then the value of a  b  c  d  4r is equal to

9. Two different numbers are taken from the set {0, 1, 2, 3, 4, 5, 6, 7, 8, 9, 10}. The probability that their sum and
positive difference, are both multiple of 4, is x 55 then x equals?

10. From the sequence of first n natural numbers three consecutive terms are removed. Arithmetic mean of the
155
remaining numbers is . Value of n is?
11

11. The sum of 3 .nC0 8 .nC1 + 13 .nC2  18 .nC3 + . . . upto (n+1) terms is?

12. The slope of the tangent to curve y = f (x) at any of its point is given by 2x  4. If the curve passes through
the point (2 , 12) then 3 times the area of the region bounded by the curve , the x-axis & the line x + 1 = 0 is ?

13. In the triangle ABC coordinates of A are (1 , 2), Bisector of internal angle B is 2x – y = 5 and
perpendicular bisector of the side AC is 3x – 4y = 20. If Eq. of BC is ax + by = c, then c  a  ?
b
COLLEGES: ANDHERI / BORIVALI / CHEMBUR / DADAR / KALYAN / KHARGHAR / NERUL / POWAI / THANE # 63
 2 4 7 
14. Let A denotes the value of expression 4  cos + cos – cos – cos  and B denotes the value
 15 15 15 15 
of 8 cot (), where tan , tan , tan  are the real roots of the cubic x – 8(a – b) x2 + (2a – 3b) x –
3

4(b + 1) = 0. Find absolute value of (AB).

 sin x
1 
lim   sin x  x sin x
1 x 
15. The value of e  x 0  x   lim x
x 1  is?
 

16. The value of 'a2' for which the ordinate of vertex of the parabola y = x2  4ax + a4 assumes the least value is?

1 9
 e x 1  x  dx

17. Evaluate 1 8 .
 e x 1  x  dx


18. Points O, A, B, C ...... are shown in figure where OA = 2AB = 4BC = ....... so on. If A is the centroid of a
7 5
triangle whose orthocentre and circumcenter are (2, 4) and  ,  respectively. If an insect starts moving from
2 2
the point O(0, 0) along the straight line in zig-zag fashion and terminates ultimately at point P() then find the
value of ()

If the point  2a ,2a  a  is nearer to the point (1, 1) as compared to the line x + y = 0, then the least
2 2
19.
positive value of a is?

20. Find the area enclosed in the curve   x 4  x 2 y 2  y 4   x 2  y 2 .

21. Transeverse axis of a rectangular hyperbola is x + y = 2 & center is (1, 1). If the hyperbola passes through (5, 2)
then its T.A. = k 2 , where k = ?

If y = (1 + 1/x)x then e y '


2
22.  y '  yy" , at x = 1, is equal to ?

23. The circle passing through the distinct points (1, t) , (t, 1) & (t, t) for all values of ' t ', passes through the point
(a, b), then a2 + b2 is?

u 2  1
 11
24. If A   1 1 2  where u  and det  adj  adjA     23 Then the value of u is
4

3
 2 1 1 

25. If 7 = 2, then find the absolute value of the expression y = sec  + sec 2 + sec 4.


4  x10 e  x dx
2

26. The value of 0



is .
7  x 6 e  x dx
2

COLLEGES: ANDHERI / BORIVALI / CHEMBUR / DADAR / KALYAN / KHARGHAR / NERUL / POWAI / THANE # 64
27. Let p  x   x 5  x 2  1 have roots x1, x2, x3, x4 and x5, g  x   x 2  2 , then the value of

g  x1  g  x 2  g  x 3  g  x 4  g  x 5   30g  x1x 2 x 3 x 4 x 5  , is ?

28. The length of the legs of a right angled triangle are 1 and 2 . The smallest angle is . If the value of cos 8 can
p
be expressed in the lowest form as where p, q  N, then (p + q) equals
q

29. Let A, O, B & C be fixed points on a straight line such that OA = OB = BC = 1. A semicircle is drawn on AB as
diametre and a variable point P is chosen on the circumference such that POC   . If The value of  for

which the area enclosed as Region (APCBA) is maximum is , then k = ?
k

100
mnt
30. If  r.2
r 1
r 1
 m.2n  t , where m, n, t are positive integers with HCF 1, then value of
100
is?

1 2 dx 1 
31. Let I  a, b    , then 3 I  ,1 is?
 0  a 2 cos 2 x  b 2 sin 2 x  2
2 

32. The co-efficient of x5 in the expansion (1 + x + x2)15 is 3 nC4 + nC5 , then n is?

12   20  3 
33. If sin  = ,    ,   and cos  = ,   , 2  then the value of cosec( + ) can be
37 2  101  2 
p
expressed in the lowest form as (p, q  N) then find the value of (p + q).
q

34. If a, b, c are distinct and the vectors aiˆ  bjˆ  ck,


ˆ biˆ  cjˆ  akˆ and ciˆ  ajˆ  bkˆ are coplanar then the
value of a + b + c is

35. The maximum value of the function f(x) = 2x3 – 15x2 + 36x – 48 on the set A = {x| x2 + 20  9x} is

36. The triangle formed by the tangent to the parabola y = x2 at the point with abscissa h, the y-axis and the straight
line y = h2 has the greatest area where h  1,3 . Then the value of h is?

x f a  f a
4
37. Let f  x   x
,I   xf  x 1  x   dx and I 2   f  x 1  x   dx where
1
4 2 f 1a  f 1a 

I
f  a   f 1  a  then the value of 2
is
I
1

38. The reciprocal of the value of the product


 5   5   5   5   5   5 
sin  11  cos  11  cos  10  cos  9  ......... cos  3  cos  2  equals.
2  2  2  2  2  2 
COLLEGES: ANDHERI / BORIVALI / CHEMBUR / DADAR / KALYAN / KHARGHAR / NERUL / POWAI / THANE # 65
39. a, b, c are cube roots of p( p  0) then for any permissible value of x, y, z which is given that
xa  yb  zc
xb  yc  za

2 2

 a1  2b1   [ x]  [ y ]  [ z ]  2  0 where  is the cube root of unity and a1 , b1 are prime

numbers. Then [ x  a1 ]  [ y  b1 ]  [ z ] is equal to (where [.] denotes G.I.F.)

40. The minimum area bounded by the function y = f (x) and y = ax + 9 (a R) where f satisfies the relation f(x + y) =
f(x) + f(y) + y f (x) , x, y R and f(0) = 0 is 9A, value of A is

41. Find the number of ways in which six digits, 1, 2,.....6, respectively, can be assigned to six faces of a cube
(without repetition of digit) so that one arrangement cannot be obtained from another by a rotation of the cube .

  n n n 
42. lim  lim    ...  2 2 2 

x0 n  1  n 2 x 2
  4n x
2 2
n  n x  ?

11
43. Let A denotes the value of expression x4 + 4x3 + 2x2 – 4x + 7, when x = cot and B denotes the value of
8
1  cos 8 1  cos 8
the expression + , when x = 9°. Find the value of (AB).
tan 2 4 cot 2 4

44. If the equation x 4  ax3  bx 2  ax  1  0 has real roots, then minimum value of 5(a 2  b 2 ) is

45. Number of words of 4 letters that can be formed with the letters of the word IITJEE is N, then sum of digits of
N is?

46. Let a line be drawn through the point P(t, 1) to meet the parabola x2 = 4y in the points A & B. If PA  PB  3 t ,
then the maximum value of t is?

2
If I   x  ln x  dx , then the value of 4I  8  ln 2   4 ln 2 is?
2 2
47.
1

 2008 
48. Let 100nan  100  n  2  an 1  1 & a0  0 for all n  1 . Then   an  , where [x] denotes greatest integer
 n 0 
less then or equal to x, is equal to?
a
If the minimum value of the expression E  8   16 
cos x sin x
49. can be expressed in the form , where a & b
b
are least possible natural numbers, then (a + b) is?

50. If 0 < [x] < 2 ; –1 < [y] < 1 and 1 < [z] < 3. Where [.] denotes the greatest ineger function, then the maximum
[ x ]  1 [ y] [z]
value of the determine [ x ] [ y]  1 [z] is ?
[x] [ y] [ z ]  1

51. A drawer contains a mixture of red socks and blue socks, at most 17 in all. It so happens that when two socks
are selected randomly without replacement, there is a probability of exactly 1 2 that both are red or both are
blue. The largest possible number of red socks in the drawer that is consistent with this data, is?
COLLEGES: ANDHERI / BORIVALI / CHEMBUR / DADAR / KALYAN / KHARGHAR / NERUL / POWAI / THANE # 66
2
sec 2 2 x  1
52. The value of  1  tan 2 2 x
d x is?
0

 2 2 2 
53. If A =  sin x  cos 2 x  , then det.[adj {adj {adj ..... {adj A }}}]. [ Adjoint is taken ‘r’ times ]
2
 1
cos 2 x cos 4 x sin 4 x 

 1 2 cos  4 cos 3 8 cos 7 16 cos 15


54. If  = then find value of S = + + + +
31 cos  cos 2 cos 4 cos 8 cos 16

55. Let Pi and Pi' be the feet of the perpendiculars drawn from foci S, S' on a tangent Ti to an ellipse whose
10
length of semi-major axis is 20. If  (SPi ) (S' Pi' )  2560 , then find the value of 100e, where 'e' denotes
i 1
eccentricity.
y
du d2y
56. If x =  , then = ky where k has the value equal to?
0 1  9 u2 dx 2

57. An ellipse is drawn with major and minor axes of lengths 10 and 8 respectively. Using one focus as
centre,a circle is drawn that is tangential to the ellipse, with no part of the circle being outside the ellipse.Find
radius of the circle.

58. Two freinds A & B are playing a game in which a coin is flipped. If it shows HEAD ‘A’ gets a point and if it shows
TAIL ‘B’ gets a point. The one who gets 20 points first wins 8 Silver coins. At a point when ‘A’ is leading B with
14 to 10 points, if the game is stopped, then the number of coins should be given to ‘B’ in a fair distribution is?

59. The least positive integral value of the parameter 'a' for which the function
f(x) = 8ax a sin 6xsin 5x 7x , increases & has no critical points for all x R, is?
Sn 1 15
60. If Sn = nC0 . nC1 + nC1 . nC2 + ...... + nCn - 1 . nCn and if = then the sum of two possible values of n
Sn 4
is?

1/n
  2 3 n 
61. Value of lim  tan tan tan ... tan  is equal to?
n 
 2n 2n 2n 2n 

62. Tangents are drawn from the point (1, 2) on the parabola y2 = 4 x . If the length these tangents will intercept
on the line x = 2 is k, then k2 =?

 4   4    4  
3
63. Lim
If Sn = 1  tan 3 1  tan 4 .........1  tan n  then find n 
 2  2   2  Sn

64. The common chord of the circles S1 : x2 + y2 = 8 and S2 : (x - a)2 + y2 = 8 subtends a right angle at the origin.
Positive Value of a must be?

/2 / 4 I1
65. If I1=  n (sin x) dx & I2 =  n (sin x  cos x) dx, then the value of
0  /4
I 2 is?
COLLEGES: ANDHERI / BORIVALI / CHEMBUR / DADAR / KALYAN / KHARGHAR / NERUL / POWAI / THANE # 67
66. If the complex numbers Z1, Z2, Z3 represent the vertices of an equilateral triangle such that | Z1 | = | Z2 | =
| Z3 |, then Z13  Z23  Z33  3Z1Z 2 Z3 is?

1 | x |, | x | 1
67. Let f (x) =  and g (x) = f (x – 1) + f (x + 1). Then the number of points where g is not differentiable
0, | x | 1
is

68. In ABC, circumradius is 3 and inradius is 1.5 units. If the value of a cot2A + b2cot3B + c3cot4C
is m n where m and n are prime numbers, then find the value of (m + n).

69. Let x & y be the real numbers satisfying the equation x2 - 4x + y2 + 3 = 0. If the maximum and minimum values
of x2 + y2 are M & m respectively, then the numerical value of (M - m) is?

/ 2 / 2
70. If the value of the integral  x 2 cos ec 2 x dx is – k  log sin x dx , then the numerical quantity k should be?
0 0

71. The position vectors of two points A and C are 9iˆ  ˆj  7kˆ and 7iˆ  2ˆj  7kˆ respectively. The point of
intersection of vectors AB = 4iˆ  ˆj  3kˆ and CD = 2iˆ  ˆj  2kˆ is P. If vector PQ is perpendicular to AB
and CD and PQ = 15 units, then the position vector of Q is 6iˆ  9ˆj  µkˆ find the value of µ.

 
3/2 12
1 2 1 1
72. If f  x   a cos   x   b, f '     and  f  x  dx    1 , then find the value of,  sin a  cos b .
2 1/2 

1 1 2r
73. Let ai & bi be two sequences of real numbers such that a  a  r 4  3r 2  1 &
r r 1

2br  ar  2  3ar 1 , r  N . The value on r for which br attains maximum value is?

74. The perimeter of a right triangle is 12 + 8 3 . The sum of the square of all three sides is 294 sq. units.
If the area of the triangle in square units is , then find 2.

75. Let f (x) be a differentiable function such that f '(0)  1 , and the sequence {an } is defined as a1  2 and
2
lim 2  a   9
f  n 1   f (0)  , n  N if  ai  2 2 1 then k is?
k
an  x
x     x   i 1

  
76.      
Consider the three planes r. i  j  ak  2, r. 2i  a j  k  1, r. ai  2 j  2k  1. If The three planes

intersect in a common line, then the shortest distance of this line from the line r  i   2    j is?

/2
k
77. If  sin  log sin  d  log
e
, then the numerical quantity k should be?
0

78. If Z 
1
2
  
3  i and the least positive integral value of ' n ' such that Z 101  i109 
106
 Z n is ' k ' then 2 k / 5 = ?

COLLEGES: ANDHERI / BORIVALI / CHEMBUR / DADAR / KALYAN / KHARGHAR / NERUL / POWAI / THANE # 68
79. In a knockout tournament 2n equally skilled players; S1, S2, ............. S are participating. In each round
2n

players are divided in pair at random and winner from each pair moves in the next round. If S2 reaches the
1
semifinal then the probability that S1 wins the tournament is . The value of 'n' equals?
20

80. If circumradius and inradius of ABC be 10 and 3 respectively, then find the value of a cotA + b cotB + c
cotC.

81. Locus of point of intersection of the lines given by the equations, x cos   y sin   4 , x cos   y sin   4 ,
 
where  &  are variables satisfying 2 sin sin = 1, is a parabola, length of whose latus rectum is?
2 2

x
c tan dy
82. If y = e k
is a solution of the differential equation sin x = y logy then the numerical quantity k should
dx
be?

2 3
83. Let f  x   x 3  3x 2  2x , then find the number of solutions of f  x   k , such that 0  k  , is?
9

 
84. If and  are the roots of the equation x 2  px  q  0 and also x 3900  p1950 x1950  q1950  0 and if , are
 
the rooots of x n  1   x  1  0 , then the value of ‘n’ must be equal to?
n

85. The altitudes from the angular points A,B and C on the opposite sides BC, CA and AB of ABC are 210, 195
m
and 182 respectively. If the length of the side BC can be expressed as rational (in the lowest form ), then find
n
(m + n).

86. In the adjacent figure, ‘P’ is any arbitrary interior point of the triangle ABC, H a , H b and H c are the length of
altitudes drawn from vertices A, B and C respectively. If xa , xb and xc represent the distance of ‘P’ from sides
xa xb xc
BC, AC and AB respectively then H  H  H is allways equal to?
a b c

1 x 1  2x 1  3x
87. Sum of the series 1  nC1 + nC2 2  C3
n
+ ...... to (n + 1) terms is equal to.
1  nx 1  nx 1  nx3

88. In a multiple choice question there are four alternative answers of which one or more than one is correct. A
candidate will get marks on the question only if he ticks all the correct answers. The candidate ticks the
answers at random. If the probability of the candidate getting marks on the question is to be greater than or
equal to 1/3 the least number of chances he should be allowed is?

4x 4x
89. Normal to the curve y = f(x) at (1, 1) is 3x + 4y = 7. Also f  x   for x  1 and f  x   for x  1 ,
3 3
3f '  x   2f  x   2x
where f(x) is twice differentiable everywhere in its domain. Evaluate lim .
x 1 f  x   x2
COLLEGES: ANDHERI / BORIVALI / CHEMBUR / DADAR / KALYAN / KHARGHAR / NERUL / POWAI / THANE # 69
90. The numebr of integral values of y for which the chord of the circlce x2 + y2 = 125 passing through the point
P(8, y) gets bisects at the point P(8, y) and has integral slope is?

91. If a, b, c be three natural numbers in A.P. and a + b + c = 30, then the possible number of values of a, b, c is?

92. If r1 & r2 are two distinct roots of tan x  x and I   2sin  r1t  sin  r2 t  dt then value of I ?
0

93. Circles are drawn on chords of the rectangular hyperbola xy = 4 parallel to the line y = x as diameters. All
such circles pass through two fixed points for which |x| + |y| = ?

x 2 y2
94. Consider the ellipse   1 , having it’s eccentricity equal to e. P is any variable point on it and P1, P2
a 2 b2
are the foot of perpendiculars drawn from P to the x and y-axis respectively. The line P1P2 will always be
a normal to an ellipse whose eccentricity is equal to k.e, then find the value of k.

95. In  ABC, if C = 3A, BC = 27 and AB = 48, then find the length of the side AC.

96. If the co-efficient of x4 in the expansion of (1 x + 2x2)12 is n + 2Cr + 1 + r. n + 1Cr + nCr, then the coefficient of xr in
the expansion of (1 x )n / 2 is .

Let A      sin    sin   e d & B      cos    sin   e cos  d ,where A(0) = -1 & B(0) = 0. Find
cos 
97.

the value of

ln  A       B    
2 2
.
cos 

98. A trapezium is inscribed in the parabola y2 = 4x such that its digonals pass through the focus and are of length
25/4 units. If area of this trapezium is A, then 4A is?

 x
99. If f  x, y   f  xy,  for all nonzero real x & y and f(k, 1) = 4 for all integral values of k, then find
 y
f (256, 256).
k 20

k 1
2
  6
100. If   ei2 7 and f  x   A 0   A k x and the value of f  x   f  x   f  x  .....  f  x is  

k A A x A x
0 7
7
14
14
 then find the value of k.
101. The ratios of the lengths of the sides BC and AC of  ABC to the radius of circumscribed circle are equal to 2

and
3
respectively. If the ratio of the lengths of the bisectors of the interior angles B and C is
  1 
where

2  
, ,   N, then find the value of ( +  + )

102. In a triangle ABC, A  ( ,  ) , B  (1, 2) , C  (2,3) and point A lies on the line y  2 x  3 , where  ,   I .
If the area of triangle ABC be such that [ ]  2 , where [.] denotes the greatest integer function, then the number
of all possible coordinates of A must be?

COLLEGES: ANDHERI / BORIVALI / CHEMBUR / DADAR / KALYAN / KHARGHAR / NERUL / POWAI / THANE # 70
103. Let A be the set of all 3  3 symmetric matrices whose elements are selected from {0, 1, 2} & exactly 6 of the
elements of both A & B are 1. Number of all such matrices is?

104. Let f (x)  x3 12x2  3ax  2 . If the largest possible interval in which f(x) is a decreasing function is
(–3, –1) then find the value of ‘a’ ?

105. In the process of numbering the pages of a book, the printer uses 1890 digits. The numbers of pages in the book
is?

 /4  1  tan x   ln x / 2  x
106. I1  0 ln   dx, I 2  1 dx & I3   ln  tan  dx .
 1  tan x  1 x 2 0
 2
If mI1 = I3 & nI1 = I2, then state (m + n).

107. The least integral positive values of ' a ' for which the equation, x  1  2a  x  a  1  0 has no real solutions
4 2 2

is?

108. Two boys A and B find the jumble of n ropes lying on the floor. Each takes hold of one loose end randomly. If the
1
probability that they are both holding the same rope is then the number of ropes is equal to?
101

109. Tangents parallel to the three sides of  ABC are drawn to its incircle. If x, y, z be the lengths of the parts of the
x y z
tangents within the triangle (with respect to the sides a, b, c) then find the value of   .
a b c

200 r 200
110. Consider two polynomials f (x) and g (x) as g  x     r x and f  x     x .
r
r 0 r
r 0
200
Given (i)   1r  100 , (ii) f (x + 1) = g (x). Let A    r .
r r 100
Find the remainder when A is divided by 15.

111. An unbiased normal coin is tossed 'n' times. Let


E1 : event that both Heads and Tails are present in 'n' tosses.
E2 : event that the coin shows up Heads atmost once.
The value of 'n' for which E1 and E2 are independent, is?

112. A real valued function satisfies following conditions :


(i) f  x  1  f  x  1  2f  x  for x < -1 or x > 7.
(ii) f(x) = x for 0  x  7 .
Find the value of f(87)
x 2 y2
113. From a point P two tangents are drawn to   1 , such that their chord of contact subtends right angle at
2 3

x 2 y2
its center. If locus of P is the ellipse   1 , then 6(a2 + b2) =?
a 2 b2

114. The value of the expression n+1


C2 + 2 (2C2 + 3C2 + 4C2 + ...... + nC2) , for n = 6, is ?

COLLEGES: ANDHERI / BORIVALI / CHEMBUR / DADAR / KALYAN / KHARGHAR / NERUL / POWAI / THANE # 71
/ 2
115. Let In   0  sin x  cos x  dx  n  2  , then find the value of nI n  2  n  1 In 2 .
n

116. An ellipse of major axis 4 and minor axis 3 is drawn tangential to the lines x  3 y  2  0 & 3 x  y  4  0 .
Centre of this Ellipse will always lies on the circle. Find the radius of this circle.

117. Let the lengths of the altitudes drawn from the vertices of a  ABC to the opposite sides are 2, 2 and 3. If the
area of  ABC is  then find the value of 2 2  .

118. All the face cards from a pack of 52 playing cards are removed. From the remaining pack half of the cards are
randomly removed without looking at them and then randomly drawn two cards simultaneously from the
p( 38C 20 )
remaining. If the probability that, two cards drawn are both aces, is 40 , then the value of p is?
C 20 · 20C 2

 2 x   15 1 
119. A real valued function satiffies f  x, y   f  x y, 2  & f  2, 2   4 . Find the value of f  2 , 5  .
 y   2 
1
120. If f  x   17x    60t 2  12x 2 t  f  t  dt , then find the value of f(0).
0

121. Line l is a tangent to a unit circle S at a point P. Point A and the circle S are on the same side of l, and the distance
from A to l is 3. Two tangents from point A intersect line l at the point B and C respectively. Find the value of
(PB)(PC).

122. The number of ways in which 4 distinct balls can be put into 4 boxes labelled a, b, c, d. so that
exactly one box remains empty is n. Then n =?

123. The lower corner of a page in a book is folded over so as to reach the inner edge of the page. If the fraction
of the width folded over when the area of the folded part is minimum is k/3, then k is?

124. If two of the three normals drawn from a point (x, y) to the parabola y 2  8 x are concurrent, then

4  x  4  ky2 , where k is?


3

6
1
max | 2 | x ||, 4 | x |, 3 dx is.
19 6
125. The value of

126. A sequence is obtained by deleting all perfect squares from set of natural numbers. The remainder when the
2003rd term of new sequence is divided by 2048 , is?

127. If a  b  c  0 & a 2  b 2  c 2  1 then the value of 2a4 + 2b4 + 2c4 is?

128. A circle is inscribed in a triangle with sides of lengths 3, 4 and 5. A second circle, interior to the triangle, is tangent
to the first circle and to both sides of the larger acute angle of the triangle. If the radius of teh second circle can
sin k
be expressed in the form where k and w are in degrees and lie in the interval (0, 90°), find the value of
cos w
k + w.

COLLEGES: ANDHERI / BORIVALI / CHEMBUR / DADAR / KALYAN / KHARGHAR / NERUL / POWAI / THANE # 72
    1
129. ˆ and ˆ are two unit vectors and r is a vector such that r . ˆ  0 and 2(r  ˆ )  3(r  ˆ )  ˆ , then  2
|r|
equals?
25 1 4
130. If the area bounded by the curves x2 + y2 = 25, 4y = 4  x and x = 0 above the x-axis is sin
2
 k , then
2 5
the value of ‘k’ is?

131. A variable straight line is drawn through the point P(1, 1) to meet the lines x + y = 3, 3x + 4y = 12 & 4x + 3y = 12
10 5 7
respectively in points A, B & C. Find the greatest value of   .
PB PC PA

b cos x b  sin x
132. The greatest integral value of ' b ' such that the equation 2 cos 2 x  1  posses solu-
(cos x  3sin 2 x) tan x
2

tions, is?

n
x
133. Let fn(x) = cosn x & g  x   lim
n 

r 0
f r   . Find the largest value of ‘a’ such that g(x) is continuous in the
4
interval (0, a).

134. If P is a moving point in the xy plane in such a way that perimeter of triangle PQR is 16 units, where Q is (3, 5 )
& R is (7, 3 5 ) then maximum area of triangle PQR is?

10
 3  m
135. If  tan 1 9r 2  3r  1  = cot 1 n  (where m and n are coprime), then find (2m + n).
r 1

136. The number of seven digit integers, with sum of the digits equal to 11 and formed by using the digits 1, 2 and 3
only, is
90

 (x  r)2 , r  1  x  r  1  f (x)dx
137. Let f be a function defined by f (x)   , where r  3k, k  I then 0
is equal
 1, r 1  x  r  2 10
to?

dx 3y
138. The differential equation dy = represents a family of hyperbolas (except when it represents a pair of lines)
2x
with eccentricity e. Number of possible values of e is ?

x nf  x   h  x   1 1
139. Let g  x   lim be continuous at x = 1 & g 1  lim  ln  ex   ln x .Find the value of
n  2x  3x  3
n
x 1

2  f 1  g 1   h 1 .

140. Find the value of


[cos1– cos–11] – [sin1– sin–11]+[tan1– tan–11] – [cot1– cot–11]+[sec1 – sec–11] – [cosec1 – cosec–11] where [x]
denotes greatest integer less than or equal to x.

COLLEGES: ANDHERI / BORIVALI / CHEMBUR / DADAR / KALYAN / KHARGHAR / NERUL / POWAI / THANE # 73
141. If a chord of the circle x2 + y2 - 4x - 2y - 10c = 0 is trisected at the points,  1 , 1  and  ,  then the value
8 8
 3 3  3 3
of c is?

142. An n-digit number is a positive number with exactly n-digits. Nine hundred distinct n- digit numbers are to be
formed using only the three digits 2, 5 and 7. The smallest value of n for which this is possible is?

143. Normal chords to the parabola y2 = 8x at the point P(2, 4) & Q meets the curve again in RIf Circum center
of PQR is (h, k) then h + k = ?
f x
144. If the function f :  0,    R satisfy the relation f  x  e    x , then find lim
f x
.
x  ln x

        
145. Let A, B, C be vectors of length 3, 4, 5 respectively. Let A be perpendicular to B  C, B to C  A and
     
C to A  B and if the length of vector | A  B  C | is k 2 then find the value of k.

1 50 1
146. If Z1 , Z2 , Z3 , . ., Z50 , are the roots of the equation 1+ Z +Z + . . . + Z = 0, then the value of 
2 50
5 r 1 1  Zr
is?

d 2U n
147. Given that U n  x n (1  x) n for n  N , n  2 and  n(n  1)U n 2  2n(2n  1)U n 1 further if
d 2x
1
Vn   e xU n dx , then for n  2 , Vn  k1n(2n  1)Vn 1  k2 n(n  1)Vn2  0 where k1 , k2  I then k1  k2 is equal
0

to?

148. Let P be a point on the hyperbola x 2  4 y 2  4 , N being the foot of the perpendicular from P on the transverse
axis. The tangent to the hyperbola at P meets the transverse axis at T1 and O is the centre of the hyperbola
OT1 .ON = ?

149. If range of the function f(x) = sin–1x + 2 tan–1x + x2 + 4x + 1 is [p, q] then find the value of (p + q).

d 2f  x   df  x  
2

150. Let 2  f  x    f x     0 & f  0   f 1  1 . Area of region bounded by y = 0, x = 0,


2

dx 2  dx 
 e1/a  1 
2
x = 1 & y = (2x – 1)f(x) is  1/a  , then ‘a’ is?
 e 

151. The Least integral value of   x  satisfying the inequality x 2  5x  24  x  2 is?

1
152. Let f  x   & g(x) = x2 – 3x + 2. Number of points at which f(f(g(f(f(x))))) is not differentiable is?
x

153. The shortest distance from the point (1, 2, 3) to x 2  y 2  z 2  xy  yz  zx  0 is ‘d’, then d2 =?

COLLEGES: ANDHERI / BORIVALI / CHEMBUR / DADAR / KALYAN / KHARGHAR / NERUL / POWAI / THANE # 74
154. Two circles whose radii are equal to 4 and 8 intersect at right angles. Length of one of their common tangents is?

1 100    1 1  1    1 1  1 
155. Find the value of   tan   cos    tan   cos   .
2 n 1   4 2  n  4 2  n  

156. If the coefficient of xr (0  r  n 1) in the expression (x + 2)n-1 + (x + 2)n-2 . (x + 1) + (x + 2)n-3 . (x + 1)² + ......
10
b7
+ (x + 1)n-1 is nbr , then is equal to?
10

 2 t  sin x  
157. If the two lines AB :     1 dx  x + y = 3t and AC : 2t x + y = 0 intersect at a point A, then
0  x  

p
x-coordinate of point A as t  0, is equal to (p and q are in their lowest form). Find (p + q).
q

158. Consider a complex number Z satisfying Z  3  Z  Z  3  12 . If the greatest integer less than or equal to
Z is ‘a’ and the least integer greater than or equal to is ‘b’, then the value of ‘a + b’ is?

159. Number of ordered tripplets  a, b, c  where 1  a, b, c  10, such that 2a  3b  5c is a multiple of 4, is?

f x  – 5
160. Let f : R  R be a continuous function satisfying f (x + 1) = x  R  4n : n  Z . If f(0) = 1, then
f x  – 3 
find f(4).

161. A parabola is drawn through (2, 0) & (2, 0), having a tangent to x 2  y 2  a 2 as its directrix. If the locus
of focus of this parabola is an ellipse, then find the least integral value of a.


162. If f (x) = a | cos x | + b | sin x | (a, b  R) has a local minimum at x = – and satisfies
3
2

 f (x ) 
2
dx = 2. Find the values of a and b and hence find b 2 a 2
 2

163. If the equation x 3  bx 2 y  cxy2  y3  0 represents three straight lines of which two are mutually perpendicular,,
then the value of (b + c) is ?

( 2x 3  1) dx A 1 C A
164. If the value of the definite integral I =  6 3 2 can be expressed in the form cot where
1 x  2x  9x  1 B D B
C
and are rationals in their lowest form, find the value of (A + B2 + C3 + D4).
D

165. If , are the roots of the quadratic equation, x 2  2p  x  4   15  0 , then the greatest integral value of
p for which one root is less than 1 & the other root is greater than 2 is?
COLLEGES: ANDHERI / BORIVALI / CHEMBUR / DADAR / KALYAN / KHARGHAR / NERUL / POWAI / THANE # 75
166. Let , ,  and  be the roots of equation x4 – 3x3 + 5x2 – 7x + 9 = 0. If the value of
a
| tan (tan–1 + tan–1 + tan–1 + tan–1 ) | =
b
where a and b are coprime to each other, then find the value of (ab + ba + aa + bb + ab).

167. Entries of a 2 × 2 determinant are chosen from the set {–1, 1}. The probability that determinant has zero value
is a/b, where a and b are coprime, then find a + b.

  – 3 
1 1
xn ex x n ex
168. The value of lim 2 (where n  N) is
x  xn

169. From an external point P, two tangents are drawn to the parabola x2 = 8y. If chord of contact of these tangents
is tangential to the circle x2 + y2 = 4, then locus of P will be a hyperbola length of whose transeverse axis is?

170. How many terms of the sequence cot–1 3, cot–1 7, cot–1 13, cot–1 21, ..................must be taken to have their
1  24 
sum equal to cos–1  .
2  145 
n
  n ln  e/ 2 1
171. lim     sin  (when n  Q) is equal to?
n   n  1 n 
 

172. If  and  are the roots of the equation x2 – 4x + 1 = 0 ( > ) then find the value of

3  1 1    3  1 1  
f (, ) = cosec2  tan  + sec2  tan .
2 2  2 2  

173. The distance between the two parallel lines is 4 3  3 unit . A point 'A' is chosen to lie between the lines at a
distance 4 3  3 from one of them. Triangle ABC is equilateral with B on one line and C on the other parallel
line. Find length of the side of this equilateral triangle.

 
174. Consider the curve y = tan–1x and a point A 1,  on it. If the variable point Pi (xi, yi) moves on the curve for
 4
r
 1 
i = 1, 2, 3, ..... n (n  N) such that yr =  tan 1 2m 2  and B(x, y) be the limiting position of variable point Pn
m 1
as n  , then the value of reciprocal of the slope of AB will be

f (x)g(x) [f (x)]g(x ) 1
2
175. If f(x) = log10x and g(x) = eix and h(x) = f (x 2 )g(x 2 ) [f (x 2 )]g( x )
0 then the value of h (10) is?
3
f (x 3 )g(x 3 ) [f (x 3 )]g(x )
1

COLLEGES: ANDHERI / BORIVALI / CHEMBUR / DADAR / KALYAN / KHARGHAR / NERUL / POWAI / THANE # 76
n  x 
176. Let R denotes the value of f ' (2) where f (x) = Lim
n 
 tan 1 n(n  1)  x 2 
n 1  
S denotes the sum of all the values of x satisfying the equation
6
tan–1(x + 1) + tan–1(x – 1) = tan–1   .
 17 
1
T denotes the value of the expression 2 sin x + cos x + 4 tan x where x = 2 tan–1  
3
Find the value of 5R + 6S + 7T.

177. If 2p  3n  6m is NOT a multiple of 5, where p, m, n  {1, 2, 3, 4, ..., 36}, then the number of
values of ‘p’ is?

178. Number of values of ' p ' for which the equation ,

p 2
  
 3p  2 x 2  p 2  5p  4 x  p  p 2  0 possess more than two roots, is?

1 1 1 1 1 1


179. If A = cot–1   + cot–1   + cot–1   and B = 1 cot–1(1) + 2 cot–1(2) + 3 cot–1(3) then | B – A | is
1 1 2 2 3  3
a c
equal to + cot–1(3) where a, b, c, d  N and are in their lowest form, Find (a + b + c + d).
b d


 
180. Find the distance of the point i  2j  3k from the plane r . i  j  k  5 measured parallel to the vector

2i  3j  6k .

181. If Z1 and Z 2 are two complex numbers such that Z1  2 and 1  i  Z 2  1  i  Z 2  8 2, then the minimum
value of Z1  Z 2 is?

182. A circle of radius 4 units is inscribed in an equilateral triangle ABC, then an equilateral triangle is inscribed
in the circle, a circle again is inscribed in the later triangle and so on. In this way the process continues infinitely.
If r , x1 , x2 , . . ., xn, . . . be the radii of the these circles respectively, then the sum of radii of all the circles is?
x 2 y2
183. Area of triangle formed by the asymptotes and any tangent to 2  2  1 is 36 sq. units, where a & b are
a b
integers, then number of such hyperbolas is?

x2 y2
184. If the normal at any given point P on ellipse + = 1 meets its auxillary circle at Q and R such that
a 2 b2
a 4  2b 4
angle QÔR = 90 , where O is centre of ellipse, then the least value of 2 2
0
is?
5a b  2a 3 b
185. Number of all five digit numbers of the forms 34x5y (x & y are digits) divisible by 36, is?

1 x x 1
186. f(x) = 2x x(x  1) (x  1)x then f(100) is equal to?
3x(x  1) x(x  1)(x  2) (x  1)x(x  1)

COLLEGES: ANDHERI / BORIVALI / CHEMBUR / DADAR / KALYAN / KHARGHAR / NERUL / POWAI / THANE # 77
ANSW ER KEY
1. [54] 2.  3. [01] 4. [2] 5. [360]
6. [07] 7. [00] 8. [00] 9. [06] 10. [25]
11. [00] 12. [07] 13. [12] 14. [4] 15. [02]
16. [02] 17. [09] 18. [8] 19. [01] 20. [04]
21. [04] 22. [4] 23. [02] 24. [04] 25. [4]
p 17
26. [09] 27. [07] 28. 98 where,  29. [03]
q 81
30. [02] 31. [10] 32. [16] 33. [7442] 34. [00]
35. [07] 36. [03] 37. [02] 38. [1024] 39. [06]
40. [72] 41. [30] 42. [01] 43. [12] 44. [04]
45. [03] 46. [04] 47. [03] 48. [10] 49. [09]
50. [04] 51. [10] 52. [04] 53. [01] 54. [0]
55. [60] 56. [09] 57. [02] 58. [03] 59. [07]
60. [06] 61. [01] 62. [72] 63. [32] 64. [04]
65. [02] 66. [00] 67. [05] 68. [16] 69. [08]
70. [02] 71. [09] 72. [06] 73. [01] 74. [108]
75. [09] 76. [02] 77. [02] 78. [04] 79. [04]
80. [0026] 81. [08] 82. [02] 83. [08] 84. [1950]
85. [0849] 86. [01] 87. [00] 88. [05] 89. [07]
90. [06] 91. [19] 92. [00] 93. [04] 94. [01]
95. [35] 96. [20] 97. [02] 98. [75] 99. [04]
100. [07] 101. [18] 102. [04] 103. [44] 104. [09]
105. [666] 106. [01] 107. [02] 108. [51] 109. [1]
110. [01] 111. [03] 112. [07] 113. [65] 114. [91]
115. [02] 116. [05] 117. [9] 118. [06] 119. [04]
120. [09] 121. [3] 122. [144] 123. [02] 124. [54]
125. [02] 126. [00] 127. [01] 128. [54] 129. [07]
130. [02] 131. [05] 132. [00] 133. [08] 134. [12]
135. [32] 136. [161] 137. [05] 138. [02] 139. [01]
140. [4] 141. [02] 142. [07] 143. [08] 144. [01]
145. [05] 146. [05] 147. [01] 148. [04] 149. [0004]
150. [04] 151. [03] 152. [05] 153. [02] 154. [08]
155. [5050] 156. [84] 157. [05] 158. [07] 159. [500]
160. [01] 161. [03] 162. [03] 163. [02] 164. [99]
165. [02] 166. [5050] 167. [03] 168. [00] 169. [04]
170. [11] 171. [02] 172. [56] 173. [10] 174. [2]
175. [00] 176. [38] 177. [09] 178. [01] 179. [40]
180. [07] 181. [02] 182. [08] 183. [10] 184. [01]
185. [03] 186. [00]

COLLEGES: ANDHERI / BORIVALI / CHEMBUR / DADAR / KALYAN / KHARGHAR / NERUL / POWAI / THANE # 78
COMPREHENSION TYPE QUESTIONS
PARAGRAPH - 1
Directrix of a parabola is x + 2y = 3 and it is touching the line x + y = 2 at (0 , 2). Answer the following questions

1. Equation of tangent to the parabola at the vertex is__


(A) 10 x  20 y  39 (B) 10 x  20 y  39 (C) 10 x  20 y  39 (D) 10 x  20 y  39

2. Equation of the axis of this parabola can be___


(A) 10 x  5 y  7  0 or 2 x  y  2  0 (B) 10 x  5 y  7  0 or 2 x  y  2  0
(C) 10 x  5 y  7  0 or 2 x  y  2  0 (D) none of these

PARAGRAPH - 2
Two of the vertices of a triangle of area A lie on the lines 3x  4 y  2, 4 x  3 y  2 and the third vertex is
(2, 2). Let the perimeter of the triangle be c units and the length of the side along 4 x  3 y  2 is proportional to
the perimeter.

3. If c is an integral multiple of length of the side along 4 x  3 y  2 , then area of triangle is


(A) maximum (B) minimum (C) constant (D) none of these

4. If the ratio of c and length of the side along 4 x  3 y  2 is not a rational number, then Area of the triangle will
be maximum if
(A) a : b  1: 2 (B) a : b  2 :1 (C) a = b (D) none of these

5. If the maximum area of the triangle is 20 2  28 unit2, then c is equal to


(A) 1 (B) 100 (C) 10 (D) none of these

PARAGRAPH - 3
If the line ax  y  1 meets the circle x 2  y 2  2 x  4 y  4  0 in two points P & Q, and O is the origin, then
answer the following questions.

6. Locus of point of intersection of tangents drawn to the circle at P & Q is


(A) x  3 y  2  0 (B) x  3 y  2  0 (C) x  3 y  2  0 (D)None

7. Number of possible values of a such that POQ  , is
2
(A) 3 (B) 1 (C) 2 (D) 4

8. If POQ  , then the possible point of intersection of tangents to the given circle at P & Q is
2
 5 1  14 8  5 1  14 8 
(A)   ,  or  ,  (B)  ,  or   , 
 4 4  5 5 4 4  5 5
5 1  14 8   5 1  14 8 
(C)  ,  or  ,  (D)   ,  or   , 
4 4  5 5  4 4  5 5

COLLEGES: ANDHERI / BORIVALI / CHEMBUR / DADAR / KALYAN / KHARGHAR / NERUL / POWAI / THANE # 79
PASSAGE - 4
In  ABC as shown, XX1 = d1 ; XX2 = d2; XX3 = d3 and X is the centre of the circumscribed circle around
the ABC. a, b and c as usual are the sides BC, CA and AB respectively.
a b c  abc
9. If      = , then the value of '' is equal to
 d1 d 2 d 3  d1d 2d 3
(A) 1 (B) 2 (C) 4 (D) 8

10. If R is the radius of the circumcircle of the ABC and a(d2 + d3) + b(d3 + d1) + c(d1 + d2) = kR(a + b + c)
then the value of 'k' is
1 1
(A) 1 (B) (C) (D) 2
2 3

11. Let ha, hb and hc are the altitudes of the ABC from the angular points A, B and C respectively.
If (a2 + b2 + c2) = t (had1 + hbd2 + hcd3) then 't' equals
(A) 1 (B) 2 (C) 3 (D) 4

PARAGRAPH - 5
Three circles touch each other and have a common tangent as shown in the adjoining figure.If R1 = 1 & R3 = 4
and centers of the first and the third circles in order of appearance are  2,1 &  2, 4  .

12. Radius of second circle is


(A) 2 (B) 4/9 (C) 3 (D) 1/3

13. Coordinates of centre of the second circle are (a, b), then
(A) 12a  9b  4 (B) 12a  9b  4 (C) 12a  9b  4  0 (D) None of these

14. Slopes of common tangent of the three circles is


(A) {0, 3/4} (B) {0, 24/7} (C) {0, –3/4} (D) {0, –24/7}

PARAGRAPH-6
Let S1 & S2 be the two foci of an ellipse and P be a point of the ellipse such that S1PS2 is a right angle. Area &
perimeter of triangle S1PS2 be respectively 30 & 30.

15. Length of major axis of the ellipse is


(A) 17 (B) 17/2 (C) 15 (D) 15/2

16. Distance between the focii is


(A) 17 (B) 13/2 (C) 13 (D) 15/2

COLLEGES: ANDHERI / BORIVALI / CHEMBUR / DADAR / KALYAN / KHARGHAR / NERUL / POWAI / THANE # 80
17. Eccentricity of the ellipse is
(A) 17/15 (B) 13/15 (C) 13/17 (D) 15/17

PARAGRAPH -7
/ 2 / 2 2
sin (2 n  1) x  sin n x 
Let A n  
0
sin x
dx & Bn  
0

 sin x 
 dx .

18. If An + 1 = k . An , then the value of k is


n 1 n
(A) 1 (B) (C) (D) none
n n 1

19. Bn + 1 - Bn is equal to
(A) An (B) An - 1 (C) An + 1 (D) An + 1 – An

20. Bn - 1, Bn, Bn + 1 are in


(A) A.P. (B) G.P. (C) H.P. (D) none

PARAGRAPH-8
 ax 2  b, x  1
Let f :[ 2, 2]  R , given by the derivative of function g(x) =  2 , be every where con-
bx  ax  4, x  1
tinuous then answer the following questions
21. Value of (a, b) must be
(A) (2, 3) (B) (2, 3) (C) (2, 3) (D) none of these

22. For the above values of (a, b), minimum value of g(x) is
(A) 5 (B) 11/3 (C) 1 (D) none of these

23. Number of integral values of  for which the equation f(x) =  has a solution, for the above values of (a, b), is
(A) 22 (B) 24 (C) 23 (D) 15

PARAGRAPH - 9
Consider the quadratic equations x 2  2  a  1 x  a 2  1  0 and x 2  2ax  a 2  2a  2  0 where a  R.
Answer the following questions.
24. Set of values of real values of a for which both the quadratics will have real roots is
(A)  ,1 (B) 1,  (C)  1,1 (D) R

25. Number of real values of a for which the two equations will have exactly one common real root is
(A) 3 (B) 2 (C) 1 (D) 0

26. Set of real values of a for which both the roots of first equation are negative and those of second equation are
of opposite sign is


(A) 1  3, 3  1  
(B) 1  3,1  
(C) 1  3, 1  (D) none

COLLEGES: ANDHERI / BORIVALI / CHEMBUR / DADAR / KALYAN / KHARGHAR / NERUL / POWAI / THANE # 81
PASSAGE-10
Let  and  are the roots of the equation x3 + 6x + 3 = 0
 
and A = cos1 sin (  ) 1  (   ) 1  (   ) 1 
 1          
B = cos  tan  sin    
   2 
C = sec–1 cos ec 1   1  1    .

27. If the range of the quadratic trinomial g (x) = x2 – 2Bx + k is [0, ), then range of k equals
(A) [1, ) (B) (1, ) (C) {1} (D) (– , 1]

28. The value of (5A + B – C) is equal to


(A) 1 (B) 10 (C) 5 (D) 0

(5A  C) x 5  6Bx 2
29. Range of the function f ( x )  , is
x 4  (B  1) x 3  1
(A) [3, ) (B) [0, 3] (C) [– 3, 3] (D) (– , )

PARAGRAPH - 11
Let three circles be given as S1 : x 2  y 2  25, S 2 : x 2  y 2  2 x  15, S3 : x 2  y 2  8 x  15  0 From any point
P of S1 tangents of length a & b are drawn to S2 & S3. Answer the following questions.

30. If b  2 a , the coordinates P can be


(A) (4 , 3) (B) (0 , 5) (C) (5 , 0) (D) none

If 4a  b  40 , then area of the triangle formed by P and centres of S2 & S3 is –


2 2
31.
25 3 25 3 25 3
(A) (B) 25 3 (C) (D)
8 4 2

PARAGRAPH-12

 i4   1112i    12 5i 



A
Consider a triangle having vertices at the points  2 e ,
 B 2 e ,
 C 2 e  . Let the in-circle of  ABC
     
touches the sides BC , CA and AB at D, E and F respec tively. Which are represented by the complex

number Z d , Ze , Z f in order. Let P  z  be any point on the in circle.

32. AP 2  BP 2  CP 2 is equal to
(A) 12 (B) 15 (C) 16 (D) 27/2

 1 1 1 
33. Re    
 zd ze z f  is equal to
 
1 1
(A) 2 (B) (C)  (D) 0
2 2

COLLEGES: ANDHERI / BORIVALI / CHEMBUR / DADAR / KALYAN / KHARGHAR / NERUL / POWAI / THANE # 82
34. If the altitude through vertex ' A ' cuts the circum circle of  ABC at Q, then the complex number representing
' Q ' is

5  5 1 i 54
(A) 2e i
4 (B) 2e i
4 (C) e i
4 (D) e
2

PARAGRAPH -13

 
Consider the hyperbola passing through 4, 7 & having the bisectors of coordinate axes as it asymptotes.

Also let there be a circle with center (0, 8) and touching the line x  y  8  3 2 .

35. Common tangent of the two curves is


(A) x = 3 (B) 3x  4y  17 (C) 4x  3y  9  0 (D) y = 8

36. The point on the hyperbola closest to the circle is

(A) (3, 0) (B) (5, 4) (C)  10,1  (D) none of these

PARAGRAPH -14
d 2f  x   df  x  
2

Let 2  f  x    f x     0 & f  0   f 1  1 .


2
2
dx  dx 
37. Interval of values of x such that f(x) is strictly increasing, is
1   1
(A)  ,   (B)  ,  (C)  ,0  (D) 1,  
2   2

38. Number of points at which f(x) has local extremum is


(A) 0 (B) 1 (C) 2 (D) infinite

39. Area of region bounded by y = 0, x = 0, x = 1 & y = (2x – 1)f(x) is


 e1/ 4  1   e1/ 4  1   e1/ 4  1 
2
(A)  1/4  (B)  1/4  2
(C)  1/ 4  (D) 2  e  1
1/ 4

 e   e   e 1

PARAGRAPH-15
To find the point of contact P  x1 , y1  of a tangent to the graph of y = f (x) passing through origin O, we equate
the slope of tangent of y = f (x) at P to the slope of OP. Hence we solve the equation x1 f '  x1   f  x1  to get
x1 and y1. Now consider the equation |ln mx| = px.

40. The given equation has a single root for (m > 0)


(A) 0 < p < m/e (B) p > m/e (C) 0 < p < e/m (D) p > e/m

41. The given equation has exactly two roots for (m < 0)
(A) p = m/e (B) p = e/m (C) p  e/m (D) p  m/e

42. The given equation has exactly three roots for (m > 0)
(A) p < m/e (B) e/m < p < 0 (C) 0 < p < m/e (D) p < e/m

COLLEGES: ANDHERI / BORIVALI / CHEMBUR / DADAR / KALYAN / KHARGHAR / NERUL / POWAI / THANE # 83
PARAGRAPH - 16
Consider the quadratic equation ax 2   2a  1 x  a  3  0 and answer the following questions.
43. If both the roots are real and are of opposite sign, then complete set of values of a is
1
(A)  0,3 (B)  , 0    3,   (C)   (D) none
2
n2  n
44. If ‘a’ is given as , where ‘n’ is a natural number, then both the roots are necessarily
2
(A) integers (B) rational numbers (C) even integers (D) none

45. As per the data of previous question both the roots will lie in the interval
(A) [-1, 2] (B) [1,  ) (C) [1, 2] (D) none

PASSAGE-17
1
Let Pn(x) = (sinnx + cosnx)  n  N and
n
Q(m) = (cos 63°)m + (cos 57°)m + (cos 63°)m – 1 · (cos 57°)m – 1  m  N.
Also given log 2 = 0.3010; log 3 = 0.4771.
46. The value of log1. 3 Q(2) is equal to
(A) 0 (B) 2 (C) 1 (D) – 1


47. The value of 12P4 ( x )  P6 ( x )  at x = equals
10
(A) 1 (B) 3 (C) 6 (D) 1/12

100
8 
48. Number of zeroes after decimal before a significant figure start in  P2 ( x )  is
9 
(A) 30 (B) 31 (C) 35 (D) 36

PARAGRAPH -18

sin x
If 
0
x
dx  I , then answer the following questions.


sin ax cos bx
49. The value of 
0
x
dx is, if a > b > 0,

(A) 2I (B) 0 (C) I/2 (D) I


sin 2 x
 The value of  x 2 dx is,
0
(A) 2I (B) 0 (C) I/2 (D) I

COLLEGES: ANDHERI / BORIVALI / CHEMBUR / DADAR / KALYAN / KHARGHAR / NERUL / POWAI / THANE # 84
PARAGRAPH -19
Let a line L1 cuts the coordinate axes at A(7, 0) & B(0, 5) respectively. A variable line PQ is drawn perpendicular
to AB meeting the coordinate axes in P & Q. The lines AQ & BP intersect in R. T is the point on locus of R,
farthest from the origin. A parabola C1 with vertex at T and axis parallel to x-axis is drawn with latus rectum 2
units opening towards right of y-axis. Answer the following questions.
51. Coordinates of the point T are
(A) (7,5) (B) (7, 5) (C) (7, 5) (D) (7, 5)

52. Equation of the parabola C1 is


(A) y 2  10 y  2 x  39  0 (B) y 2  10 y  2 x  9  0
(C) y 2  10 y  2 x  9  0 (D) None of these

53. A light ray travelling along the line y = 3, gets reflected from the interior of the parabola. equation of the
reflected ray is
(A) 4 x  3 y  45 (B) 4 x  3 y  45 (C) 4 x  3 y  45 (D) none of these

PARAGRAPH -20
Let z1, z 2 , z 3 be the complex number associated with vertices A, B, C of a triangle ABC which is circum scribed
by the circle |z| = 1. Altitude through A meets the side BC at D and circumcircle at E. Let P be the image of E
about BC and F be the image of E about origin.

54. The complex number of point P is


z1  z 2  z3 2z1  z 2  z3 
(A) (B) (C) z1  z 2  z 3 (D) none of these
3 3

55. The complex number of point E is


z1z 2 z 2z3 z z
2 3 z1z2
(A) z (B) z (C)  z (D)  z
3 1 1 3

56. The distance of point C from F i.e. CF is equal to


z1  z3 z1  z 3
(A) z1  z 2 (B) z1  z2 (C) (D)
2 2

PARAGRAPH -21
If f be a twice differentiable function such that f ”(x) > 0 x  R . Let h (x) be defined by

h (x) = f (sin2 x) + f (cos2 x) where, | x | < .
2

57. The number of critical points of h(x) are


(A) 1 (B) 2 (C) 3 (D) more than 3

58. Interval of values of x for which h(x) is increasing, is


       
(A)   ,  (B)   ,     , 
 4 4  2 4 4 2
         
(C)   , 0    ,  (D)   ,     0, 
 4  4 2  2 4  4

COLLEGES: ANDHERI / BORIVALI / CHEMBUR / DADAR / KALYAN / KHARGHAR / NERUL / POWAI / THANE # 85
PARAGRAPH-22
The roots of the equation x3 – x2 + ax + b = 0 are real and are in A.P.

59. Maximum value of a is


1 1 2
(A) (B) (C) (D) none
3 2 3
60. Minimum value of b is
1 1
(A) 0 (B) (C)  (D) none
27 27

PASSAGE-23
Consider XYZ whose sides x, y and z opposite to angular points X, Y and Z are in geometric progression.

61. If r be the common ratio of G.P. then


5 1 5 1 5 2 52
(A) <r< (B) <r<
2 2 2 2
5 1 5 1 5 2 52
(C) <r< (D) <r<
3 3 3 3
sin Y
62. The integral values of is
sin X
(A) prime only (B) even (C) composite (D) odd

sin Z
63. The maximum value of is
sin Y
(A) irrational number (B) rational number but not integer
(C) integer (D) not defined

PARAGRAPH-24
b 1
For a given function f, suppose we approximate the definite integral  f ( x ) dx as ( b – a) (f(B) + f(A)) and for
a 2
b 1 1
more accuracy we define 
a
f(x) dx =
2
(c – a) ( f(A) + f(C)) + (b – c) (f (B) + f ((C)) for some c such that
2
1
a < c < b. When c = ( a + b),
2
b 1
a
f(x) dx =
4
(b – a) (f (A) + f (B) + 2f (C)).

2
64.  sin x dx equals
0

(A)

 
2 1 (B)

8
 
2 1 (C)
8

2
 
2 1 (D)

4 2
2
1
 f (x )dx – 2 t – a f t   f a  = 0
t

a
65 If f (x) is a polynomial and L = lim  a  R, then degree of f (x)
t a t – a 3
cannot exceed
(A) 2 (B) 4 (C) 3 (D) 1

COLLEGES: ANDHERI / BORIVALI / CHEMBUR / DADAR / KALYAN / KHARGHAR / NERUL / POWAI / THANE # 86
1 1
66. If f (x) < 0  x  (a, b) and F (t) = (t – a) ( f (A) + f(t)) + (b – t) (f(t) + f (D)) is maximum at some
2 2
point c  (a, b), then f (C) equals
1 f b  – f a 
(A) (f(D) + f(A)) (B) (C) 0 (D) f (A) + f (D)
2 b–a

PARAGRAPH -25
1
A line is drawn through the point P(–1,1) to meet the curve y  in the points A and B (Points A and B lie on
x
same side of P). A point R is chosen on this line such that PA, PR and PB are in A.P. Let locus of the point R
be the curve C = 0.
67. Focus of the curve C = 0 is
 1 1  1 1 1 1
(A)  2  ,  2   (B)   ,
2
 
 2 2  2 2 2

 1 1
(C)  2  , 2   (D) none of these
 2 2

68. Equation of tangent to C = 0 at  1,1 is


(A) x + 2y = 1 (B) x + y = 0 (C) x  y  2  0 (D) 2x  y  3  0

69. Area of the region bounded by the curve C = 0, x-axis & x = 1 is

3 e 1 3
(A) ln (B) ln (C) ln (D) none of these
e 3 2 e

PARAGRAPH -26
2 k 2 k
If  0 , 1 ,  2 , .....,  n 1 are root nth roots of unity, then  k  cos  i sin where 0  k  n  1 .
n n
Also

70. Value of (1  1 )....(1   n 1 ) is


(A) n (B) n  1 (C) (1)n (D) 0

n 1     2   n 1 
71. Value of 2 sin   sin   ....sin    is
n  n   n 
(A) 1 (B) n (C) n  1 (D) n / 3

PARAGRAPH 27
Let f :  2, 2    R be defined as f(x) = x + cos x – a.

72. f(x) is increasing in (exhaustive interval)


 3 
(A)  2, 0 (B)  0, 2 (C)  2, 2 (D)  , 2  
 2 
COLLEGES: ANDHERI / BORIVALI / CHEMBUR / DADAR / KALYAN / KHARGHAR / NERUL / POWAI / THANE # 87
73. The set of values of ‘a’ for which f(x) = 0 has only one positive root , is
(A) 1  2, 1 (B) 1,1  2  (C)  1,1  2  (D) 1  2,1

74. The set of values of ‘a’ for which f(x) = 0 has only one negative root , is
(A) 1  2, 1 (B) 1,1  2  (C)  1,1  2  (D) 1  2,1

PARAGRAPH-28
Consider a triangle formed by the points A(1, 1) , B(2, 1) & C(1, 2). Let P be a point inside the triangle such
1
that d  P, AB   d  P, BC   d  P, CA   , where d  P, AB  & other similar notations mean distances of
2
the point P from the respective lines each measured parallel to bisector of internal angle between the other two
lines.

75. The region containning all such points P , is


(A) a line segment (B) interior of part of a circle
(C) interior of a triangle (D) interior of a trapezium

76. Area of region containing all such points P is


(A) 2 (B) 1/2 (C)  / 6 (D) 2
77. Locus of P such that d  P, AB   d  P, CA is
(A) a line segment (B) arc of a circle (C) two fixed points (D) part of a parabola

PASSAGE 29
In ABC with internal angles A, B and C, points O, I and H respectively denotes circumcentre, incentre
and othercentre. Given that the points A, H, I, O and B are concyclic.
78. Angle C is equal to
   5
(A) (B) (C) (D)
6 4 3 12

79. The ratio HI : IO equals


1
(A) (B) 1 (C) 2 (D) 3
2

80. If AH = HI then the angles A, C and B are in arithmetic progression. The common difference of the A.P.
is
   
(A) (B) (C) (D)
18 12 9 10

PARAGRAPH 30
Let Vn denote the sum of first ‘n’ terms of an arithmetic progression whose first term is ‘n’ and the common
difference is (2n 1). Let Tr  Vr 1  Vr and Qr  Tr 1  Tr for r = 1, 2, 3, ..., n.
81. Value of the sum V1  V2  ....  Vn is

(A)
n
2
 
 n  1 3n 2  n  1 (B)
n
12

 n  1 3n 2  n  2 
n
(C)
2

2n 2  n  1  1
3
3
(D) 2n  2n  3  
COLLEGES: ANDHERI / BORIVALI / CHEMBUR / DADAR / KALYAN / KHARGHAR / NERUL / POWAI / THANE # 88
82. Which of the following is a correct statement?
(A) Q1 , Q2 , Q3 …. are in A.P. with common difference 5.
(B) Q1 , Q2 , Q3 …. are in A.P. with common difference 6.
(C) Q1 , Q2 , Q3 …. are in A.P. with common difference 11.
1.
(D) Q1 = Q2 = Q3 = ........

PARAGRAPH -31
A conic passing through the point A(1, 4) is such that the segment joining any point P on the conic and the point
of intersection of the normal at P with the x-axis is bisected by the y-axis . Answer the following questions .

83. Equation of this conic is


(A) 2 x 2  y 2  18 (B) x 2  2 y 2  33 (C) 2 x 2  y 2  14 (D) x 2  2 y 2  31

84. Equation of the circles touching the conic at A and passing through its focii are
(A) 3  x 2  y 2   4 x  20 y  30  0 & 3  x 2  y 2   7 x  2 y  69  0

(B) 3  x 2  y 2   4 x  20 y  30  0 & 3  x 2  y 2   7 x  2 y  69  0

(C) 3  x 2  y 2   4 x  20 y  25  0 & 3  x 2  y 2   7 x  2 y  50  0
(D) None of these

PARAGRAPH -32
d2 y 2x d y y 9/5
Consider the differential equation   0.
d x 2 1  x 2 d x 1  x 2 2

85. The substitution x  tan  , reduces the given D.E. to


d2 y d2 y 9/5 dy d2 y 1
(A) x0 (B) y 0 (C)  9/5  0 (D) none
d 2 d2 d d 2
y

86. The D.E. obtained after substitution in previous question may also be put in the form
3
 dy  d  1 dx  y
9/5

     , where f (x) is
 dx  dy  1  x dy  f  x 
2

(B) 1  x 2  (C) 1  x 2 
2 3
(A)1  x 2 (D) none

If solution of the given D.E. is f  x   a  ay 4/5  5 


5/ 4
87.  b , then f(x) is
1 2x
(A) tan 1 x (B) (C) (D) none
1 x2 1 x2

PARAGRAPH -33
x

Let f(x) be a polynomial with positive leading coefficient satisfying f(0) = 0 & f (f (x))  x  f (t) dt  x  R .
0

88. Two perpendicular tangents to the curve y = f(x) will intersect on the curve/line
3 3 2 3x
(A) x2 + y2 = 3 (B) y  (C) y   (D) y 
4 4 16

COLLEGES: ANDHERI / BORIVALI / CHEMBUR / DADAR / KALYAN / KHARGHAR / NERUL / POWAI / THANE # 89
89. If a line having slope 1 and passing through ( , 0) neither cuts nor touches the curve y = f(x) . then the value of
 must lie in the interval.
 3   3 3   3 
(A) (1, ) (B)  4 ,   (C)  ,   (D)   ,  
   4   4 
PASSAGE -34
An altitude BD and a bisector BE are drawn in a triangle ABC from the vertex B. It is known that the length of
side AC = 1, and the magnitude of the angles BEC, ABD, ABE, BAC from an arithmetic progression.

90. The area of circle circumscribing ABC is


  
(A) (B) (C) (D) 
8 4 2

91. Let 'O' be the circumcentre of ABC, the radius of the circle inscribed in BOC, is
1 1 1 1
(A) (B) (C) (D)
8 3 4 3 2 3 2

92. Let B' be the image of point B with respect to side AC of ABC, then the length BB' is equal to
3 2 1 3
(A) (B) (C) (D)
4 4 2 2

PARAGRAPH-35
b
Let f  n, k    x n  ln x  dx
k
a

f  n, k 
93. f  n, k  1 , a = 0 & b = 1 is equal to
n 1 n 1 n 1
(A)  (B)  (C)  (D) none
k 1 k 1 k 1

d
94. f  n, k  , a = 0, b = 1, is equal to
dn
(A) f  n, k  1 (B) f  n  1, k  (C) f  n, k  1 (D) none

81 2 81
If f  3, 2  
ln 3  ln 3  4  ln 2   ln 4  C , a = 2, b = 3, then C is equal to
4
95.
4 8
(A) 65/33 (B) 65/32 (C) 64/33 (D) 0

PARAGRAPH-36
x2
1  t  f '  t   dt  2x 2f (x) for 2  x   &
2
Let f be a non-negative real valued function satisfying 
2

3
f (4)  .
4
96. Range of f(x), is
 1 1 
(A)  0,   (B) 0,  (C)  ,   (D) none
 2 2 
COLLEGES: ANDHERI / BORIVALI / CHEMBUR / DADAR / KALYAN / KHARGHAR / NERUL / POWAI / THANE # 90
97. Range of values of a such that the equation f(x) = ax has a solution, must be
 1 1   1 1  1 1
(A)   ,  (B)   ,  (C)   ,  (D) none
 4 4 2  4 4  8 8

PARAGRAPH-37
 x  2 2  x  0  x  2 5  x  3
 
Consider f  x    2x  2 0  x  1 and g  x    x  1 3  x  1 .
2  2x 1  x  3 2x  2 1  x  1
 

98. Range of fog is


(A)  4, 2 (B)  4, 3 (C)  4,0 (D)  4, 3

99. If fog = gof , then the set of values of x is


(A)  1,3 (B) 2,0, 1 (C) {1} (D)  2, 2

PARAGRAPH-38
Let 4f  sin x   3f  cos x   x, 1  x  1 .
100. Range of f(x) is
 5 2   3 3   5 5 
(A)   ,  (B)   ,  (C)   ,  (D) none
 7 7   7 7   7 7 

101. Value of f  sin x   f  cos x  is


  3 2
(A) (B) (C) (D)
7 14 14 7
1  2 
102. f  x   is equal to
 7 
(A) sin x (B) cos x (C)- sin x (D) - cos x

PARAGRAPH-39
Let C1 be an ellipse with center at the origin and major axis along x-axis & C2 be a parabola with focus same as
right hand side focus of the ellipse and vertex at origin. Also let eccentricity of ellipse be 1/2 & equation of
common chord of the two curves be x = 1.

103. length of major axis of the ellipse is


(A) 8 (B) 6 (C) 10 (D) 11/2

104. Length of latus rectum of the parabola is


(A) 8 (B) 6 (C) 10 (D)11/2

105. Equation of common tangents of the two curves are


(A) 2y  3x  3 & 2y  3x  3 (B) 2y  x  6 & 2y  x  6
(C) 3y  2x  3 & 3y  2x  3 (D) 2y  x  3 & 2y  x  3

COLLEGES: ANDHERI / BORIVALI / CHEMBUR / DADAR / KALYAN / KHARGHAR / NERUL / POWAI / THANE # 91
PARAGRAPH-40
x2 y2
If a normal to the hyperbola   1 at the point P() passes through the point R(2a, 0) and meets the
a2 b2
Y – Axis in a point Q. Answer the following questions .

106. Range of eccentricity of the hyperbola is


(A)  2,  (B) 1,  
(C) 1, 2  (D) None

107. In the above case, the parameter of the point on the hyperbola where the normal passes through (0 , 2b) is
such that
(A)  1  tan<1 (B)  1  tan<0 (C)  < tan1 (D)  1  tan< 

108. Locus of the mid point PQ as  varies is a/an


(A) ellipse (B) hyperbola
(C) rectangular hyperbola (D) circle

PARAGRAPH-41
 a , 2b , 3c 
Each of the two triplets of numbers (a, b, c) &   are in G.P..
 2b 3c a 
109. If a triangle with lengths of sides as a, b, & c may be formed, then what type of triangle it must be?
(A) Acute angled triangle (B) Obtuse angled triangle
(C) Right angled triangle (D) Such a triangle is not possible

110. Ratio of inradius to circum radius of triangle mentioned in previous problem is


85 607 611
(A) (B) (C) (D) none of these
432 432 432

111. Let f  x   ax 3  3bx 2  3cx  1, f : R  R , then f (x)


(A) is always increasing (B) is always decreasing
(C) changes its monotonic behavior twice (D) changes its monotonic behavior once

PARAGRAPH-42
Two of the sides of a triangle are given as 4x  3y  1 & 3x  4y  7 and the centroid is (1, 3).

112. Orthocentre of the triangle is


(A) (1, 1) (B) (5, 3) (C) (1, 5) (D) none of these

113. Circumcentre of the triangle is


(A) (1, 2) (B) (1, 4) (C) (1, 1) (D) 1, )

COLLEGES: ANDHERI / BORIVALI / CHEMBUR / DADAR / KALYAN / KHARGHAR / NERUL / POWAI / THANE # 92
PARAGRAPH-43
Four points P, Q, R & S of the parabola y 2  8 x are concyclic and three of these points P, Q & R are such that

the normals at these are concurrent at a point T  4 , 4  . If S is the vertex of the parabola, answer the following
questions :
114. Centeroid of a triangle, whose vertices are the points P, Q & R, is
8  8  4 
(A)    1 , 0  (B)    1 ,0  (C)    1 , 0  (D) None
3  3  3 

115. Centre of the circle passing through P, Q, R & S will be ___


(A)  2  2 ,   (B)  2   ,2    (C)  2   ,2  (D)  4 , 2 

116. If radius of the circle passing through P, Q, R & S remains constant, then locus of T will be
(A) an ellipse (B) a straight line (C) a parabola (D) a circle

PARAGRAPH-44
Binomial expansion of 1  x  , when n is a negative integer or a proper fraction and x  1 , is given by
n

n  n  1 2 n  n  1 n  2  3
1  x 
n
 1  nx  x  x  ... till  terms , where the general term is
2! 3!
n  n  1 n  2  ...  n  r  1 r
x . Using the above expansions sum of various series may be obtained like the
r!
following example.
1 1
 
 1 2 1 1 1.3 1 1.3.5 1  1 2
1   1  .  . 2 . 3  ... till  terms   1    2.
 2 2 2 2! 2 3! 2  2

Particularly when n is a positive integer this expansion may be represented as 1  x  =  1
n r n  r 1
Cr x r .
r 0

Here when n = 1, note that the expansion becomes an infinite G.P.


1 1.3 1.3.5 1.3.5.7
117. Sum of the series 1     ... till  terms , is
3 2!32 3!33 4!34
3 1
(A) 3 (B) 2 (C) (D)
2 3

118. If  x 2  3x  2  is represented as sum of two infinite G.P.s with common ratio x & x/2, then range of values of
1

x is
1
(A) x  1 (B) x  (C) x  2 (D) all real numbers
2

119. Coefficient of x in the expansion of 1  x  x 2  ...  x10  , is


3

(A) 91 (B) 100 (C) 82 (D) 0

COLLEGES: ANDHERI / BORIVALI / CHEMBUR / DADAR / KALYAN / KHARGHAR / NERUL / POWAI / THANE # 93
PARAGRAPH-45
2k
In complex numbers n nth roots of unity are given by e n
i
, where ei  cos   i sin  and k takes values 0, 1,
2 4
2, ...,(n 1). e.g. the three cube roots of unity are 1, e 3 i & e 3 i . These n roots of unity form a G.P. as
2
i
1,  ,  ,..., 
2 n 1
, where   e n
&  n  1 . Properties of these roots include

n if r  multiple of n
1   r   2r  ...   
n 1 r
 . This property is used to find sum of coefficients selected
0 otherwise
at equal intervals in any expansion as follows. To find sum of coefficients at an interval of k, we put k roots of
unity it expansion one by one and add all the expansions to get the desired sum.

120. In the expansion 1  x  x 2   a0  a1 x  a2 x 2  ...  a2 n x 2 n , a0  a3  a6  ... is


n

(A) 3n (B) 3n - 1 (C) 3n - 2 (D) 32n - 1

121. If 1  x  x 2  x 3   a0  a1 x  a2 x 2  ...  a3n x 3n , then a0  a3  a6  ...  a3n is


n

4n  2 4n1  2 4n 4n  2
(A) (B) (C) (D)
3 3 3 3

122. If 1  x  x 2  x 3   a0  a1 x  a2 x 2  ...  a3n x 3n , then a0  a4  a8  ... is


n

(A) 4n (B) 4n - 1 (C) 4n - 2 (D) 43n - 1

PARAGRAPH-46

 
7
Let R  3 11  9 and also let [R] denote the greatest integer less than or equal to R & {R} = R[R].
2 2 2
 
123. R  R R is equal to
(A) 1814 (B) 914 (C) 187 (D) 637

124.  R  is
2

(A) an odd integer (B) div. by 18 (C) div. by 36 (D) none of these

COLLEGES: ANDHERI / BORIVALI / CHEMBUR / DADAR / KALYAN / KHARGHAR / NERUL / POWAI / THANE # 94
ANSW ER KEY
1. (B) 2. (A) 3. (C) 4. (A) 5. (C) 6. (B)

7. (C) 8. (A) 9. (C) 10. (A) 11. (D) 12. (B)

13. (A) 14. (B) 15. (B) 16. (C) 17. (C) 18. (A)

19. (C) 20. (A) 21. (A) 22. (B) 23. (C) 24. (A)

25. (C) 26. (C) 27. (C) 28. (A) 29. (B) 30. (B)

31. (C) 32. (B) 33. (D) 34. (A) 35. (A) 36. (B)

37. (B) 38. (B) 39. (A) 40. (B) 41. (A) 42. (C)

43. (A) 44. (B) 45. (A) 46. (D) 47. (A) 48. (C)

49. (D) 50. (D) 51. (B) 52. (A) 53. (C) 54. (C)

55. (C) 56. (A) 57. (C) 58. (A) 59. (A) 60. (C)

61. (A) 62. (D) 63. (D) 64. (B) 65. (D) 66. (B)

67. (A) 68. (C) 69. (B) 70. (A) 71. (B) 72. (C)

73. (B) 74. (A) 75. (C) 76. (B) 77. (A) 78. (C)

79. (B) 80. (C) 81. (B) 82. (B) 83. (A) 84. (B)

85. (C) 86. (C) 87. (B) 88. (C) 89. (B) 90. (B)

91. (B) 92. (D) 93. (A) 94. (C) 95. (B) 96. (B)

97. (C) 98. (A) 99. (C) 100. (A) 101. (B) 102. (B)

103. (B) 104. (B) 105. (B) 106. (C) 107. (B) 108. (B)

109. (B) 110. (D) 111. (A) 112. (A) 113. (B) 114. (B)

115. (A) 116. (A) 117. (A) 118. (A) 119. (C) 120. (B)

121. (A) 122. (B) 123. (A) 124. (A)

COLLEGES: ANDHERI / BORIVALI / CHEMBUR / DADAR / KALYAN / KHARGHAR / NERUL / POWAI / THANE # 95
MATRIX MATCH TYPE QUESTIONS
1. Column I Column II

2 0 7  –x 14x 7x 
  0
(A) A =  0 1 0  is inverse of B =
 1 0  , (p) 3
1 –2 1 
 x –4x –2x 
then the value of x is?
 2  1
(B) If A =   and |A3| = 125 then  is equal to (q)
 2  5

(C) If a, b, c are three complex numbers such that (r) 1


a2 + b 2 + c2 = 0 and
b 2  c2 ab ac
2 2
= ab c a bc = ka2b2c2
ac bc a 2  b2

cos 2  cos  sin  – sin 


(D) If f() = cos  sin  sin 2  cos  then f  2  is (s) 4
 
sin  – cos  0  3 

2. Column I Column II
5 1
(A) If 2sin x, sin 2x & 2cos x are in A.P., then the (P)
2
value of |sin x + cos x| is
3
(B) If the angles of a triangle are in A.P. with the common (Q)
2
1
difference equal to of the greatest angle then, ratio of
3
the greater two sides is
(C) Let the sides AB, BC, CD & DA of a cyclic quadrilateral (R) 2 5
BD
ABCD are in G.P. with common ratio 2, then the value of is
AB
(D) Let a, b, c be in A.P. & a2, b2, c2 be in H.P. If a  c , then a : c is (S) 32

3. Match the entries in column-I representing equations of various families of curves with their orthogonal trajectories
given in Column-II.
Column-I Column-II
(A) 2x  y  a
2 2 2
(p) xy  C
(B) x  y  a
2 2 2
(q) x  y  cxy
(C) 2x 2  y 2  a 2 (r) y 2  cx
(D) x 3  y3  a 3 (s) xy 2  C

COLLEGES: ANDHERI / BORIVALI / CHEMBUR / DADAR / KALYAN / KHARGHAR / NERUL /


4. Let ‘k’ be a non zero positive rational number number.
Column I Column II
(A) x 2  2kx  k 2  1  0 (P) Roots are rational
(B) x 2   k  1 x  k  0 (Q) Roots are negative
(C) kx 2  3x  k  0 (R) Roots are imaginary
(D) kx 2  2  k  1 x  k  2  0 (S) Roots are of opposite sign

5. Column -I Column - II
dx 1  tan 2 x 
(A)  sin  (p) tan 1  
4
x  cos 4 x 2  2 
sin 2 x 3  8 cos x
(B)  (3  4 cos x) dx 
3 (q)
16(3  4 cos x ) 2
1  sin x 2x x
x
(C)  1  cos x e dx  e 2
P( x) (r) sec
2
then P (x ) is
dx 1 4x  3
(D)   2 x  3x  4
2

2
sin 1 f ( x) (s)
41
then f (x) is

6. COLUMN – I COLUMN – II
(A) The function y defined by the equation xy – ln y = 1 (p) 24
satisfy x(yy + y2) + kyy = 0. the value of k is
(B) If the function y(x) represented by x = sint, y = ae
t/ 2
 be t / 2 , (q) 1/2
  
t    ,  satisfies the equation (1 – x2)y – xy = ky,,
 2 2
then k is equal to

(C) Let F(x) = f(x)g(x)h(x) for all real x, where f(x), g(x) and h(x) are (r) 1
differentiable functions. At some point x0, if F(x0) = 21F(x0),
f(x0) = 4f(x0), g(x0) = –7g(x0) and h(x0) = kh(x0) then
k is equal to

(D) Let f(x) = xn, n being an integer greater than 2. The least integral (s) 3
value of b/a for which the inequality f '( a  b)  f '( a )  f '(b)
is valid for all b, a > 0 is

COLLEGES: ANDHERI / BORIVALI / CHEMBUR / DADAR / KALYAN / KHARGHAR / NERUL /


7. Column I Column II


2
(A)  x  sin  sin x   cos  cos x   dx
2 2
(P)
0 16
2
 /4

  2sin 
x  x cos x dx 2
(B) (Q)
0 2
 /4

  
ln 1  sin 2 x dx 2
(C) (R)
ln 2  / 4 4

8 x3 cos 4 x sin 2 x 2
(D) 0  2  3 x  3x 2 dx (S)
32

8. Let ABC be a isosceles triangle with AB = AC. Also let eq. of AB be x  y  10 and that of AC be 7 x  y  30
and area of triangle be 20 sq. units.
Column -I Column -II
(A) Coordinate of point B can not be (p) (10, 0)
(B) Coordinate of point C can not be (q) (4, -2)
(C) Centroid of  ABC can not be (r) (-5/2, 5/2)
(D) Circum center of  ABC cannot be (s) (8/3, 13/3)

9. Consider the Quadratic equation ax 2  2  a  1 x  a  2  0, a  R .


Column I Column II
 1
(A) Both the roots are real & distinct (P)  , 
 4
(B) Both the roots are greater than 2 (Q) no value of a
 1
(C) Both the roots are less than 2 (R)  , 6    0, 
 4
(D) 2 lies between the roots (S) ( 6,  1)

10. COLUMN – I COLUMN – II

(A) The shortest distance from the point M (-7, 2) to the (p) 2
circle x² + y² - 10x -14y - 151 = 0 is
(B) The distance from the centre of the circle x² + y² = 2x (q) 4
to the straight line passing through the points of
intersection of the two circle x² + y² + 5x - 8y + 1 = 0 &
x² + y² - 3x + 7y - 25 = 0 is
(C) If from any point P on the circle x² + y² + 2gx + 2fy + c = 0, (r) 1
tangents are drawn to the circle
x² + y² + 2gx + 2fy + c sin² 20 + (g² + f²) cos² 20 = 0,
then the angle between the tangents in degree is
(D) The number of common tangent(s) to the circles (s) 3
x² + y² + 2x + 8y - 23 = 0 & x² + y² - 4x - 10y + 19 = 0 is

COLLEGES: ANDHERI / BORIVALI / CHEMBUR / DADAR / KALYAN / KHARGHAR / NERUL /


11. Column - I Column - II

(A)  4
ln  sin 2 2x  dx (p) 2
0

 
  x sin 2x  1 e (q) 
cos2 x
(B) 2
dx
0 2
    5   3  
(C) If f  x    f  x    f  x   & (r) 2
ln  cos x  dx
 24   24   24  0

at
 f  x  dx is independent of a, then t =
a

5 x 4  4 x5 
(D) If f  x    dx & f  0   1 (s)
x  x  1 2
5 2

then f   1 

12. COLUMN – I COLUMN – II


(A) TP & TQ are tangents to the parabola y2 = 4x & the (P) 1
normals at P & Q meet at a point R on the curve. Centre
of the circle circumscribing the triangle TPQ lies on the
parabola 2 y² = a(x - a), where a is
(B) Two parabolas have the same focus. If their directrices (Q) 4
are the x - axis & the y - axis respectively, then the slope of
their common chord may be
(C) If M is the foot of the perpendicular from a point P of a (R) 3
parabola y2 = -4x to its directrix and SPM is an equilateral
triangle, where S is the focus, then SP is equal to
(D) The normal chord at a point P(t) on the parabola y2 = 4ax (S) 2
subtends a right angle at the vertex . Then t2 is equal to

13. Column I Column II


2 2
(a) If a variable line y = kx + 2h is tangent to an ellipse (p)
3
2x2 + 3y2 = 6, then locus of P(h, k) is a conic C whose
eccentricity is
x 2 y2 7
(b) The value of a for the ellipse  = 1 (a > b), (q)
a 2 b2 3
if the extremities of the latus-rectum of the ellipse having
positive ordinate lies on the parabola x2 = – 2 (y – 2), is
(c) An ellipse is inscribed in a circle and a point within the (r) 2
circle is chosen at random. If the probability that this
point lies outside the ellipse is 2/3 then the eccentricity
of the ellipse is
(d) The radius of the circle passing through the foci (s) 4
of the ellipse, 9x2 + 16y2 = 144 and having its centre
at (0 , 3) is

COLLEGES: ANDHERI / BORIVALI / CHEMBUR / DADAR / KALYAN / KHARGHAR / NERUL /


14. Column-I Column-II
(A) Number of five digit integers, with sum of the digits equal to 43 are (p) 12
12 !
(B) The sum of all numbers of the form a !b !c ! where (q) 15

a, b, c  N satisfy a  b  c  12 is 3k, where k is

(C) The number of ways to select 6 people out of 6 married couples (r) 7
in which exactly one couple is present, is n, then (n/30) =
(D) Nine hundred distinct n- digit numbers are to be formed using only (s) 16
the digits 2, 5 and 7. The smallest value of n for which this is
possible is

15. Match the curve on which the roots of the equation in column-I lie  n  N 
Column I Column II
(A) zn = (1 – z)n (p) x = 0
(B) (1 – z)n = (1 + z)n (q) x = 1/2
(C) z = 2 + 21i
n
(r) circle with centre at the origin
(D) (z – i)n = 21  2i (s) circle with centre at z = i

16. Column-I Column-II


x 1 y   z 1
(A) If the line   entirely lies in the (P)   4
 –6 2
plane 2x  y  6z  0 , then
(B) If planes 2x  y  z  2 , x  3y  z   & x  y  z  5 (Q)   1
have no common point, then
(C) If a, b, c are +ve real numbers and ax  by  z  0 , (R)   3
x  by  cz  0 & ax  y  cz  0 have infinite common
points, abc   , then
(D) If     0 and the planes x  y  z  0 , x  y  z  0 (S)   3
&  x   y  z  0 have one common point, then
2 2

17. Column - I Column- II


d(F(x)) esin x
(A) Let  , x 0. (p) 16
dx x
4
3
xe
sin x 3
If dx = F (K) – F (1) then one
1
of the possible values of K is
x2

 sec
2
t
(B) The value of Lt 0
dt  (q) 64
x 0 x sin x
(C) If f ( x)  e x , g ( x)  x , x  0 ; and
t
F  t    f  t  x  g  x  dx then, F'  log 2  = (r) 03
0
(D) The degree of the differential equation (s) 01
which represent the Family curves
y 2  2c ( x  c )( c  0)
COLLEGES: ANDHERI / BORIVALI / CHEMBUR / DADAR / KALYAN / KHARGHAR / NERUL /
18. COLUMN – I COLUMN – II
lim [(x5 + 7x4 + 2) x] is finite & non zero.
(A) x (p) 2
Value of the limit is
cos ec x
 3 
(B) lim    ek , then k = (q) 1/12
x 0  1  4  x 
 
(C) The greatest value of f(x) = (x + 1)1/3 (x1)1/3 (r) 7/5
on [0, 1] is
(D) minimum value of the function f : R  R , (s) none
f(x) = x(x + 1) (x + 2) (x + 3) is

19. Column I Column II


(A) If the quadratic equations 3x + ax + 1 = 0 & 2x + bx + 1 = 0
2 2 (P) 10
have a common root, then the value of the expression
5ab  2a2  3b2 is
(B) If the roots of the quadratic equation x2 + 6x + b = 0 are real (Q) 5
and distinct and differ by atmost 4 then the least value of b is
(C) If x R, integral value of the expression 7 + 5x 2x2 (R) 1
cannot exceed
(D) If the inequality (m 2)x2 + 8x + m + 4 > 0 is true for all x R, (S) none
then the least integral m is

20. COLUMN – I COLUMN – II


n
 1
(A) The sum of the binomial coefficients of 2 x   is equal (p) 10
 x 
to 256. The constant term in the expansion is
10
x 3
(B) The coefficient of x in 256.   2  is
4
(q) 1120
2 x 
(C) If the 6th term in the expansion of the binomial (r) 5050
8
 1 2 
 x 8 / 3  x log10 x is 5600, then x is
 
(D) (1 + x) (1 + x + x2) (1 + x + x2 + x3) ... (s) 405
... (1 + x + x2 + ...... + x100), when written in the ascending
power of x then the highest power of x is

COLLEGES: ANDHERI / BORIVALI / CHEMBUR / DADAR / KALYAN / KHARGHAR / NERUL /


21. A showroom has 10 different pairs of expensive shoes. A thief steals 4 shoes randomly from the showroom.
Match the following:
Column - I Column - II
3
(A) The probability that there is no matching pair among (P)
19 17
the 4 shoes stolen is
96
(B) The probability that there is atleast one matching pair among (Q)
19 17
the 4 shoes stolen is
20
C4  10C4 .24
(C) The probability that there is exactly one matching pair among (R) 20
C4
the 4 shoes stolen is
14  16
(D) The probability that there are two matching pairs among (S)
19 17
the 4 shoes stolen is (T) none of the above

22. Column I Column II


x2 x2
(A) A tangent to  y 2
 1 cuts  y 2  1 in points P and Q. (P) 1
4 4

Locus of the midpoint of PQ is  x 2  4y 2   k  x 2  4y 2  ,


2

then k =
(B) A variable straight line of slope 4 intersects the hyperbola (Q) 4
xy = 1 at two points. Locus of the point which divides the line
segment between points in the ratio 1 : 2 is 16x2 + y2 + 10
xy = k, then k =
(C) Locus of a point whose chord of contact touches the circle drawn (R) 2
on the line joining the foci of the hyperbola x 2  y 2  2 as
diameter is x 2  y 2  k 2 , then k =
(D) The locus of a point, from where tangents to the hyperbola (S) None
x2 – y2 = a2 contain an angle of 45° is

x  y 2   4a 2  x 2  y 2   ka 4 , then k =
2 2

COLLEGES: ANDHERI / BORIVALI / CHEMBUR / DADAR / KALYAN / KHARGHAR / NERUL /


23. Column - I Column - II
x

(A) If lim
 f  t  dt  f  x   f  a    , then
a
(p)

x  a 2
3
x a 3

f  a   f '"  a  

cos 2 x  t  1 
(B)    dt , 0  x  = (q) - 1/2
 t  1 t  t 1  t  4
sin 2 x

1
 n 1 2 2  n2
 
r
 n r 
(C) If lim  r 1 n 2  n   e k , then k = (r) 2
n   
n
 
 



(D) If  f  sin x  sin x dx  k  2 f  cos x  cos x dx , (s) None of the above
0 0

then k =

24. Column I Column II


(A) Let coordinates of A be (1, 2) & those of B be (p) 2x + 3y + 4 = 0
(2, 3). If perpendicular bisector of BC is x + 2y = –2,
then the perpendicular bisector of AC is
(B) Let Mid points of AB & AC be (1, 1) & (3, 4). If A (q) 3x + 7y + 12=0
is (3, -3), then altitude on BC is
(C) Equations of sides of a parallelogram are (r) 3x + y = 3
3x  7xy  2y  12x  18y  12  0 &
2 2

3x 2  7xy  2y2  5x  2  0 . One of the digonals is


(D) Coordinates of A & B are (0, 3) & (, 0) resp- z(s) 2x + 3y + 3 = 0
ectively. If bisector of internal angle ABC is
x 3y = 1, then equation of BC is

25. COLUMN – I COLUMN – II


(A) Given two circles x² + y² = 6 & x² + y² - 6x + 8 = 0. (p) x² + y²-2x-4y + 3 = 0
The equation of the circle through their points
of intersection and passing through (1 , 1) is
(B) The locus of the mid point of a chord of the circle (q) x² + y² = 2
x² + y² = 4 which subtends a right angle at the origin is
(C) The equation of the circle having the pair of lines (r) x² + y² - 3x + 1 = 0
y² - 2y + 4x - 2xy = 0 as its diameters & passing through
the point (2 , 1) is
(D) The locus of middle points of the chords of the circle, (s) x2 + y2 = 3
x2 + y2 = 4, of length 2 unit is

COLLEGES: ANDHERI / BORIVALI / CHEMBUR / DADAR / KALYAN / KHARGHAR / NERUL /


26. Consider the quadratic equation, ax 2  2bx  c  0, where a, b, c are positive & distinct real number..
Column I Column II

(A) a, b, c are in A.P. (p) roots are necessarily real & distinct

(B) a, b, c are in G.P. (q) roots are necessarily real & equal

(C) a, b, c are in G.P. (r) roots are necessarily imaginary

(s) none of these

27. Column - I Column - II

x
1  sin
(A) If I   2 dx  a 2 tan 1  sin  x      b , (p) 1
x   
3  sin   2a  
2
then a =

 ln  sin x  dx  f  f  g  x    ,
cot x 1
(B) If (q)
2

f 1  x 
then lim 
x 0 g x

dx
(C)  x 2 1  x 2 
3/ 2

(r) 4
1/ 3

cos3 x 1 1
(D) f x   dx and lim f  x   , (s)
7
sin x x
 5 5
2


then f   
4

28. Column-I Column-II


x 1 y  1 z  1 1 6
(A) If the line   lies in the plane (P) sin
1 2  25
3x – 2y + 5z = 0, then  is equal to
(B) If (3, ,  ) is a point on the 2x + y + z – 3 = 0 = x – 2y + z – 1 (Q) – 7/5
then    is equal to
(C) the angle between the line x = y = z and (R) –3
the plane 4x – 3y + 5z = 2 is
1 8
(D) The angle between the plane x + y + z = 0 and 3x – 4y + 5z = 0 is (S) cos
75

COLLEGES: ANDHERI / BORIVALI / CHEMBUR / DADAR / KALYAN / KHARGHAR / NERUL /


29. Column I Column II
(a) If a chord of x  4y  1 is touching y  4 x ,
2 2 2
(p) x 2  4  2y
then locus of its mid point is
(b) If length of major axis (along x-axis)of a standard (q) x 3  4xy 2  16y 2  0
ellipse is constant equal to 2, then the end point of its
latus rectum in first quadrant lies on the curve
(c) If chord of contact of tangents drawn from P to (r) 5x 2  8xy  5y 2  4
x 2  xy  y 2  1 is tangent to x 2  y 2  1 , then locus
of P is

30. COLUMN – I COLUMN – II


(A) The locus of the middle points of the focal chords of the (P) y2 = 4 (x + 4)
parabola, y2 = 4x is
(B) Let the tangent to the parabola, y2 = 4x meets the axis in (Q) y2 = 2 (x – 1)
T & the tangent at the vertex A in Y . If the rectangle TAYG is
completed, then the locus of G is
(C) Locus of the middle points of all chords of the parabola, (R) y2 = - x
y2 = 4x passing through the vertex is
(D) The equation of the parabola whose Axis is x-axis, the (S) y2 = 2x
directrix is x + 5 = 0 & latus rectum is of length 4, is

31. There are 2 Indian couples, 2 American couples and one unmarried person
Column -I Column II
(A) The total number of ways in which they can sit in a row (p) 5760
such that an Indian wife and an American wife are always
on either side of the unmarried person, is
(B) The total number of ways in which they can sit in a row (q) 24320
such that the unmarried person always occupy the middle
position, is
(C) The total number of ways in which they can sit around a (r) 40320
circular table such that an Indian wife and an American
wife are always on either side of the unmarried person, is
(D) If all the nine persons are to be interviewed one by one (s) 22680
then the total number of ways of arranging their interviews
such that no wife gives interview before her husband, is

32. Column - I Column - II

1 1 x2 1  2 x
(A)   x  1 x2 1
dx  (p)
2
ln
x2  1  2 x
C

1 1 3
  3x x  2 x  3  C
3/ 2
(B) dx  (q)
2
 1 x  1 2 9

(x  1) dx x 1  x 1
(C)  = (r)
x 1
x2 2x2  2x 1

2 x2  2 x  1
(D)   x  1 2 x 5  3x 4  x 2 dx  (s)
x
+C

COLLEGES: ANDHERI / BORIVALI / CHEMBUR / DADAR / KALYAN / KHARGHAR / NERUL /


33. COLUMN I COLUMN II
1
1  a 3   8e x
2
(A) If a, b  Z & lim 1 (p) 3
1 2  b  b e
x 0 2 x

then the value of a 2  b 2 is


n
1  e3x e9x e27x ...e3 x

(B) lim x /3n


 3 10 , (q) 2
x 01  e e e ...e
x /3 x/9 x/27

then n  Z is
(C) The number of integral values in the range of (r) 11
f  x   1  sin x    cos x  1   tan x  , for
1

x   0, 2 , Where . denotes the G.I.F. is

(D) If f (x)  (2x  )3  2x  cos x, and


d 1
dx
f (x)   (s) 4

at x   is

34. Column I Column II


(A) If z = i log (2 – – 3 ), then cos z = (p) -1
(B) If z3 + (3 + 2i) z + (–1 + ia) = 0 has one real root, then the (q) 2
least integral value of a =
(C) If for z as real or complex, (1 + z2 + z4)8 = (r) 1
C0 + C1 z2 + C2z4 + ... + C16 z32, then
C0 – C1 + C2 – C3 + ... + C16 =
(D) If |z1| = 1, |z2| = 2, |z3| = 3 and (s) none
| 9z1z2 + 4z1z3 + z2z3 | = 12, then the value of z1  z 2  z 3 

35. Column I Column II


(A) Vertices of a triangle are (1, 2), (3, 4) & (2, 4) (p)   7
(B) Equations of sides of a triangle are x + y = 3, (q) triangle is obtuse angled
3x 5y + 7 = 0 & x 3y + 1 = 0
(C) Orthocenter & circumcenter of a triangle are (r) (1, 1) lies inside the triangle
5 
(1, 0) &  ,1 respectively
2 
(D) Equations of sides of a triangle are 3x – 4y – 13 = 0, (s) centroid lies on 3y = 2
8x – 11 y – 33 = 0 & 2x – 3y +  = 0

COLLEGES: ANDHERI / BORIVALI / CHEMBUR / DADAR / KALYAN / KHARGHAR / NERUL /


36. COLUMN – I COLUMN – II
(A) The co-ordinate of the point on the circle (p) (1, 2)
x² + y² - 12x - 4y + 30 = 0 , which is farthest from
the origin are
(B) If one end of a diameter of the circle (q) (9, 3)
x² + y² - 4x - 6y + 11 = 0 is (3, 4) then the co-ordinates
of the other end are
(C) The centre of the smallest circle touching the circles (r) (5, 1)
x² + y² - 2y - 3 = 0 & x² + y² - 8x -18y + 93 = 0 is
(D) A circle passes through the points (-1, 1), (0, 6) and (s) (2, 5)
(5, 5). The points on this circle, the tangent(s) at which
is parallel to the straight line joining the origin to
its centre is/are

37. Consider the equation x2 + (2a + 3)x + a2  4 = 0


Cloumn I Column II
(A) Roots are real and distinct (P) a  2
25
(B) both the roots are positive (Q) a  
12
(C) Both the roots are negative (R) 2  a  2
25
(D) One root is positive & other is negative (S)   a  2
12

38. COLUMN I COLUMN II


(A) A real valued function f(x) satisfies (p) 3
 x  y  2f (x)  2f (y)  4
f  for all
 3  6
real x & y. If f '(0)  2 , then f(2) =
(B) The least values of , (q) 1

2    2  
4 4
4  x2 4  x2
f x 
2
(C) Let f(x) be a differentiable non zero function (r) 1/4
1
satisfying f ' 1  & 4f  xy   f  x  f  y  .
4

The value of
dx

d 1

f  x  at x = 4 is

 n 1, m0

f  m,n   f  m 1,1 , m  0,n  0,m,n  Z
(D) (s) 2
f  m 1,f  m,n 1  , m  0,n  0

f(1, 1) is equal to

COLLEGES: ANDHERI / BORIVALI / CHEMBUR / DADAR / KALYAN / KHARGHAR / NERUL /


39. Column I Column II
(A) Focus of a hyperbola is (1, 2) & directrix other than its
corresponding directrix is 6x + 8y + 3 = 0. If T.A. = 2, (P) 2 3
then e =
(B) Asymptotes of a hyperbola are x = 1, y = 2. It passes through (Q) 2 2
(2, 3), then T.A. =
(C) Area of triangle formed by a tangent & asymptotes of the (R) 2
x 2 y2
hyperbola   1 , is
3 4
(D) Length of L.R. of x 2  2y 2  2x  8y  9  0 , is (S) 2

40. Column I Column II

(A) P  A C   P  B C   P   A  B  C   
(P) P A B  C 
  
(B) P A  B | C  P   A  B  | C   (Q) P  A C 

1  P (A)  P (B  C)  P  A   B  C  
(C)  (R) P   A  B  C 
1  P (B  C)

 
(D) P A  B  P  A  B   (S) 1
(T) none of the above

41. COLUMN – I COLUMN – II


(A) The number of distinct inegral terms in the (p) 5

 
64
expansion of 3  51/8 is
(B) The number of values of r satisfying (q) 2
69
C3r 1  Cr2  Cr 2 1  C3r , is
69 69 69

(C) If (r + 1) term in the expansion of (2 + 3x)25 is (r) 9


the greatest, where x = 1/6, then r is
(D) Remainder, when 262 is divided by 7, is (s) 4

COLLEGES: ANDHERI / BORIVALI / CHEMBUR / DADAR / KALYAN / KHARGHAR / NERUL /


42. Column I Column II
(A) If the function y = e4x + 2e–x is a solution (p) 3
d3y dy
3
– 13
of the differential dx dx = K
y
then the value of K/3 is
(B) Number of straight lines which satisfy the
2
dy  dy 
differential equation  x   – y = 0 is (q) 4
dx  dx 
(C) If real value of m for which the substitution, (r) 2
y = um will transform the diffrential equation,
dy 4
2x4y +y = 4x6 into a homogeneous
dx
equation then the value of 2m is
(D) If the solution of differential equation (s) 7
d2y dy
x2 2
 2x = 12y is y = Axm + Bx–n
dx dx
the |m + n| is
(t) 5
   
43. Let A(a), B(b) & C(c) be three given points, where the three position vectors are linearly independent & P(r )
be a moving point in a three dimentional coordinate system.
Column-I Column-II
   
(A) Locus of P if (r  a).(r  b)  0 (p) A Sphere
     
(B) Locus of P if c.(r  b)  a.(b  c) (q) A Straight Line
     
(C) Locus of P if 2 r  a  a  b  c  a (r) A Circle
  2 2   
(D) Locus of P if (r  a)  b & (r  a).b  0 (s) A Plane

44. Column I Column II


x 2 y2
(a) Equation of a tangent to   1 from (6, 0) is (p) 9x  8y  43
18 8
x 2 y2
(b) equation of chord of   1 , whose mid (q) x  y  4
16 12
point is (3, 2), is
(c) equation of chord of contact of the tangents drawn (r) 2x  3y  12
1  y2
from  ,1  to the ellipse x 2   1 , is
6  4
x 2 y2
(d) Equation of a common tangent of  1 (s) none of the above
6 10
& x 2  y 2  8 , is

COLLEGES: ANDHERI / BORIVALI / CHEMBUR / DADAR / KALYAN / KHARGHAR / NERUL /


45. COLUMN – I COLUMN – II
(A) The x-coordinate of points on the axis of the parabola (P) 3
y2 – 4x – 2y + 5 = 0 from which all the three normals
to the parabola are real is
(B) The shortest distance between the parabola y2 = 4x (Q) 5
and y2 = 2x – 6 is
(C) Two perpendicular tangents PA and PB are drawn (R) 6
to x2 = -8y, minimum length of AB is equal to
(D) The number of points with integral coordinates that lie in (S) 8
the interior of the region common to the circle x 2  y 2  16
and the parabola y 2  4 x is

46. Column I Column II


 xe x ,
2
x0
(A) f x   is (p) [0, 1/3]
a x  2ax  x , x0
2 2 3

 1
increasing in  1,  , then set of values of a
 3
 p5  3
(B) f (x)    1  x  3x  log 5 is (q) (1, 4]
 1 p 
 
decreasing for all x, then values of p
(C) f (x)  sin 2 x    2  2  1 sin x , where (r) 1, 3)
 
 x . If f (x) has exactly one local
2 2
exremum, then the set of values of 
(D) values of ‘a’ for which one negative and two (s)  1, 0   1, 2 
positive real roots of the equation
x3 – 3x + a2 a = 0 are possible

COLLEGES: ANDHERI / BORIVALI / CHEMBUR / DADAR / KALYAN / KHARGHAR / NERUL /


47. Column I Column II

 x2
 log e x  ,1  x  3
(A) For   
f x  x  2 (p) 0
x 2  , 7
3 x 
 2
number of points of discontinuity is
 1
 1 
 2  sin  e x if x0
(B) For f  x    x (q) 5

 0 if x0
number of points of extremum is

(C) 
number of points at f  x   max 3  x ,3  x
2 3
 (r) 2
is not differentiable
  1 
(D) 2 lim x 2 1  2  3  .......     =, where [x] (s) 1
 
 x 
x 0

denotes greatest integer less than or equal to x

48. COLUMN – I COLUMN – II


(A) If the straight lines y = a1x + b and y = a2x + b, (a1¹ a2) (p) a12  a 2 2  4
and b Î R meet the coordinate axes in concyclic points, then
(B) If the chord of contact of the tangents drawn to x2 + y2 = 4 (q) a1 + a2 = 20
2
and b Î R from any point on x + y = a , touches the circle
2 2
1

x2 + y2 = a22 , (a1¹ a2) , then


(C) If the circles x2 + y2 + 2a1x + 2a2y - 1 = 0 and (r) a1a2 = 4
x2 + y2 - 2a1x - 2a2x - 1 = 0, (a1¹ a2) cut orthogonally, then
(D) If the shortest and largest distance from the point (s) a1a2 = 1
(8, 5) to the circle x2 + y2 – 4x + 6y – 12 = 0
are a1 and a2 respectively, then

49. Column I Column II


(A) The medians of a triangle ABC are 9, 12 and (p) 196/5
15 cm respectively. Area of the triangle is
(B) Two rays including an angle of 43º emanate (q) 72
from the point A. Lines L1, L2 and L3 (no two
of which are parallel) each form an isosceles
triangle with these rays. Largest angle of the
triangle formed by the lines in degree is
(C) The equations of the two sides of a rhombus are (r) 129
3x - 10 y + 37 = 0 & 9x + 2 y - 17 = 0 and equation
of one diagonals is 3x - 2y - 19 = 0. Equations one of
two other sides is 3x - 10 y = c, where c =
(D) The equation of the hypotenuse of a right isosceles (s) 59
triangle ABC is x - 2y - 3 = 0. If the vertex is C(1, 6)
then area is

COLLEGES: ANDHERI / BORIVALI / CHEMBUR / DADAR / KALYAN / KHARGHAR / NERUL /


50. Column - I Column - II

4  x2 , x0

(A) f  x    2  3x, 0  x  1 (p) f(x) is ONE - ONE & ONTO
 2x  4, x 1

(B) f : R  R, f  x   x 3  x 2  4x  sin x (q) f(x) has exactly one local extremum


1 1
(C) f '  x   3x 2 sin  x cos , x  0, (r) x = 0 is a point of local maximum
x x
f  0   0  f 1 /  

(D) f  x   3x 4  4 x 3  6 x 2  b , b R (s) f(x) is continuous at x = 0

51. Given A (3 + 4i), B(-4 + 3i) and C(4 + 3i) be the vertices of ABC which is inscrived in a circle S = 0. Let AD,
BE, CF are altitudes through A, B, C which meet the circle S = 0 at Dz1 , Ez 2  and Fz 3  respectively..
Column I Column II
(A) Complex number z1 is equal to (p) 3 - 4i
(B) Orthocentre of triangle ABC is at (q) 3 + 10i
z1z 2 z 3
(C) The value of is equal to (r) 3 + 4i
25

52. COLUMN – I COLUMN – II


(A) 10C0  4 10C1  7 10C2  . . .  3110C10  (p) 219
(B) 20C0  20C2  20C4  . . .  20C10  (q) 215
(C) 8C0  32 . 8C1  52 . 8C2  ...  17 2 . 8C8  (r) 211

 10 10
C1 10C2 10
C10 
(D) 11  0 C    ...   1  (s) 89.2
8

 2 3 11 

53. Column - I Column - II


(A) Let f : RR be such that f(1) = 3 & f(1) = 6 (p) 1/2
1/ x
 f (1  x) 
and
lim
x 0    e k , then k is
 f (1) 

lim 1  x  x 2  ex
(B) x 0 = (q) 
x2
xf (x)  sin x 1 f (x)  x 2  1
(C) lim  lim
, then x  0 = (r) 2
x 0 x 2 sin x 3 x sin x
2e1/ x  3e 1/ x
(D) lim  (s) limit does not exist
x  0 e1/ x  e 1/ x

COLLEGES: ANDHERI / BORIVALI / CHEMBUR / DADAR / KALYAN / KHARGHAR / NERUL /


54. Column I Column II

(A) a, b, c, d, e are five numbers in which the first three are in AP (P) G.P.
and the last three are in HP. If the three numbers in the middle are
in GP then the numbers in the odd places are in
(B) Let A1B1C1 , A 2 B2C2 & A3B3C3 be three triangles right angled (Q) A.P.
at C1 , C2 & C3 respectively. Areas of these triangles are in A.P..
and perimeters are equal. Hypotenuse of the three triangles are in
(C) If a, b, c are in A.P. and a, c, b are in G.P.,  a  b  c  , (R) H.P.
then c, a, b are in
(D) If the roots of the following equation (S) none
cx ax 1
  a  c
x   a  b  x   b  c  x   a  b    x   b  c 
are equal in magnitude, real and are of opposite sign,
then a, b, c are in

   
55. Consider the A(a) ; B(b) ; C(c) and D(d) x is the distance of the point A from the plane BCD. y is
the distance of the point D from the plane ABC
Column I Column II
             
(A) | b  c  c  d  d  b | x  [b c d] equals (p) [a b c]  [b d c]  [d a c]
                
(B) | a  b  b  c  c  a | y  [a b c] equals (q) [a b c]  [a c d]  [a d b]
         
(C) [a b c] is equal to (r) [d a b]  [d b c]  [d c a]
(when the points A, B, C and D are coplanar)
         
(D) [d a b] is equal to (s) [b c d]  [c a d]  [d a b]
(when the points A, B, Cand D are coplanar)

56. Column-I Column-II


(A) Four different movies are running in a town. Ten students go to (p) 11
watch these four movies. The number of ways in which every
movie is watched by atleast one student, is (Assume each way
differs only by number of students watching a movie) (q) 36
(B) Consider 8 vertices of a regular octagon and its centre. If T denotes
the number of triangles and S denotes the number of straight lines
that can be formed with these 9 points then the value of (T – S) equals
(C) In an examination, 5 children were found to have their mobiles in their (r) 52
pocket. The Invigilator fired them and took their mobiles in his
possession. Towards the end of the test, Invigilator randomly returned
their mobiles. The number of ways in which at most two children did (s) 60
not get their own mobiles is
(D) The product of the digits of 3214 is 24. The number of 4 digit
natural numbers such that the product of their digits is 12, is
(E) The number of ways in which a mixed double tennis game can
be arranged from amongst 5 married couple if no husband &
wife plays in the same game, is (t) 84

COLLEGES: ANDHERI / BORIVALI / CHEMBUR / DADAR / KALYAN / KHARGHAR / NERUL /


57. Column I Column II
(A) Two numbers a and b are selected from the set of natural (P) 12/25
2 2
number then the probability that a + b is divisible by 5 is
(B) A coin is biased so that heads is four times as likely to (Q) 9/25
appear as tails. If such a coin is tossed twice, the probability
that head occurs at least once, is
(C) 5 different marbles are placed in 5 different boxes randomly. (R) 2/5
the probability that exactly two boxes remain empty, given each
box can hold any number of marbles.
(D) The probability that an automobile will be stolen and found within one (S) 24/25
week is 0.0006. The probability that an automobile will be stolen is
0.0015. Probability that a stolen automobile will be found in one week is

58. Column I Column II


(a) Length of latus rectum of the ellipse (p) 5
3x 2  4y 2  6x  16y  7  0 , is
x 2 y2
(b) Minimum distance between   1& (q) 4
8 32
x 2  y 2  12x  12y  67  0 , is
(c) Area of triangle formed by three points on the ellipse (r) 3
2
x
 y 2  1 ,whose eccentric angle differ by 1200, is
16
x 2 y2
(d) Radius of the director circle of   1 , is (s) 3 3
6 10

59. COLUMN – I COLUMN – II


(A) The chord of contact of the pair of tangents drawn from (P) 2
each point on the line 2x + y = 4 to the parabola
y2 = - 4x passes through a fixed point, whose distance
from the y-axis is
(B) Number of common tangents to the parabolas (Q) 3
y  x 2  5 x  6 & y  x 2  x  1 is
(C) Number of distinct normals drawn to y2 = 4x, from (R) 1/2
the point (11/4, 1/4) is
(D) Let S be the focus and PQ be a focal chord of the parabola
y 2  4ax , such that SP & SQ are the roots of the equation (S) 1
t 2  4t  2  0 , then a =

60. Match the following


Column-I Column-II
(A) The number of ways in which 9 mangoes can be (p) 81
distributed among 3 boys such that each can have
any number of mangoes
(B) The number of triangles whose vertices are at the vertices (q) 55
of a decagon with exactly one side common with the decagon
(C) The number of zeroes at the end of 85! is (r) 60
(D) The number of positive integral solutions of x1x2x3 = 210 are (s) 45

COLLEGES: ANDHERI / BORIVALI / CHEMBUR / DADAR / KALYAN / KHARGHAR / NERUL /


61. Column I Column II
(A) If the sum of the slopes of the normals from a point P on (P) circle
hyperbola xy = 4 is constant 2, then the locus of P is
(B) PN is the perpendicular from a point P on x2 – y2 = a2 on any of its (Q) hyperbola
asymptotes, then the locus of mid-point of PN is
(C) Tangents are drwan to x 2  y 2  a 2 from a point P. If there (R) pair of lines

x 2 y2
chord of contact touches   1 . Locus of P is
a 2 b2
(D) Locus of mid point of segment of any tangent intercepted (S) none

62. Column I Column II


(A) If A   aij  and aij  i 2  j 2 , then A is (p) singular
33

(B) A   aij  and aij  3i  j , then A is (q) skew – symmetric


  33

(C) A   aij  and aij  i 2  j 2 , then A is (r) symmetric


  33

 2 2 4 
(D) A   1 3 4  , then A is (s) idempotent
 1 2 3

COLLEGES: ANDHERI / BORIVALI / CHEMBUR / DADAR / KALYAN / KHARGHAR / NERUL /


ANSW ER KEY
1. (A) - (q), (B) - (p), (C) - (s), (D) - (r)
2. (A)  (P); (B)  (Q); (C)  (R); (D)  (S)
3. (A)  (S); (B)  (P); (C)  (R); (D)  (Q)
4. A - R, B - PQ, C - S, D - PQ
5. (A)  (P); (B)  (Q); (C)  (R); (D)  (S)
6. (A)  (S); (B)  (Q); (C)  (P); (D)  (R)
7. (A)  (Q); (B)  (Q); (C)  (R); (D)  (S)
8. (A)  (QRS); (B)  (PRS); (C)  (PQRS); (D)  (PQS)
9. A - P, B - Q, C - R, D - S.
10. (A)  (P); (B)  (P); (C)  (Q); (D)  (S)
11. (A)  (R); (B)  (Q); (C)  (S); (D)  (P)
12. (A)  (P); (B)  (P); (C)  (Q); (D)  (S)
13. (A)  (Q); (B)  (R); (C)  (P); (D)  (S)
14. A - q, B - p, C - s, D - r
15.  A    q  ;  B   p  ;  C    r  ;  D    s 
16. (A) - (p,s), (B) - (q,r,s), (C) - (p), (D) - (p,q,r,s)
17. (A)  (Q); (B)  (R); (C)  (S); (D)  (R)
18. (A)  (R); (B)  (Q); (C)  (P); (D)  (S)
19. A - R, B - Q, C - P, D - Q
20. A - q, B - s, C - p, D - r
21. (A) - (s), (B) - (r), (C) - (q), (D) - (p)
22. (A)  (Q); (B)  (R); (C)  (P); (D)  (Q)
23. (A)  (P); (B)  (P); (C)  (Q); (D)  (R)
24. (A)  (Q); (B)  (S); (C)  (P); (D)  (R)
25. (A)  (R); (B)  (Q); (C)  (P); (D)  (S)
26. (A)  (P); (B)  (Q); (C)  (R);
27. (A)  (R); (B)  (P); (C)  (Q); (D)  (S)
28. (A) - (q), (B) - (r), (C) - (p), (D) - (s)
29. (A)  (Q); (B)  (P); (C)  (R)
30. (A)  (Q); (B)  (R); (C)  (S); (D)  (P)
31. (A - s, B - p, C - r, D - q)
32. (A)  (R); (B)  (P); (C)  S); (D)  (Q)
33. (A)  (Q); (B)  (R); (C)  (S); (D)  (P)
34.  A    q  ;  B   s  ;  C    r  ;  D   q 
35. (A)  (QS); (B)  (QRS); (C)  (S); (D)  (P)
36. (A)  (Q); (B)  (P); (C)  (S); (D)  (R)
37. A - Q, B - S, C - P, D - R
38. (A)  (Q); (B)  (Q); (C)  (R); (D)  (P)
39. (A)  (R); (B)  (Q); (C)  (P); (D)  (S)
40. (A) - (r), (B) - (q), (C) - (p), (D) - (s)
41. A - r, B - q, C - p, D - s
42. (A)  (Q); (B)  (R); (C)  (P); (D)  (S)
43. (A) - (p), (B) - (s), (C) - (q), (D) - (r)
44. (A)  (R); (B)  (P); (C)  (R); (D)  (Q)
45. (A)  (QRS); (B)  (P); (C)  (S); (D)  (S)
46. (A)  (S); (B)  (Q); (C)  (R); (D)  (S)
47. (A)  (Q); (B)  (P); (C)  (R); (D)  (S)
48. (A)  (S); (B)  (R); (C)  (P); (D)  (Q)
49. (A)  (Q); (B)  (R); (C)  (S); (D)  (P)

COLLEGES: ANDHERI / BORIVALI / CHEMBUR / DADAR / KALYAN / KHARGHAR / NERUL /


50. (A)  (Q); (B)  (P,S); (C)  (S); (D)  (Q,S)
51.  A    p  ;  B   q  ;  C    r 
52. A - q, B - p, C - s, D - r
53. (A)  (R); (B)  (P); (C)  (Q); (D)  (S)
54. (A)  (P); (B)  (Q); (C)  (S); (D)  (R)
55. (A)  q; (B)  r; (C)  s; (D)  p
56. A - t, B - r, C - p, D - q, E - s
57. (A) - (q), (B) - (s), (C) - (p), (D) - (r)
58. (A)  (R); (B)  (P); (C)  (S); (D)  (Q)
59. (A)  (P); (B)  (S); (C)  (Q); (D)  (R)
60. A - q, B - r, C - s, D - p
61. (A)  (R); (B)  (Q); (C)  (Q); (D)  (Q)
62. A - r, B - p, C - q, D - p

COLLEGES: ANDHERI / BORIVALI / CHEMBUR / DADAR / KALYAN / KHARGHAR / NERUL /

You might also like